You are on page 1of 327

em Matemtica em Rede Nacional

Mestrado Profissional

Iniciao Matemtica

Autores: Krerley Oliveira Adn J. Corcho

Unidade I: Captulos I e II

Dedicamos este livro as nossas esposas e lhos, que compreenderam os sbados sacricados em funo de escrev-lo e a nossos pais, por tudo o que eles representam.

Tente! E no diga que a vitria est perdida. Se de batalhas que se vive a vida. Tente outra vez! (Raul Seixas)

vi

Sumrio
Prefcio 1 Primeiros Passos
1.1 1.2 1.3 Organizando as Ideias Verdadeiro ou Falso? . . . . . . . . . . . . . . . . . . . . . . . . . . . . . . . . . . . . .

xi 1
1 5 9 10 15 18 26

Teoremas e Demonstraes . . . . . . . . . . . . . . . . 1.3.1 Mtodos de Demonstrao . . . . . . . . . . . . . . . . . . . .

1.4 1.5 1.6

Algumas Dicas para Resolver Problemas Solues dos Problemas da Seo 1.4 Exerccios

. . . . . . . . . .

. . . . . . . . . . . . . . . . . . . . . . . . .

2 Equaes e Inequaes
2.1 Equaes do Primeiro Grau 2.1.1 2.2 . . . . . . . . . . . . . . . Problemas Resolvidos . . . . . . . . . . . . . . . . . . . . . . .

31
33 37 42 46 49 50 55 59 60

Sistemas de Equaes do Primeiro Grau 2.2.1

Problemas Resolvidos . . . . . . . . . . . . . . .

2.3

Equao do Segundo Grau . . . . . . . . . . . . . . . . 2.3.1 2.3.2 2.3.3 2.3.4 Completando Quadrados . . . . . . . . . . . . . Relao entre Coecientes e Razes Equaes Biquadradas O Mtodo de Viti . . . . . . .

. . . . . . . . . . . . . .

. . . . . . . . . . . . . . . .

vii

viii

SUMRIO

2.4 2.5 2.6

Inequaes . . . . . . . . . . . . . . . . . . . . . . . . . Inequao do Primeiro Grau . . . . . . . . . . . . . . . Inequao do Segundo Grau 2.6.1 . . . . . . . . . . . . . . . .

62 63 69 75 77 77 80 81

Mximos e Mnimos das Funes Quadrticas

2.7

Miscelnea . . . . . . . . . . . . . . . . . . . . . . . . . 2.7.1 2.7.2 Equaes Modulares . . . . . . . . . . . . . . . . . . . .

Um Sistema de Equaes No lineares

2.8

Exerccios

. . . . . . . . . . . . . . . . . . . . . . . . .

3 Divisibilidade
3.1 3.2 3.3 Conceitos Fundamentais e Diviso Euclidiana Bases Numricas . . . . . . . . . . . . . . . . . . . . . . . . . .

89
90 99

Mximo Divisor Comum e Mnimo Mltiplo Comum 3.3.1 3.3.2 3.3.3 3.3.4

. 106

Mximo Divisor Comum . . . . . . . . . . . . . 106 Algoritmo de Euclides . . . . . . . . . . . . . . 111 . . . . . . . . . . . . 115

Mnimo Mltiplo Comum

Equaes Diofantinas Lineares . . . . . . . . . . 120

3.4 3.5

Nmeros Primos e Compostos . . . . . . . . . . . . . . 123 Procurando Primos . . . . . . . . . . . . . . . . . . . . 127 3.5.1 3.5.2 3.5.3 O Crivo de Eratstenes . . . . . . . . . . . . . . 127 Primos de Mersenne . . . . . . . . . . . . . . . 129

O Teorema Fundamental da Aritmtica . . . . . 133 . . . . . . . . . . . . . . . . . . . . . . . . . 139

3.6

Exerccios

4 O Princpio da Casa dos Pombos


4.1 4.2 4.3 4.4

143

Primeiros Exemplos . . . . . . . . . . . . . . . . . . . . 145 Uma Verso mais Geral . . . . . . . . . . . . . . . . . . 146 Aplicaes na Teoria dos Nmeros . . . . . . . . . . . . 149 Aplicaes Geomtricas . . . . . . . . . . . . . . . . . . 151

SUMRIO

ix

4.5 4.6

Miscelnea . . . . . . . . . . . . . . . . . . . . . . . . . 153 Exerccios . . . . . . . . . . . . . . . . . . . . . . . . . 157

5 Contagem
5.1 5.2 5.3 5.4 5.5 5.6 5.7 5.8 5.9 Princpio Aditivo da Contagem

161
. . . . . . . . . . . . . 162

Princpio Multiplicativo de Contagem . . . . . . . . . . 170 Uso Simultneo dos Princpios Aditivo e Multiplicativo 178

Permutaes Simples . . . . . . . . . . . . . . . . . . . 181 Arranjos Simples . . . . . . . . . . . . . . . . . . . . . 184

Combinaes Simples . . . . . . . . . . . . . . . . . . . 188 O Binmio de Newton . . . . . . . . . . . . . . . . . . 193 . . . . . . . . . . . . . . . 195

Contagem e Probabilidades Exerccios Propostos

. . . . . . . . . . . . . . . . . . . 197

6 Induo Matemtica
6.1 6.2 Formulao Matemtica

203
. . . . . . . . . . . . . . . . . 204

Aplicaes . . . . . . . . . . . . . . . . . . . . . . . . . 206 6.2.1 6.2.2 6.2.3 Demonstrando Identidades . . . . . . . . . . . . 206 Demonstrando Desigualdades . . . . . . . . . . 210 . . . . . 212

Induo e Problemas de Divisibilidade

6.3 6.4

Induo na Geometria

. . . . . . . . . . . . . . . . . . 215

Miscelnea . . . . . . . . . . . . . . . . . . . . . . . . . 220 6.4.1 Cuidados ao Usar o Princpio da Induo . . . . 222

6.5 6.6

Induo e Recorrncias . . . . . . . . . . . . . . . . . . 222 Exerccios . . . . . . . . . . . . . . . . . . . . . . . . . 229

7 Desigualdades
7.1 7.2 Desigualdade Triangular

233
. . . . . . . . . . . . . . . . . 234

Desigualdade das Mdias . . . . . . . . . . . . . . . . . 238

SUMRIO

7.3 7.4 7.5

Desigualdade de Cauchy-Schwarz

. . . . . . . . . . . . 245

Desigualdade de Jensen . . . . . . . . . . . . . . . . . . 246 Exerccios . . . . . . . . . . . . . . . . . . . . . . . . . 250

8 Polinmios
8.1 8.2 Algoritmo de Euclides

255
. . . . . . . . . . . . . . . . . . 263 . . . . . . . . 268 . . 269

Operaes com Polinmios . . . . . . . . . . . . . . . . 255

8.3 Sempre Existem Razes de um Polinmio? 8.3.1 8.4

Nmeros Complexos e Razes de Polinmios

Exerccios

. . . . . . . . . . . . . . . . . . . . . . . . . 272

A Apndice: Funes Referncias

279 285

Prefcio

Neste livro pretendemos oferecer ao leitor uma introduo Matemtica Elementar. Juntando as experincias didticas vividas pelos

autores individualmente no Brasil e em Cuba, e mais alguns anos juntos como treinadores de projetos de introduo Matemtica no estado de Alagoas, esperamos tornar para o leitor a Matemtica mais interessante, mostrando um pouco do imenso brilho e beleza que ela esconde. O livro foi escrito em captulos, cada um deles detalhando um tema central e trazendo alguns teoremas fundamentais. Com muitos exemplos e aplicaes dos conceitos introduzidos, pretendemos mostrar ao leitor a importncia do assunto abordado. A organizao dos exemplos tenta seguir uma linha em ordem crescente de diculdade e, para o melhor aproveitamento do livro, o trabalho com os exerccios parte fundamental. Ler o enunciado e resolver o maior nmero pos-

xi

xii

Prefcio

svel de exerccios imperativo.

Como j disse o Prof.

Elon Lima,

Matemtica no se aprende passivamente. Os exemplos e aplicaes dos conceitos, bem como os teoremas, devem ser lidos com cuidado e muita ateno. Para os estudantes

que desejem treinar para olimpadas de Matemtica, sugerimos que formem grupos de estudo para trabalhar os temas individualmente, sob a orientao de um professor. Acreditamos que o texto pode ser utilizado em uma disciplina elementar num curso de licenciatura ou bacharelado em Matemtica. O primeiro captulo para introduzir o leitor no esprito do livro e dar uma amostra do tipo de problemas e material que seguir nos demais captulos. So propostos alguns problemas, muitos deles com solues, e discutimos alguns mtodos importantes para uso no dia a dia dos estudantes. Nesta discusso inclumos o estudo de proposies matemticas, provas por contraposio, o mtodo de reduo ao absurdo e algumas outras regras bsicas e cuidados que devemos ter ao resolver problemas em Matemtica. Em seguida, estudamos as equaes do primeiro e do segundo grau. Estudamos os mtodos de resoluo dessas equaes, sistemas de equaes, relaes entre razes e coecientes, bem como alguns problemas interessantes que podem ser solucionados via essas equaes. Em seguida, estudamos inequaes do primeiro e do segundo grau. O captulo seguinte trata do conceito de divisibilidade . Tentamos introduzir o leitor nos principais aspectos bsicos, incluindo-se a divisibilidade com resto, mximo divisor comum e mnimo mltiplo comum, nmeros primos e compostos, e um pouco de equaes diofantinas lineares. Um captulo til para o estudante que deseja participar de Olim-

Prefcio

xiii

padas de Matemtica o que trata do princpio da casa dos pombos . Este captulo um belo exemplo de como algo aparentemente ingnuo pode gerar consequncias interessantes. Alguns dos exemplos esto

conectados com os captulos anteriores e aparentemente aplicam o princpio de modo inusitado, em problemas de geometria, teoria dos nmeros e em reas diversas. No captulo de contagem, comeamos com noes teis sobre conjuntos e princpios bsicos para contar os elementos de um conjunto. Nesse captulo, estamos mais preocupados com as aplicaes imediatas do assunto, sugerindo alguns problemas para o estudante iniciante. Seguimos discutindo os tipos de agrupamento de elementos e suas consequncias. Obtemos o binmio de Newton e introduzimos a noo de probabilidade de um conjunto, resolvendo alguns problemas relacionados. Em seguida, estudante se depara com uma arma poderosa do matemtico. O mtodo da induo nita estudado procurando conectar esta noo com os captulos anteriores, reobtendo com o auxlio do mtodo da induo algumas coisas que j foram deduzidas por outros mtodos. Vrios exemplos e problemas so resolvidos, alguns deles de modo surpreendente e inesperado. No prximo captulo, introduzimos algumas desigualdades populares para o uso do estudante. Algumas dessas desigualdades so muito importantes no estudo mais profundo da Matemtica e no aparecem em cursos introdutrios, apesar de suas provas e aplicaes serem elementares. Todas as desigualdades aparecem com demonstraes,

em muito dos casos utilizando-se lgebra elementar e o mtodo de induo nita. So apresentados vrios exemplos que mostram a utilidade dessas desigualdades em alguns problemas prticos. Para xar

xiv

Prefcio

o conhecimento, propomos vrios exerccios complementares. Alguns deles, cuja soluo mais elaborada, so sugeridos. No ltimo ca-

ptulo, estudamos um pouco as propriedades gerais dos polinmios. Para complementar a formao do leitor menos experiente, inclumos um apndice sobre funes. Somos gratos a muitas pessoas que colaboraram com a elaborao deste livro com sugestes e correes em verses iniciais. Entre eles, citamos: Carlos Gustavo Moreira, Ali Tahzibi, Feliciano Vitrio, Eduardo Teixeira, Chico Potiguar e vrios de nossos alunos de Iniciao Cientca e mestrado, que por vrias ocasies deram sugestes para a melhoria do texto. Um agradecimento especial vai para Fernando Echaiz, que nos ajudou ativamente nas notas do Captulo 5 que originaram este texto. Finalmente, agradecemos aos revisores pela leitura cuidadosa e ao comit editorial da SBM, na pessoa da profa. Helena Lopes, pelo excelente trabalho de editorao.

Macei, Abril de 2010

Krerley Oliveira Adn J. Corcho

1
Primeiros Passos

Neste captulo, discutiremos algumas ideias gerais e convenes que serviro como base para os diferentes mtodos de resoluo de problemas que trataremos nos captulos seguintes. Alguns dos exemplos que abordamos sero teis para orientar quanto ao cuidado que devemos ter quando discutimos problemas em Matemtica.

1.1

Organizando as Ideias

Para resolver problemas matemticos precisamos ter bem claro o que devemos provar e o que estamos assumindo como verdade. sobre

isso que falaremos agora. Comearemos observando as seguintes armaes:

Primeiros Passos

(a) A soma de dois nmeros pares sempre um nmero par.

(b) Todo brasileiro carioca.

(c) A terra um planeta.

(d) Se

o comprimento da diagonal de um retngulo de lados

b,
(e) Se

ento

c 2 = a2 + b 2 .
ento

a < 1,

a2 > a.

Todas as armaes acima se encaixam no conceito de proposio, que damos a seguir. Uma proposio ou sentena uma frase armativa em forma de orao, com sujeito, verbo e predicado, que ou falsa ou verdadeira, sem dar lugar a uma terceira alternativa. Por exemplo, as proposies (a) e (c) so claramente verdadeiras; mais adiante nos convenceremos da veracidade da proposio (d). Por outro lado, as proposies (b) e (e) so falsas. Com efeito, para constatar a veracidade da sentena (b) teramos que checar o registro de nascimento de cada brasileiro e vericar se nasceu no Rio de Janeiro, mas isto falso pois o conhecido escritor Graciliano Ramos um brasileiro nascido em Alagoas. Analogamente, para convencer-nos de que a proposio (e) falsa basta tomar

a = 1/2 e checar que (1/2)2 =

1/4

no maior do que

1/2

como a sentena arma. Em ambos os

casos temos vericado que as proposies (b) e (e) so falsas apresentando casos particulares onde as mesmas deixam de valer. Estes casos particulares so chamados de contraexemplos e so muito teis para vericar a falsidade de algumas proposies. Notemos que as proposies (d) e (e) so do tipo:

1.1

Organizando as Ideias

Se onde

P,

ento

Q,

tambm so sentenas. Por exemplo, na proposio (e)

temos que:

P: c

o comprimento da diagonal de um retngulo de lados

b,

Q: c2 = a2 + b2 ,
ou seja, estamos assumindo que mos vericar se

verdade e usando este fato deve-

verdade ou no.

Uma proposio condicional ou implicativa uma nova proposio formada a partir de duas proposies Se

Q,

que escrita na forma:

P,

ento

Q

ou

P

implica

Q,

onde para o ltimo caso usamos a notao: a proposio

de hiptese e a proposio

P = Q. Chamaremos Q de tese. A hiptese

tambm chamada de proposio antecedente e a tese, de proposio

consequente.
Por exemplo, na proposio condicional (f ) a hiptese : tese :

a<1

e a

a > a.

A partir de uma de uma proposio condicional podem-se gerar novas proposies que so de especial interesse para os matemticos. Vamos chamar o modo em que apresentamos uma proposio de forma

positiva. Por exemplo, quando enunciamos a proposio


Se como laranja, ento gosto de frutas, assumimos esta armao como sua forma positiva. Vamos descrever agora como podemos obter novas proposies a partir desta.

Primeiros Passos

Forma recproca de uma proposio condicional:


ptese pela proposio consequente e vice-versa. exemplo:

para cons-

truirmos a forma recproca, temos que trocar na forma positiva a hiVejamos em nosso

Forma da proposio Positiva Recproca

Hiptese
como laranja gosto de frutas

Tese
gosto de frutas como laranja

Assim, a recproca de proposio de nosso exemplo ento: Se gosto de frutas, ento como laranja

Forma contrapositiva de uma proposio condicional:

Para

obtermos a forma contrapositiva a partir da forma positiva de uma proposio condicional podemos fazer primeiro sua forma recproca e em seguida negamos as sentenas antecedente e consequente da recproca ou, tambm, podemos primeiro negar as sentenas antecedente e consequente da forma positiva e imediatamente fazer a forma recproca desta ltima. A forma contrapositiva tambm conhecida como forma contrarrecproca . Usando novamente nosso exemplo temos que:

Forma da Proposio Positiva Recproca Contrapositiva

Hiptese
como laranja gosto de frutas no gosto de frutas

Tese
gosto de frutas como laranja no como laranja

Portanto, a forma contrapositiva escreve-se assim:

1.2

Verdadeiro ou Falso?

Se no gosto de fruta, ento no como laranja

Em particular, a forma contrapositiva de uma proposio poder ser, eventualmente, uma forma indireta muito ecaz de vericar resultados em Matemtica.

1.2

Verdadeiro ou Falso?

Uma das coisas que distingue a Matemtica das demais cincias naturais o fato de que um tema de Matemtica discutido utilizando-se a lgica pura e, por conta disso, uma proposio em Matemtica, uma vez comprovada sua veracidade, aceita como verdade irrefutvel e permanecer assim atravs dos sculos. Por exemplo, at hoje usamos o teorema de Tales do mesmo modo que foi usado antes de Cristo e este fato continuar valendo eternamente. Vamos ilustrar melhor essa diferena com um exemplo em Geograa. Hoje, todos ns sabemos que a Terra tem aproximadamente Porm,

o formato de uma laranja, um pouco achatada nos polos.

na poca de Pitgoras, um dos grandes temores dos navegadores era encontrar o m do mundo. No pensamento de alguns destes aventureiros, a Terra tinha o formato de um cubo, e uma vez chegando em um dos seus extremos, o navio despencaria no vazio. Esse um dos muitos exemplos de como a concepo da natureza mudou ao longo do tempo, transformando uma concepo verdadeira num perodo da humanidade em algo completamente falso em outra poca. Porm,

para nossa felicidade, isso no acontece na Matemtica. Uma proposio matemtica ou verdadeira ou falsa e permanecer assim para sempre.

Primeiros Passos

Mas como saber se uma proposio verdadeira ou falsa? A primeira coisa que devemos fazer tomar muito cuidado. As aparncias enganam ou, como diziam nossos avs, nem tudo que reluz ouro. O leitor, avisado disso, pense agora na seguinte pergunta:

Pergunta 1:

Qual a chance de que pelo menos duas pessoas num

nibus com 44 passageiros faam aniversrio no mesmo dia do ano? Como j avisamos, o leitor deve ter cuidado ao responder pergunta acima, pois podemos nos enganar muito facilmente. Por exemplo, podemos formular o seguinte argumento errado: o ano tem 365 dias e, como estou escolhendo um grupo de 44 (nmero muito pequeno com respeito a 365) pessoas ao acaso, claro que podemos responder pergunta com a seguinte armao:

Resposta intuitiva:

A chance de que num grupo de 44 pessoas pelo

menos duas delas faam aniversrio no mesmo dia do ano pequena. primeira vista a resposta dada pode at parecer verdadeira, mas com uma anlise mais cuidadosa veremos que completamente falsa. Na verdade, a chance de que pelo menos duas pessoas do nibus faam aniversrio no mesmo dia do ano de cerca de 93%! Quem no acreditar nisto pode fazer duas coisas: primeiro, ir a sua sala de aula ou no seu nibus escolar, que deve ter pelo menos 44 pessoas, e fazer o experimento ao vivo. Muito provavelmente voc deve conseguir duas pessoas que fazem aniversrio no mesmo dia do ano. Se voc verica que existem duas pessoas que fazem aniversrio no mesmo dia do ano, no por acaso, pois a chance de isso acontecer muito alta. Mas, cuidado! Isso no uma prova matemtica para este fato. Para provar que este fato verdadeiro voc deve vericar que se escolhermos ao acaso um grupo de 44 pessoas ento com aproxi-

1.2

Verdadeiro ou Falso?

madamente 93% de chance, pelo menos duas delas fazem aniversrio no mesmo dia do ano! Porm, se voc faz o experimento e no encontra duas pessoas que fazem aniversrio no mesmo dia do ano (voc seria muito azarado!), no se desespere. Lembre-se de que se trata de algo que acontece com

chance de 93% e que pode no acontecer quando fazemos um teste.


Em qualquer um dos casos, para ter a certeza de que a proposio verdadeira o leitor deve demonstr-la. Captulo Vamos analisar agora outro fato aparentemente bvio. Faremos isso no nal do

Pergunta 2:

Num campeonato de futebol onde cada time joga a

mesma quantidade de jogos, cada vitria vale trs pontos, o empate vale um ponto e a derrota nenhum ponto. Em caso de empate, o

critrio de desempate entre as equipes era o seguinte:

A melhor equipe aquela que tem mais vitrias.

Os organizadores decidiram passar a adotar o critrio a seguir:

A melhor equipe aquela que tem mais derrotas.

Voc acha que este ltimo critrio adotado justo? Com respeito a esta pergunta, o leitor deve ter respondido do seguinte modo:

Resposta:

Um time que perdeu mais pior que um que perdeu me-

nos; portanto, a mudana de critrio totalmente injusta. Acertamos a sua resposta? Na verdade, no houve mudana nenhuma de critrio, ou seja, ambos os critrios nos conduzem ao mesmo ganhador.

Primeiros Passos

Para ver isso rapidamente, lembre-se de que se a equipe mais que a equipe

perdeu

e ainda assim empataram, ento ela deve ter

ganho mais, para que no m do campeonato a equipe conseguisse empatar com a equipe mente. Sejam

ainda assim

B.

Vamos mostrar isso precisa-

d1 , e1 , v1

o nmero de derrotas, empates e vitrias,

respectivamente, da equipe

A. A

Do mesmo modo, sejam

d2 , e2 , v2

nmero de derrotas, empates e vitrias, respectivamente, da equipe

B. B,

Suponhamos que a equipe ou seja, que

obteve mais vitrias do que a equipe

v1 > v2 .

Como cada equipe jogou o mesmo nmero de

jogos, temos que

d1 + e1 + v1 = d2 + e2 + v2 .

(1.1)

Por outro lado, note que o nmero de pontos obtidos pela equipe

e1 + 3v1 . B igual

Do mesmo modo, o nmero de pontos obtidos pela equipe a

e2 + 3v2 .

Como as duas empataram, temos que:

e1 + 3v1 = e2 + 3v2 .
Ou ainda,

3(v1 v2 ) = e2 e1
Como temos que:

ou

v2 v1 =

e2 e1 . 3

v1 v2 > 0, temos que e2 e1 > 0.

Reescrevendo a equao (1.1),

d1 d2 = e2 e1 + (v2 v1 ) = e2 e1
Logo, temos que

e2 e1 2 = (e2 e1 ). 3 3

teve mais

d1 d2 > 0, pois e2 e1 > 0. Isso signica que derrotas que B ; logo, qualquer um dos dois critrios de

desempate usado nos leva equipe vencedora.

1.3

Teoremas e Demonstraes

Assim, como estes dois exemplos mostram, ao depararmos com um problema em Matemtica, devemos ter cuidado ao tirar concluses apressadas para evitar que cometamos algum engano. Pode acontecer que uma situao que claramente falsa para um observador menos atento, se mostre verdadeira quando fazemos uma anlise mais criteriosa.

1.3

Teoremas e Demonstraes

Agora denimos o que entendemos por demonstrao matemtica de uma proposio. Uma demonstrao em Matemtica o processo de raciocnio lgico e dedutivo para checar a veracidade de uma proposio condicional. Nesse processo so usados argumentos vlidos, ou seja, aqueles que concluam armaes verdadeiras a partir de fatos que tambm so verdadeiros. Como exemplo de demonstrao citamos a argumentao usada para mostrar na segunda pergunta da seo anterior que os critrios de desempate eram similares. Sempre que, via uma demonstrao, comprovemos a veracidade de uma proposio passamos ento a chamar esta de teorema. Assim, um teorema qualquer armao que possa ser vericada mediante uma demonstrao. Alguns teoremas se apresentam na forma de uma proposio con-

dicional, isto , uma sentena do tipo Se


da forma  P e a sentena

= Q. Nesse caso, a Q denominada de tese.

P , ento Q ou implicativa sentena P chamada de hiptese


Ou seja, a validade da hiptese

nos implica a veracidade da tese.

10

Primeiros Passos

Um exemplo de teorema o famoso teorema de Pitgoras , cujo enunciado diz o seguinte:

Teorema 1.1

(Teorema de Pitgoras)

Num tringulo retngulo a

soma dos quadrados dos catetos igual ao quadrado da hipotenusa.

Notemos que o teorema de Pitgoras no est enunciado na forma condicional, mas pode ser reescrito nessa forma como:

Teorema 1.2 (Teorema de Pitgoras) . Se T


de catetos

um tringulo retngulo
2

e hipotenusa

c,

ento

c = a + b2 .

Observao 1.3.

Em geral, mais comum usar a palavra teorema

apenas para certas proposies que so de grande importncia matemtica, chamando-se simplesmente de proposio ao resto das proposies verdadeiras que admitem uma demonstrao. Para uma discusso mais detalhada, recomendamos [8].

1.3.1 Mtodos de Demonstrao


Quando realizamos uma demonstrao no existe um caminho nico. Dependendo do problema em questo podemos usar mtodos diferentes. A seguir ilustramos os seguintes trs mtodos:

Demonstrao direta.

Demonstrao por contraposio.

Demonstrao por reduo ao absurdo.

1.3

Teoremas e Demonstraes

11

Demonstrao Direta
A demonstrao direta aquela em que assumimos a hiptese como verdadeira e atravs de uma srie de argumentos verdadeiros e dedues lgicas conclumos a veracidade da tese.

a b Q a c

a b

Figura 1.1: Figura auxiliar para a demonstrao do teorema de Pitgoras Um exemplo de demonstrao direta a que daremos a seguir, para o teorema de Pitgoras enunciado anteriormente no Teorema 1.1. Com efeito, usando a gura acima temos que a rea do quadrado de lado

a+b

a soma das quatro reas dos tringulos retngulos

congruentes pelo critrio lado-ngulo-lado (de catetos rea do quadriltero

b)

mais a

Q,

o qual um quadrado visto que cada um dos

seus lados coincide com a hipotenusa catetos

dos tringulos retngulos de mede

b e, alm disso, cada um dos seus ngulos internos = 180 ( + ) = 180 90 = 90 (veja a Figura 1.1).
e

Portanto,

(a + b)2 = 4
de onde

ab + c2 , 2

a2 + 2ab + b2 = 2ab + c2 ,
e consequentemente

a2 + b 2 = c 2 ,

12

Primeiros Passos

como queramos.

Demonstrao por Contraposio


Este mtodo baseado no fato de que a veracidade de forma positiva de uma proposio equivalente veracidade de sua forma contrapositiva, podendo ser esta ltima, eventualmente, mais fcil de se provar. Por exemplo, a armao

Se sou alagoano, ento sou brasileiro equivalente armao

Se no sou brasileiro, ento no sou alagoano Por exemplo, provemos a seguinte proposio:

Proposio 1.4.

Hiptese: Tese:

Se

N2

par, ento

par.

N2

par.

par.

Desaamos o leitor a tentar mostrar esta proposio partindo da hiptese e tentando concluir a tese. Note que podemos vericar que nossa proposio verdadeira para vrios valores de

N2

como na tabela a

seguir, mas isso no uma prova matemtica da nossa proposio.

N2 N

4 2

16 4

36 6

64 8

100 10

144 12

1.3

Teoremas e Demonstraes

13

Mesmo vericando para um bilho de valores de restariam nmeros para serem vericados.

N 2,

sempre nos

Como nossas tentativas

de provar a forma positiva dessa proposio esto sendo frustradas, apelaremos para mostrar a forma contrapositiva da mesma, isto :

Proposio 1.5.

Se

no par, ento

N2

no par.

Neste caso, temos:

Hiptese: Tese:

no par.

N2

no par.

Demonstrao. Como estamos assumindo que

N tem que ser mpar, ou seja, existe p, nmero inteiro, tal que N = 2p +1.
no par, logo Logo,

N 2 = (2p + 1)(2p + 1) = 4p2 + 2p + 2p + 1 = 4p2 + 4p + 1 = 2(2p2 + 2p) + 1 = 2q + 1,


onde

q = 2p2 + 2p.

Logo,

N 2 = 2q + 1

mpar e conclumos assim

nossa prova.

Demonstrao por Reduo ao Absurdo


Este mtodo uma das ferramentas mais poderosas da Matemtica. O nome provm do latim reductio ad absurdum e tambm conhecido como mtodo do terceiro excludo devido ao mesmo estar baseado na

14

Primeiros Passos

lei do terceiro excludo que diz o seguinte: uma armao que no


pode ser falsa, dever ser consequentemente verdadeira. De um modo geral, o roteiro que segue uma demonstrao por reduo ao absurdo o seguinte:

Assumimos a validade da hiptese. Supomos que nossa tese falsa. Usando as duas informaes anteriores conclumos, atravs de argumentos verdadeiros, uma armao falsa; como tal fato no poder ocorrer, ento nossa tese dever ser verdadeira.

Vamos mostrar como o mtodo funciona na prtica provando a seguinte proposio:

Proposio 1.6.

Seja

um nmero positivo, ento

x + 1/x 2.

Destaquemos primeiramente a nossa hiptese e a nossa tese.

Hiptese: Tese:

um nmero positivo.

x + 1/x 2. x+
1 x

Demonstrao. Seja
falsa, isto ,

x um nmero positivo e suponhamos que a < 2. Usando que x > 0 e multiplicando por

tese este

a desigualdade anterior, obtemos que

x2 + 1 < 2x. x2 2x + 1 < 0 equivalente a (x 1)2 < 0, j que x2 2x + 1 = (x 1)2 , o que impossvel. Portanto, x + 1/x 2,
Da segue-se que

como desejvamos.

1.4

Algumas Dicas para Resolver Problemas

15

1.4

Algumas Dicas para Resolver Problemas

Nesta seo, damos algumas regras gerais que consideramos importante ter em mente na hora de resolver um problema de Matemtica. Aplicaremos estas regras a alguns problemas interessantes para ilustrar a sua importncia. Elas so:

R1) Ler bem o enunciado do problema e utilizar todas as informaes disponveis. R2) Fazer casos particulares ou casos mais simples de problemas similares, para adquirir familiaridade com o problema. R3) Mudar a representao do problema, transformando-o em um problema equivalente. R4) Usar a imaginao pesquisando caminhos alternativos. polar os limites! Extra-

A seguir propomos vrios problemas onde as regras anteriores so muito teis. O leitor deve tentar resolv-los; mas se no conseguir

achar soluo depois de muito tentar poder ento passar para a prxima seo onde os solucionamos.

Problema 1.7.
(M) (A)

Ao encontrar uma velha amiga (A), durante uma

viagem de trem, um matemtico (M) tem a seguinte conversa:  Como vo os trs lhos da senhora?  Vo bem, obrigada!

16

Primeiros Passos

(M) (A) (M) (A) (M) (A) (M)

 Qual a idade deles mesmo?  Vou lhe dar uma dica. O produto das idades deles 36.  S com essa dica impossvel!  A soma das idades deles igual ao nmero de janelas deste vago.  Ainda no sei!  O mais velho toca piano! Agora eu sei!

Voc capaz de descobrir as idades dos trs lhos da senhora?

Problema 1.8.

Numa cesta encontram-se 9 moedas idnticas, sendo

que 8 delas tm o mesmo peso e uma moeda mais leve que as demais. Usando duas vezes uma balana de dois pratos, encontrar a moeda mais leve.

Problema 1.9.

Numa pequena ilha existem 5 pessoas de olhos azuis

e 5 pessoas de olhos verdes. Existe um grande tabu nesta ilha que o seguinte: se uma pessoa descobre que possui olhos azuis ela se suicida meia-noite do dia em que descobriu, pulando do alto da prefeitura. Por conta disso, ningum conversa sobre o assunto, olha para espelhos ou v seu reexo na gua. Todos se cruzam diariamente e conhecem os olhos de seus amigos. Numa manh, um estrangeiro chegou ilha e reuniu as 10 pessoas para o seguinte pronunciamento: Nesta ilha, existe uma pessoa de olhos azuis. Pergunta-se:

1.4

Algumas Dicas para Resolver Problemas

17

(a) O que aconteceu com os habitantes da ilha? (b) Que informao nova o estrangeiro trouxe?

Problema 1.10.

Um viajante deseja se hospedar durante 31 dias num

hotel. Entretanto, percebe que est sem dinheiro e que a nica coisa que possui uma corrente com 31 elos de ouro. Para pagar sua conta, ele acertou com o gerente pagar um elo por dia, sem atrasar ou adiantar o pagamento, durante os 31 dias. O gerente pode dar troco em elos. Depois ele deseja recuperar a corrente e por isso ele quer pagar a conta cortando a corrente no menor nmero de pedaos. Quantos cortes voc conseguiria dar e pagar a conta?

Problema 1.11.

Sabendo que em cada jogada o movimento do cavalo

consiste em se deslocar duas casas na horizontal e uma na vertical ou duas na vertical e uma na horizontal, decidir se possvel sair da congurao apresentada no tabuleiro (a) e chegar congurao apresentada no tabuleiro (b) da Figura 1.2 sem que em algum momento existam dois cavalos na mesma casa.

Figura 1.2: Cavalos de xadrez

18

Primeiros Passos

Problema 1.12.

Mostre que podemos cobrir os

9 pontos no reticulado

da Figura 1.3 traando 4 segmentos de reta sem tirar o lpis do papel.

Figura 1.3: Reticulado de 9 pontos Sugerimos seguir as dicas abaixo para obter sucesso na soluo dos problemas:

Para os problemas 1.7 e 1.8 use a primeira regra. Para os problemas 1.9 e 1.10 use a segunda regra. Por exemplo, no problema 1.9 fazer primeiro o caso: uma pessoa com olhos azuis e uma com olhos verdes e depois fazer o caso: duas pessoas de olhos azuis e duas de olhos verdes; generalize.

Para os problema 1.11 use a terceira regra. Para o problema 1.12 use a quarta regra.

1.5

Solues dos Problemas da Seo 1.4

A seguir apresentamos solues para os problemas enunciados na seo anterior.

Soluo do Problema 1.7.


o produto das idades 36.

muito importante neste problema tirar

o mximo de informao das dicas da senhora. Vamos primeira dica:

1.5

Solues dos Problemas da Seo 1.4

19

Suponhamos que as idades dos lhos sejam Como

xyz = 36,

temos as seguintes possibilidades para os

36. nmeros x,

z: x y 1 1 1 1 1 2 2 3 z xyz 36 36 36 36 36 36 36 36
Assim,

1 36 2 18 3 12 4 9 6 6 2 9 3 6 3 4

A segunda dica dada pela senhora a soma das idades.

vamos agora calcular todas as possveis somas de acordo com as fatoraes de 36 dadas na tabela anterior:

x y 1 1 1 1

x+y+z 38 21 16 14

1 36 2 18 3 12 4 9 6 9 6 4

1 6 2 2 2 3 3 3

11 10

Sabemos que aps a segunda dica, o matemtico ainda no conseguiu deduzir as idades das crianas.

20

Primeiros Passos

Por que ele no conseguiu? Imagine que o nmero da casa fosse 14. Ora, de acordo com nossa tabela, s existe um terno de nmeros cujo produto 36 e a soma 14, que o terno (1,4,9). Assim, se o nmero da casa fosse 14 o matemtico teria dado a resposta aps a segunda dica. Como ele cou em dvida, olhando a tabela 2, chegamos concluso de que o nmero da casa s pode ser igual a 13. Lembremos a ltima dica: o mais velho toca piano. No incio essa dica parecia intil, mas agora ela fundamental para resolvermos o problema. De fato, como o mais velho toca piano, isso signica que existe um mais velho, o que descarta o caso (1,6,6). Assim, as idades so 2, 2, e 9.

Soluo do Problema 1.8.

Este o tipo de problema que a primeira

vista pode parecer difcil, mas que quando usamos todas as informaes do seu enunciado se torna fcil. A ideia dividir as moedas em trs grupos de trs moedas cada, que chamaremos grupos Colocaremos na balana os grupos Podem acontecer duas coisas:

A, B A e B e deixaremos o grupo C

C.

fora.

(a) Os pratos cam equilibrados. (b) Os pratos cam desequilibrados.

A e B tm o mesmo peso. Logo, a moeda mais leve deve estar no grupo C . No caso (b), um dos grupos
No caso (a), temos que os grupos cou mais leve, o que signica que a moeda mais leve est neste grupo. Assim, utilizando a balana apenas uma vez conseguiremos descobrir qual o grupo em que a moeda mais leve est. Digamos que este grupo seja o grupo

A.

Para achar a moeda mais leve, procedemos de modo

semelhante ao que zemos anteriormente: separamos as trs moedas

1.5

Solues dos Problemas da Seo 1.4

21

do grupo

colocando uma em cada prato e deixando a terceira de

fora. Podem acontecer duas coisas:

(a) Os pratos cam desequilibrados e assim a moeda mais leve est no prato mais leve. (b) Os pratos cam equilibrados, logo a moeda mais leve foi a que cou fora.

No nal, usamos a balana exatamente duas vezes.

Soluo do Problema 1.9.


na soluo.

Como em muitos problemas de Mate-

mtica, abordar casos mais simples do problema pode ajudar bastante Assim, vamos imaginar o seguinte caso mais simples:

na ilha existe somente uma pessoa de olhos azuis e a outra de olhos verdes. Pensando neste caso, a pessoa que tinha olhos azuis s via as que tinham olhos verdes. Quando o estrangeiro armou que existia

uma pessoa de olhos azuis, ela descobriu que tinha olhos azuis, pois as outras pessoas tinham olhos verdes. Assim, meia-noite ela subiu na prefeitura e pulou. Com isso, a pessoa que tinha olhos verdes descobriu que tinha olhos verdes, pois se ela tivesse olhos azuis sua companheira no se suicidaria no dia anterior. Vamos agora dar um passo crucial na soluo do nosso problema original, considerando o caso onde existem duas pessoas de olhos azuis e duas pessoas de olhos verdes na ilha. Vamos chamar as pessoas de olhos azuis de

e as pessoas de olhos verdes de

D.

No dia

em que o estrangeiro fez o seu pronunciamento, nada aconteceu, pois as pessoas

via a pessoa

D B

viam as pessoas

com olhos azuis e a pessoa

com olhos azuis e vice-versa. J no segundo dia, a

pessoa

teve o seguinte pensamento:

22

Primeiros Passos

Se eu tivesse olhos verdes, a pessoa B teria descoberto que tinha olhos azuis ontem, pois ela veria trs pessoas de olhos verdes. Como ela no se suicidou ontem, eu tenho olhos azuis.
Pensando da mesma forma, a pessoa

descobriu que tambm tinha

olhos azuis. Por isso, meia-noite do segundo dia, as pessoas se suicidaram. O que aconteceu depois? As pessoas

ainda tinham a dvida

da cor de seus olhos. Para chegar concluso de que seus olhos so verdes, no terceiro dia, a pessoa

pensou assim:

Bem, se eu tivesse olhos azuis, as pessoas

veriam

cada uma duas pessoas com olho azul. Logo, elas no teriam se suicidado no segundo dia, pois no conseguiriam deduzir a cor de seus olhos. Ufa!
Do mesmo modo, a pessoa

Logo, tenho olhos verdes.

conseguiu descobrir a cor de seus olhos.

Analisando de modo semelhante, conseguiremos deduzir que no problema original as cinco pessoas de olhos azuis descobriro que possuem olhos azuis e juntas se suicidaro no quinto dia aps o pronunciamento do estrangeiro. Agora vamos descobrir a resposta da segunda pergunta do enunciado: que informao nova o estrangeiro trouxe? Aparentemente

nada de novo foi acrescentado pela frase do estrangeiro, pois cada pessoa estava vendo alguma pessoa com olhos azuis. Mas isso no verdade. Para ver isso e descobrir qual a nova informao que o estrangeiro trouxe, vamos voltar ao caso de somente duas pessoas na ilha, uma

1.5

Solues dos Problemas da Seo 1.4

23

de olhos azuis e outra de olhos verdes. Neste caso, a pessoa de olhos azuis somente v uma pessoa de olhos verdes. Com a informao de que existe uma pessoa de olhos azuis ela pode descobrir a cor de seus olhos. Note que a pessoa de olhos verdes j sabia que existia pelo Mas ela no sabia que a pessoa

menos uma pessoa de olhos azuis.

de olhos azuis tinha conhecimento de que na ilha existia algum com olhos azuis. Essa a nova informao que o estrangeiro trouxe.

Soluo do Problema 1.10.

Uma primeira soluo cortar a cor-

rente 30 vezes, separando todos os elos. Porm, essa no a melhor soluo, como veremos a seguir. Vamos iniciar nossa anlise observando que para pagar o primeiro dia precisamos dar um corte na corrente. Assim, o gerente receber um elo. O pulo do gato do problema vem agora: para pagar o 2

dia, vamos cortar a corrente de modo a separar

dois elos de uma vez. Assim, daremos dois elos ao gerente e ele devolver um elo de troco. Com este elo pagaremos o terceiro dia. Note que pagamos trs dias fazendo dois cortes na corrente, como mostra a tabela: Gerente Elos 1, 2 Viajante 28 Para

Note que o nmero 2 denota o pedao que contm 2 elos. pagar o 4

dia, cortaremos a corrente de modo a obter um pedao Entregamos ao gerente este pedao e recebemos Com o elo solto,

com quatro elos.

de troco um elo solto e um pedao com dois elos. pagamos o 5 elos: Gerente Elos 1, 4 Viajante 2, 24

dia. Assim, no 5

dia teremos os seguintes grupos de

24

Primeiros Passos

Assim, pagamos o 6

dia com o pedao que contm dois elos e

receberemos o elo solto de troco. Finalmente pagaremos o 7

dia com

o elo solto. Note que foi possvel pagar 7 dias com apenas trs cortes na corrente. A continuao do procedimento est quase revelada. Para pagar o 8

dia, cortaremos um pedao com oito elos.

Daremos este

pedao e receberemos de troco 7 elos, sendo um elo solto, um pedao com 4 e um pedao com dois elos. Repetindo o procedimento anterior, pagaremos os 7 dias seguintes, pagando at o 15

dia sem precisar de

cortes adicionais. Ou seja, para pagar os 15 primeiros dias, precisamos de 4 cortes na corrente. Neste momento, a corrente est distribuda do seguinte modo:

Gerente Elos 1, 2, 4, 8

Viajante 16

Para pagar o 16

dia, entregaremos ao gerente o pedao com os 16

elos restantes, recebendo 15 elos divididos em pedaos de 1, 2, 4 e 8 elos. Se repetirmos o processo, pagaremos o hotel at o 31

dia sem

precisar de novos cortes. Assim, o mnimo nmero de cortes 4.

Soluo do Problema 1.11.

Para resolver este problema vamos usar

a estratgia de mudar a representao . O que signica isso? Vamos reescrever o problema com outros ingredientes, porm sem alterar em nada sua essncia. Primeiramente, enumere as casas do tabuleiro com os nmeros

1, 2, . . . , 9,

como na Figura 1.4.

Vamos agora associar ao tabuleiro, um conjunto de nove pontos tambm enumerados com os nmeros 1, 2, . . . , 9. Se for possvel sair de uma casa

e chegar casa

com apenas uma jogada do cavalo,

colocaremos um segmento ligando os pontos

j.

Por exemplo,

1.5

Solues dos Problemas da Seo 1.4

25

1 4 7

2 5 8

3 6 9

Figura 1.4: Tabuleiro de 9 casas

possvel, saindo da casa 1 chegar casa 6 e a casa 8.

Desse modo, Se

o ponto com nmero 1 est ligado com o ponto com nmero 8.

analisarmos todas as possveis ligaes entre os pontos obteremos um esquema com o mostrado na Figura 1.5

Figura 1.5: Conexes das casas

Figura 1.6: Tabuleiro (a)

Assim, se colocarmos os cavalos como no tabuleiro (a), teremos a situao descrita na Figura 1.6. Deste modo, ca evidente que no podemos trocar a posio dos cavalos branco e preto sem que em algum momento eles ocupem a mesma casa.

26

Primeiros Passos

1.6

Exerccios

1. Uma sacola contm meias cujas cores so branca, preta, amarela e azul. Sem olhar para a sacola, qual a quantidade mnima de meias que precisamos retirar da mesma para garantir pelo menos um par de meias da mesma cor? 2. O pai do padre lho nico de meu pai. O que eu sou do padre? 3. Numa mesa h 5 cartas:

Cada carta tem de um lado um nmero natural e do outro lado uma letra. Joo arma: Qualquer carta que tenha uma vogal tem um nmero par do outro lado. Pedro provou que Joo

mente virando somente uma das cartas. Qual das 5 cartas foi a que Pedro virou? 4. A polcia prende 4 homens, um dos quais culpado de um furto. Eles fazem as seguintes declaraes:

Arnaldo: Bernaldo o culpvel. Bernaldo: Cernaldo o culpvel. Dernaldo: eu no sou culpvel. Cernaldo: Bernaldo mente ao dizer que eu sou culpvel.

Se se sabe que s uma destas declaraes a verdadeira, quem culpvel pelo furto?

1.6

Exerccios

27

5. Numa cidade existe uma pessoa

que sempre mente teras,

quintas e sbados e completamente sincera o resto dos dias da semana. Felipe chega um certo dia na cidade e mantm o

seguinte dilogo com a pessoa  Felipe: Que dia hoje? 

X:

X: Sbado. X: Quarta-feira.

 Felipe: Que dia ser amanh? 

Em qual dia da semana foi mantido este dilogo?

6. Divida o relgio de parede abaixo em 6 partes iguais de forma tal que a soma das horas que cam em cada parte seja a mesma.n

7. Joo adora Gabriela, que uma aluna excelente em Matemtica. Joo armou um plano para dar um beijo nela, e descobriu que poder fazer isso apenas dizendo uma frase. Que frase essa?

8. No plano se colocam 187 rodas dentadas do mesmo dimetro, enumeradas de 1 at 187. A roda 1 acoplada com a roda 2, a 2 com a 3,

...,

a 186 com a 187 e esta ltima com a roda 1. Pode

tal sistema girar?

28

Primeiros Passos

9. Um canal, em forma quadrada, de 4 metros de largura rodeia um castelo. A ponte do castelo est fechada e um intruso quer entrar no castelo usando duas pranchas de 3,5 metros de comprimento. Ser que o intruso consegue?

10. Os nmeros

1, 2, 3, . . . , 99 so escritos no quadro-negro e permi-

tido realizar a seguinte operao: apagar dois deles e substitulos pela diferena do maior com o menor. Fazemos esta operao sucessivamente at restar apenas um ltimo nmero no quadro. Pode o ltimo nmero que restou ser o zero?

11. Algum elege dois nmeros, no necessariamente distintos, no conjunto de nmeros naturais

2, . . . , 20.

O valor da soma destes

A) e o valor do produto dos nmeros dado unicamente a Karla ( K).


nmeros dado somente a Adriano (  Pelo telefone soma.  Uma hora mais tarde,

A diz a K: No possvel que descubras minha K lhe diz a A: Ah! K


sabendo disso, j

sei quanto vale tua soma!  Mais tarde

chama outra vez a

e lhe informa: Poxa,

agora eu tambm conheo teu produto! Quais nmeros foram eleitos?

12. possvel cobrir um tabuleiro de xadrez com 31 domins onde removemos as casas dos vrtices superior esquerdo e inferior direito?

13. Num saco encontram-se 64 moedas leves e 64 moedas pesadas.

1.6

Exerccios

29

possvel separar duas moedas de pesos diferentes com 7 pesagens? 14. Quantas vezes precisamos dobrar um papel de 1mm de espessura para que a altura da pilha chegue da Terra Lua? 15. Descubra o erro da prova da armao abaixo:

Armao: Trs igual a dois.


Seja

um nmero diferente de zero. Temos que:

3x 3x = 2x 2x.
Colocando

xx

em evidncia, temos que:

3(x x) = 2(x x).


Cancelando

xx

em ambos os lados, obtemos que

3 = 2.

30

Primeiros Passos

2
Equaes e Inequaes

Na antiguidade, todo conhecimento matemtico era passado de gerao para gerao atravs de receitas. A falta de smbolos e notao adequada complicava substancialmente a vida de quem precisava usar a Matemtica e de quem apreciava sua beleza. Por exemplo, o uso de letras (x,

y, z

etc.) para representar nmeros desconhecidos no tinha Isso s veio ocorrer por volta dos meados do

sido inventado ainda.

sculo XVI, ou seja, a menos de 500 anos atrs. Apesar disso, o conhecimento matemtico das antigas civilizaes era surpreendente. Os egpcios, babilnios, gregos e vrios outros povos tinham o domnio de mtodos e tcnicas que so empregados hoje, como solues de equaes do primeiro e segundo graus, inteiros que so soma de quadrados e vrios outros conhecimentos. Especialmente os gregos, cuja cultura matemtica resistiu aos tempos com a preservao de Os Elementos de Euclides , tinham desenvolvido e catalisado

31

32

Equaes e Inequaes

muitos dos avanos da poca. Entretanto, todos os resultados tinham uma linguagem atravs dos elementos de geometria, mesmo aqueles que s envolviam propriedades sobre os nmeros. Essa diculdade deve-se em parte aos sistemas de numerao que eram utilizados pelos gregos e, posteriormente, pelos romanos, que eram muito pouco prticos para realizar operaes matemticas. Por volta de 1.100, viveu na ndia Bhaskara, um dos mais importantes matemticos de sua poca. Apesar de suas contribuies terem sido muito profundas na Matemtica, incluindo-se a resultados sobre equaes diofantinas, tudo indica que Bhaskara no foi o primeiro a descobrir a frmula, que no Brasil chamamos de frmula de Bhaskara, assim como Pitgoras no deve ter sido o primeiro a descobrir o teorema que leva o seu nome, j que 3.000 a.c. os babilnios tinham

conhecimento de ternas pitagricas de nmeros inteiros bem grandes. Apesar de ter conhecimento de como solucionar uma equao do segundo grau, a frmula que Bhaskara usava no era exatamente igual a que usamos hoje em dia, sendo mais uma receita de como encontrar as razes de uma equao. usavam a seguinte regra: Para encontrar essas razes, os indianos

Multiplique ambos os membros da equao pelo nmero que vale quatro vezes o coeciente do quadrado e some a eles um nmero igual ao quadrado do coeciente original da incgnita. A soluo desejada a raiz quadrada disso.
O uso de letras para representar as quantidades desconhecidas s veio a se tornar mais popular com os rabes, que tambm desenvolveram um outro sistema de numerao, conhecido como indo-arbico . Destaca-se tambm a participao do matemtico francs Franois

2.1

Equaes do Primeiro Grau

33

Viti, que aprimorou esse uso dos smbolos algbricos em sua obra In

artem analyticam isagoge e desenvolveu um outro mtodo para resolver a equao do segundo grau. Na seo seguinte estudaremos com detalhe a equao do primeiro grau, e como podemos utiliz-la para resolver alguns problemas em Matemtica.

2.1

Equaes do Primeiro Grau

Iniciamos nossa discusso resolvendo o seguinte problema:

Exemplo 2.1.

Qual o nmero cujo dobro somado com sua quinta

parte igual a 121? Soluo: Vamos utilizar uma letra qualquer, digamos a letra
designar esse nmero desconhecido. Assim, o dobro de quinta parte mos que:

x, para 2x e sua

x/5.

Logo, usando as informaes do enunciado, obte-

2x +
ou ainda,

x = 121, 5

10x + x = 605,
onde

11x = 605.

Resolvendo, temos que

x = 605/11 = 55.

Se voc j teve contato com o procedimento de resoluo do exemplo acima, notou que o principal ingrediente a equao do primeiro

grau em uma varivel.

Denio 2.2.

Uma equao do primeiro grau na varivel

uma

expresso da forma

ax + b = 0,

34

Equaes e Inequaes

onde

a = 0, b R

um nmero real a ser encontrado.

Por exemplo, as seguintes equaes so do primeiro grau:

(a) (b)

2x 3 = 0. 4x + 1 = 0. 3 x = 0. 2

(b)

Para trabalhar com equaes e resolv-las, vamos pensar no modelo da balana de dois pratos. Quando colocamos dois objetos com o mesmo peso em cada prato da balana, os pratos se equilibram. Quando os pratos esto equilibrados, podemos adicionar ou retirar a mesma quantidade de ambos os pratos, que ainda assim eles permanecero equilibrados. Essa uma das principais propriedades quando estamos trabalhando com uma equao. Em geral, para resolver uma equao, utilizamos as seguintes propriedades da igualdade entre dois nmeros:

Propriedade 1.

Se dois nmeros so iguais, ao adicionarmos a

mesma quantidade a cada um destes nmeros, eles ainda permanecem iguais. Em outras palavras, escrevendo em termos de letras, se e

so dois nmeros iguais, ento

a+c

igual a

b + c,

ou seja,

a=b

= c

a + c = b + c.

Note que podemos tomar

um nmero negativo, o que signica

que estamos subtraindo a mesma quantidade dos dois nmeros. Por exemplo, se

um nmero qualquer que satisfaz

5x 3 = 6,

2.1

Equaes do Primeiro Grau

35

somando-se satisfazer:

3 a ambos os lados da equao acima, obtemos que x deve (5x 3) + 3 = 6 + 3,


ou seja,

5x = 9.

Propriedade 2.

Se dois nmeros so iguais, ao multiplicarmos a

mesma quantidade por cada um destes nmeros, eles ainda permanecem iguais. Em outras palavras, escrevendo em termos de letras, se e

so dois nmeros iguais, ento

ac

igual a

b c,

ou seja,

a=b
Por exemplo, se igualdade por

ac = bc.

5x = 9

podemos multiplicar ambos os lados da

1/5

para obter

x=

5x 9 = , 5 5 5x 3 = 6.

encontrando o nmero que satisfaz a equao

Para nos familiarizarmos um pouco mais com a linguagem das equaes, vamos pensar no seguinte problema:

Exemplo 2.3.
brincadeira:

Para impressionar Pedro, Lucas props a seguinte

- Escolha um nmero qualquer. - J escolhi, disse Pedro. - Multiplique este nmero por 6. A seguir, some 12. Divida o que voc obteve por 3. Subtraia o dobro do nmero que voc escolheu. O que sobrou igual a 4! Pedro realmente cou impressionado com a habilidade de Lucas. Mas no h nada de mgico nisso. Voc consegue explicar o que Lucas fez?

36

Equaes e Inequaes

Soluo: Na verdade, Lucas tinha conhecimento de como operar com


equaes. Vamos ver o que Lucas fez de perto, passo a passo, utilizando a linguagem das equaes. Para isso, vamos chamar a quantidade que Pedro escolheu de

x:

Escolha um nmero:

x. 6x.

Multiplique este nmero por 6: A seguir, some 12:

6x + 12.

6x + 12 = 2x + 4. 3 Subtraia o dobro do nmero que voc escolheu: 2x + 4 2x = 4.


Divida o que voc obteve por 3:

O que sobrou igual a 4!

Observao 2.4.

Devemos ter cuidado na hora de efetuar divises em

ambos os lados de uma equao, para no cometer o erro de dividir os lados de uma igualdade por zero. Por exemplo, podemos dar uma prova (obviamente) falsa de que

1 = 2,

utilizando o seguinte tipo de

argumento: sempre verdade que

x + 2x = 2x + x.
Logo,

x x = 2x 2x
Colocando

( x x)

em evidncia:

1(x x) = 2(x x)
Dividindo por

1 = 2.

Qual o erro?

( x x)

os dois lados da igualdade acima, temos que

2.1

Equaes do Primeiro Grau

37

Para encontrar a soluo da equao seguinte modo:

ax + b = 0,

procedemos do

Somamos

a ambos os lados da equao, obtendo

ax + b + (b) = 0 + (b) ax = b.
Note que como somamos a mesma quantidade aos dois lados da equao, ela no se alterou.

Dividimos os dois lados da equao por altera a igualdade e nos d que o valor

a = 0. Isso tambm no de x :

b b ax = x = . a a a
Assim, a soluo da equao

ax + b = 0 b x= . a

2.1.1 Problemas Resolvidos


Vamos ver agora alguns problemas que podem ser solucionados utilizando as equaes do primeiro grau.

Problema 2.5.

Se

representa um dgito na base 10 e a soma

x11 + 11x + 1x1 = 777,


quem

x?

38

Equaes e Inequaes

Soluo: Para resolver este problema, precisamos nos recordar que se

abc

a escrita de um nmero qualquer na base

10,

ento esse nmero

igual a

102 a + 10b + c.

Assim, temos que

x11 = 100x + 11 11x = 110 + x 1x1 = 101 + 10x


Logo, temos a seguinte equao do primeiro grau:

100x + 11 + 110 + x + 101 + 10x = 777


Logo,

ou

111x + 222 = 777

x=

777 222 = 5. 111

Problema 2.6.

Determine se possvel completar o preenchimento

do tabuleiro abaixo com os nmeros naturais de

9,

sem repetio,

de modo que a soma de qualquer linha seja igual a de qualquer coluna ou diagonal.

1 9

Soluo: Primeiro, observe que a soma de todos os nmeros naturais


de

45. Assim, se denotamos por

o valor comum da soma dos

elementos de uma linha, somando as trs linhas do tabuleiro, temos que:

45 = 1 + 2 + + 9 = 3s,
Onde

deve ser igual a

15.

Assim, chamando de

o elemento da

primeira linha que falta ser preenchido,

2.1

Equaes do Primeiro Grau

39

x
9

temos que que

1 + x + 6 = 15. Logo, x = 8. Assim, observando a coluna contm 8 e 9, temos que sua soma maior que 15. Logo, no

possvel preencher o tabuleiro de modo que todas as linhas e colunas tenham a mesma soma. Os quadrados de nmeros com essas propriedades se chamam qua-

drados mgicos . Tente fazer um quadrado mgico. Voc j deve ter


percebido que no centro do quadrado no podemos colocar o nmero

9.

De fato, vamos descobrir no exemplo abaixo qual o nmero que

deve ser colocado no centro de um quadrado mgico.

Problema 2.7.

Descubra os valores de

de modo que seja possvel

completar o preenchimento do quadrado mgico abaixo:

x
Soluo: Para descobrir

x,

vamos utilizar o fato de que a soma de

qualquer linha, coluna ou diagonal igual a

15,

j obtido no exemplo

anterior. Se somarmos todas as linhas, colunas e diagonais que contm

x,

teremos que a soma ser

linha, uma coluna e duas diagonais que contm

4 15 = 60,

pois existem exatamente uma

x.

Note tambm que

cada elemento do quadrado mgico ser somado exatamente uma vez, exceto

que ser somado quatro vezes. Assim:

1 + 2 + 3 + 4 + + 9 + 3x = 60,
onde temos que

45 + 3x = 60

e consequentemente

x = 5.

40

Equaes e Inequaes

O problema a seguir um fato curioso que desperta nossa ateno para como a nossa intuio s vezes falha.

Problema 2.8.

Imagine que voc possui um o de cobre extrema-

mente longo, mas to longo que voc consegue dar a volta na Terra com ele. Para simplicar a nossa vida e nossas contas, vamos supor que a Terra uma bola redonda (o que no exatamente verdade) sem nenhuma montanha ou depresso e que seu raio de exatamente

6.378.000

metros.

O o com seus milhes de metros est ajustado Terra, cando bem colado ao cho ao longo do equador. Digamos agora que voc

acrescente 1 metro ao o e o molde de modo que ele forme um crculo enorme, cujo raio um pouco maior que o raio da Terra e tenha o mesmo centro. Voc acha que essa folga ser de que tamanho?
Nossa intuio nos leva a acreditar que como aumentamos to pouco o o, a folga que ele vai ter ser tambm muito pequena, digamos alguns poucos milmetros. Mas veremos que isso est completamente errado!

Soluo. Utilizaremos para isso a frmula que diz que o comprimento

de um crculo de raio

C = 2r,
onde

(l-se pi ) um nmero irracional que vale aproximadamente 3, 1416 (veja a observao a seguir). De fato, o comprimento da Terra CT calculado com essa frmula
aproximadamente:

CT = 2rT = 2 3, 1415 6.378.000 = 40.072.974

metros,

2.1

Equaes do Primeiro Grau

41

onde

rT

o raio da Terra.

x o tamanho da folga obtida em metros e rf o raio do o, temos que a folga ser igual a x = rf rT . Logo, basta calcular rf . Por um lado, o comprimento do o igual a CT + 1 = 40.072.975.
Se chamamos de Logo,

40.072.975 = 2rf 6.378.000, 16 rf rT = 0, 16


a

onde

rf = rf

40.072.975 . 2
aproximadamente igual

Fazendo o clculo acima, temos que metros. Assim,

aproximadamente igual a

x =

metros, ou seja, 16 centmetros!

Observao 2.9.

Vale observar que a folga obtida aumentando o o

no depende do raio em considerao. Por exemplo, se repetssemos esse processo envolvendo a Lua em vez da Terra, obteramos que ao aumentar o o em um metro, a folga obtida seria dos mesmos 16 centmetros. Verique isso!

Observao 2.10.

De fato, podemos denir (e calcular!) o nmero

de vrias maneiras prticas. Vamos considerar dois experimentos

(que se voc no conhece

deve fazer):

Experimento 1: Experimento 2:

Pegar um cinto e fazer um crculo com ele. Calcule

o comprimento do cinturo e divida pelo dimetro do crculo obtido. Pegar uma tampa de uma lata e medir o compri-

mento do crculo da tampa e dividir pelo dimetro da tampa. Se voc efetuou os clculos acima com capricho, deve ter notado que o nmero obtido aproximadamente o mesmo. Se nossos crculos fossem perfeitos (eles nunca so: sempre tm algumas imperfeies) obteramos o nmero

Uma aproximao para

= 3, 1415926535897932384626433832795.

42

Equaes e Inequaes

2.2

Sistemas de Equaes do Primeiro Grau

Nesta seo iremos discutir situaes onde queremos descobrir mais de uma quantidade, que se relacionam de modo linear, ou seja, atravs de equaes do primeiro grau. Por exemplo, considere o seguinte problema:

Exemplo 2.11.

Joo possui 14 reais e deseja gastar esse dinheiro em

chocolates e sanduches para distribuir com seus 6 amigos, de modo que cada um que exatamente com um chocolate ou um sanduche. Sabendo que cada chocolate custa 2 reais e cada sanduche custa 3 reais, quantos chocolates e sanduches Joo deve comprar?
Para resolver esse problema, vamos chamar de chocolates que Joo deve comprar e

a quantidade de

o nmero de sanduches. Assim,

como Joo deseja gastar 14 reais, temos que

2x + 3y = 14.

(2.1)

Como Joo comprar exatamente 6 guloseimas, uma para cada amigo, temos que

x + y = 6.

(2.2)

Note que no encontramos uma equao do primeiro grau em uma varivel e sim duas equaes do primeiro grau em duas variveis. Esse um caso particular de um sistema de equaes do primeiro grau em vrias variveis.

Denio 2.12.
. . . , xn

Uma equao do primeiro grau nas variveis

x1 , x2 ,

uma expresso da forma

a1 x1 + a2 x2 + + an xn + b = 0,

2.2

Sistemas de Equaes do Primeiro Grau

43

onde os nmeros real.


Por exemplo,

a1 , a2 , . . . , an

so diferentes de zero e

um nmero

2x 3y = 0
uma equao do primeiro grau nas variveis

y.

Assim como,

2a b +
Dizemos que os nmeros

c =5 3 a, b
e

uma equao do primeiro grau nas variveis

c.

(r1 , r2 , . . . , rn ) formam uma soluo da equao, se substituindo x1 por r1 , x2 por r2 , . . . , xn por rn , temos que a equao acima satisfeita, isto , a1 r1 + a2 r2 + + an rn + b = 0. Por exemplo, (3, 2) uma soluo da equao 2x 3y = 0 acima,
pois

2 3 3 2 = 0. (2, 3) no soluo da equao 2x 3y = 0, j que 2 2 3 3 = 5 = 0. c Do mesmo modo, (2, 0, 3) soluo da equao 2a b + = 5, pois 3 3 2 2 0 + = 5. 3 Denio 2.13. Um sistema de equaes do primeiro grau em n variveis x1 , x2 , . . ., xn um conjunto de k equaes do primeiro grau em algumas das variveis x1 , x2 , . . . , xn , isto , tem-se o seguinte
Note que a ordem que apresentamos os nmeros importa, pois

conjunto de equaes

a11 x1 + a12 x2 + + a1n xn + b1 = 0, a x + a x + + a x + b = 0, 21 1 22 2 2n n 2 ak1 x1 + ak2 x2 + + akn xn + bk = 0,

(2.3)

44

Equaes e Inequaes

onde alguns dos elementos

Porm, em cada uma das equaes do sistema algum coeciente diferente de zero e, alm disso, cada varivel equao com coeciente distinto de zero.
Dizemos que os nmeros sistema de equaes (2.3) equaes simultaneamente.

aij (1 i k, 1 j n) xj

podem ser zero.

aij

aparece em alguma

(r1 , r2 , . . . , rn ) formam uma soluo do se (r1 , r2 , . . . , rn ) soluo para todas as

Quando resolvemos um sistema de equaes do primeiro grau, podem acontecer trs situaes:

(a) o sistema tem uma nica soluo;

(b) o sistema tem uma innidade de solues;

(c) o sistema no possui soluo.

A seguir ilustramos com exemplos cada uma das situaes acima.

Situao (a):

Retomamos o sistema proposto no Exemplo 2.11, o

qual se encaixa neste caso.

2x + 3y = 14, x + y = 6.
Isolamos o valor de uma das variveis numa das equaes. Por convenincia nos clculos isolamos o valor de

x na segunda equao, obtendo:

x = 6 y.
A seguir, substitumos esse valor na outra equao, obtendo uma equa-

2.2

Sistemas de Equaes do Primeiro Grau

45

o do primeiro grau. Resolvendo temos:

2(6 y ) + 3y = 14, 12 2y + 3y = 14, y = 2. x = 6 2 = 4.

Assim,

y = 2.

Imediatamente, encontramos o valor de

Vamos agora resolver alguns problemas semelhantes.

Situao (b):
x, y
e

Consideremos os sistema de primeiro grau nas variveis

dado por

x + y z 1 = 0 , x y 1 = 0.

(2.4)

Da segunda equao segue-se que

x = y + 1.
Substituindo esta expresso na primeira equao obtemos

(2.5)

(y + 1) + y z 1 = 0,

2y z = 0,

z = 2y.

(2.6)

Notemos que as variveis

xez

so resolvidas em funo da varivel

y,

a qual no possui nenhuma restrio, de modo que se valor real valor

assumir um

ento

cam automaticamente determinadas por este

t.

Isto , para todo

real, de (2.5) e (2.6) tem-se que

x = t + 1,

y = t,

z = 2t

soluo do sistema (2.4) e, portanto, temos innitas solues para este.

46

Equaes e Inequaes

Situao (c):
variveis

Consideremos agora o sistema de primeiro grau nas

x, y

dado por

x + y + 2z 1 = 0, x + z 2 = 0, y + z 3 = 0 .
Neste caso, da segunda e da terceira equao segue-se que

(2.7)

x=2z

y = 3 z.

Substituindo estas expresses na primeira equao obtm-se

(2 z ) + (3 z ) + 2z 1 = 0 4 = 0,
o que uma incompatibilidade. Logo, este sistema no tem soluo.

Observao 2.14. Os sistemas de equaes de primeiro grau so tambm conhecidos como sistemas de equaes lineares. Quando um sistema de equaes lineares envolve muitas variveis no to fcil resolv-lo se no se organiza com cuidado seu processo de resoluo. Existe uma teoria bem conhecida e amplamente divulgada sobre mtodos de resoluo para esse tipo de sistemas. Um dos mtodos mais usado e eciente para resolver sistemas lineares o mtodo de eliminao gaussiana. O leitor interessado pode consultar [7].

2.2.1 Problemas Resolvidos


O problema a seguir foi proposto na primeira fase da Olimpada Brasileira de Matemtica.

2.2

Sistemas de Equaes do Primeiro Grau

47

Problema 2.15.

Passarinhos brincam em volta de uma velha rvore.

Se dois passarinhos pousam em cada galho, um passarinho ca voando. Se todos os passarinhos pousam, com trs em cada galho, um galho ca vazio. Quantos so os passarinhos? Soluo: Vamos chamar de

o nmero de passarinhos e

o nmero

de galhos da rvore. Temos que se dois passarinhos pousam em cada galho, um passarinho ca voando, ou seja,

2g = p 1.
Alm disso, se todos os passarinhos pousam, com trs em um mesmo galho, um galho ca vazio:

3(g 1) = p.
Substituindo na equao anterior, temos que segue-se que

g=4

p = 9. A1

2g = 3g 3 1,
e

onde

Problema 2.16.

Quanto medem as reas

A2

na gura abaixo,

sabendo que o quadrado tem lado 1 e as curvas so arcos de crculos com centros nos vrtices

V1

V2

do quadrado, respectivamente.

V2 A2 A1

V
Soluo: Aplicando relaes de reas na gura temos que

A1 + A2 = , 4 A1 + 2A2 = 1,

48

Equaes e Inequaes

ou seja, chegamos a um sistema de equaes do primeiro grau com duas incgnitas

A1

A2 .

Da primeira equao temos que

A1 =

A2 ; 4

substituindo esta na segunda equao obtemos

A2 + 2A2 = 1, 4
de onde

Logo,

A2 = 1

+ A2 = 1. 4 A1 = 1 = 4 4

1.

Problema 2.17.

Carlos e Cludio so dois irmos temperamentais

que trabalham carregando e descarregando caminhes de cimento. Para Carlos e Cludio tanto faz carregar ou descarregar o caminho, o trabalho realizado por eles o mesmo. Quando esto de bem, trabalham juntos e conseguem carregar um caminho em 15 minutos. Cludio mais forte e trabalha mais rpido conseguindo carregar sozinho um caminho em 20 minutos. (a) Um dia, Cludio adoeceu e Carlos teve que carregar os caminhes sozinho. Quanto tempo ele leva para carregar cada um? (b) Quando os dois brigam, Carlos costuma se vingar descarregando o caminho, enquanto Cludio o carrega com sacos de cimento. Quanto tempo Cludio levaria para carregar o caminho com Carlos descarregando? Soluo: Vamos chamar de
rega por minuto e

a quantidade de sacos que Cludio car-

a quantidade de sacos que Carlos carrega por

2.3

Equao do Segundo Grau

49

minuto. Como Cludio carrega mais que Carlos, sabemos que

y < x.

Do enunciado, sabemos que os dois juntos carregam um caminho em 15 minutos. Se um caminho tem capacidade para

c sacos, temos que:

15x + 15y = c.
Alm disso, sabemos que Cludio sozinho carrega o mesmo caminho em

20

minutos. Logo,

20x = c.
Assim, igualando as duas equaes, temos que

15x + 15y = 20x,

onde

15y = 20x 15x = 5x. 5, temos que 3y = x.


Assim, Clu-

Logo, dividindo ambos os lados por

dio carrega trs vezes mais sacos que Carlos e a resposta do primeiro item

Para descobrir quanto tempo os dois levam para carregar o caminho quando esto brigados, observamos que a cada minuto eles carregam

20 3

minutos, j que

60y = 20 3y = 20x = c.

minutos, j que

xy

sacos, ou seja,

3y y = 2y 30 2y = 60y = c.

sacos. Logo, precisam de

30

2.3

Equao do Segundo Grau

Como j mencionamos em nossa introduo, o conhecimento de mtodos para solucionar as equaes do segundo grau remonta s civilizaes da antiguidade, como os babilnios e egpcios. Apesar disso, a frmula que conhecemos por frmula de Bhaskara , em homenagem ao matemtico indiano de mesmo nome e que determina as solues de uma equao do segundo grau, s veio a aparecer do modo que usamos muito mais tarde, com o francs Viti. Nesta seo iremos

deduzir esta frmula e aplic-la a alguns problemas interessantes.

50

Equaes e Inequaes

2.3.1 Completando Quadrados


Um modo de resolver uma equao do segundo grau o mtodo de

completar quadrados. Ele consiste em escrever a equao numa forma


equivalente que nos permita concluir quais so as solues diretamente. Vamos ilustrar isso com um exemplo, resolvendo a equao

x2 6x 8 = 0.
Podemos escrever essa equao como:

x2 6x = 8.
Somando

ao lado esquerdo, obtemos

(x 3)

. Assim, somando 9 a ambos os lados da equao, obtemos:

x2 6x + 9

que o mesmo que

Logo,

x3=

17

x 3 = 17. Logo, as solues x1 = 3 + 17 e x2 = 3 17.


ou

(x 3)2 = 9 + 8 = 17.

so:

Denio 2.18.

A equao do segundo grau com coecientes

a, b

uma expresso da forma:

ax2 + bx + c = 0,
onde

(2.8)

a = 0, b, c R

uma varivel real a ser determinada.

Para encontrar as solues desta equao, vamos proceder do seguinte modo: isolando o termo que no contm a varivel direito da igualdade na equao (2.8)

do lado

ax2 + bx = c

2.3

Equao do Segundo Grau

51

e dividindo os dois lados por

a,

obtemos:

b c x2 + x = . a a
Agora vamos acrescentar um nmero em ambos os lados da equao acima, de modo que o lado esquerdo da igualdade seja um quadrado perfeito. Para isso, observe que necessrio adicionar dois lados da igualdade. Assim, temos que:

b2 aos 4a2

b x+ 2a

b =x +2 x+ 2a
2

b 2a

c b2 4ac b2 = . 4a2 a 4a2

Em geral, chamamos a expresso (2.8) e denotamos pela letra

b2 4ac de discriminante da equao maiscula (l-se delta ) do alfabeto

grego. Assim, podemos escrever a igualdade anterior como:

b x+ 2a

b2 4ac = 2. 2 4a 4a

(2.9)

Por isso, para que exista algum nmero real satisfazendo a igualdade acima, devemos ter que

igualdade maior ou igual a zero. Extraindo a raiz quadrada quando

0,

j que o termo da esquerda na

0,

temos as solues:

b x+ = 2a

b2 4ac 2a

b b2 4ac x+ = . 2a 2a

Assim, obtemos as seguintes solues:

b x1 = + 2a
e

b2 4ac b + = 2a 2a

52

Equaes e Inequaes

b x2 = 2a
Em resumo,

b2 4ac b = . 4a2 2a

Se Se Se

>0 =0 <0

existem duas solues reais. s existe uma soluo real ( x1 no existe soluo real.

= x2 = b/2a).

A seguir apresentamos alguns exemplos.

Exemplo 2.19.

Encontre as solues da equao

2x2 4x + 2 = 0.

Soluo: Observe que

a = 2, b = 4

c = 2.

Logo,

= b2 4ac = (4)2 4 2 2 = 0.
Assim, a nica soluo

Exemplo 2.20.
grau:

x=

b 4 = = 1. 2a 4

Encontre as razes da seguinte equao do segundo

x2 x 1 = 0 .
Soluo: Basta aplicarmos diretamente a frmula que acabamos de
deduzir. Como

a = 1, b = 1

c = 1,

calculando

temos:

= b2 4ac = (1)2 4 1 (1) = 5.


Logo, as solues so

b + 1+ 5 = x1 = 2a 2

b 1 5 x2 = = . 2a 2

2.3

Equao do Segundo Grau

53

Exemplo 2.21.

Sabendo que

um nmero real que satisfaz

x=1+
determine os valores possveis de

1 1+ 1 x

x.

Soluo: A soluo desse problema consiste numa simples manipulao algbrica, que feita com cuidado nos levar a uma equao do segundo grau. Com efeito,

1+
Logo,

1 x+1 = . x x x 1 + 2x = . 1+x 1+x

1+

1 1+

Ento devemos ter

1 x 1 + 2x , x= 1+x

=1+

de onde segue-se que

x2 + x = 1 + 2x x2 x 1 = 0.
Resolvendo a equao tem-se

1+ 5 x1 = 2

1 5 x2 = . 2
5)/2 chamado de razo urea.

Observao 2.22.

O nmero

(1 +

Este nmero recebe essa denominao pois, frequentemente, as propores mais belas e que a natureza nos proporciona esto prximas da razo urea. Por exemplo, no arranjo das ptalas de uma rosa, nas espirais que aparecem no abacaxi, na arquitetura do Parthenon, nos quadros de da Vinci e nos ancestrais de um zango podemos encontrar a razo urea.

54

Equaes e Inequaes

O problema a seguir est relacionado com a seqncia de Fibonacci e com a razo urea. Dizemos que uma seqncia de nmeros

an

satisfaz a relao de Fibonacci se, para todo

n 0,

temos que (2.10)

an+2 = an+1 + an .

Exemplo 2.23.
para algum

Encontre todas as sequncias

an

da forma

an = x n

x=0

que satisfazem a relao de Fibonacci.

Soluo: Sabendo que


forma

an

satisfaz a relao de Fibonacci e que

an

da

, podemos concluir que para todo

n0

tem-se

xn+2 xn+1 xn = 0.
Colocando

xn

em evidncia na equao acima, temos que

xn (x2 x 1) = 0
Logo, temos duas possibilidades:

x = 0,

temos que

xn = 0

e,

soluo do Exemplo 2.20 temos que as nicas sequncias so

xn = 0 ou x2 x 1 = 0. Como 2 portanto, x x 1 = 0. Observando a


n
ou

an =

1+ 5 2

an =

1 5 2

Observao 2.24. Se an e bn satisfazem a relao de Fibonacci (2.10),


ento dados nmeros reais

qualquer sequncia da forma

an +

bn
com

satisfaz a relao. Pode-se provar que as sequncias dessa forma,

an = x n 1

bn = x n 2

calculados anteriormente, so as nicas

sequncias que satisfazem a relao. Veja, por exemplo, [4].

2.3

Equao do Segundo Grau

55

2.3.2 Relao entre Coecientes e Razes


ax2 + bx + c = 0, com a = 0, j calculamos explicitamente as suas razes x1 e x2 . Vamos estabelecer agora as relaes entre a, b e c e as razes x1 e x2 . Vamos supor 0. Como j sabemos,
Dada a equao

temos que

Assim,

b + b x1 = e x2 = . 2a 2a somando x1 com x2 tem-se 2b b b + b + = = . x1 + x2 = 2a 2a 2a a

(2.11)

Por outro lado, fazendo o produto

x1 x2

obtemos

b + b x1 x2 = 2a 2a b+ b b2 = = 4a2 4a2 4ac c = 2 = . 4a a


Em particular, quando

(2.12)

a = 1,

temos o seguinte resultado.

Teorema 2.25.

Os nmeros

so as razes da equao

x2 sx + p = 0
se, e somente se,

(2.13)

+ =s

= p.

(2.14)

56

Equaes e Inequaes

Demonstrao. Com efeito, se

so as razes de (2.13) ento os

clculos feitos em (2.11) e (2.12) nos do (2.14). Reciprocamente, se vale (2.14) ento da igualdade

(x )(x ) = x2 sx + p
segue-se que

so as razes de (2.13).

Observao 2.26.

igualdade

ax2 + bx + c = 0, com a = 0, podemos escrev-la como a(x2 sx + p) = 0, com s = b/a e p = c/a. Supondo que a equao x2 sx + p = 0 tem razes e , a
Em geral, dada a equao

ax2 + bx + c = a(x2 sx + p) = a(x )(x )


nos permite concluir que grau

(2.15)

so as razes da equao de segundo

ax + bx + c = 0.
raiz de um polinmio do se-

A equao (2.15) mostra que se

gundo grau, ento a diviso desse polinmio pelo polinmio

uma diviso exata. Voltaremos a tratar desse assunto no Teorema 8.5.

(x )

Exemplo 2.27.

Paulo cercou uma regio retangular de rea 28

m2

com 24 metros de corda. Encontre as dimenses dessa regio. Soluo: Se chamamos de

os lados do retngulo construdo por

Paulo, as condies sobre o permetro e a rea desse retngulo nos levam s seguintes equaes:

a + b = 12, ab = 28.
Como j observamos,

so razes da equao

Calculando o discriminante, obtemos

x2 12x + 28 = 0. = 122 4 28 = 32. Utilizando

2.3

Equao do Segundo Grau

57

a frmula, temos que as solues so

12 + 32 a= =6+2 2 2
e

12 32 b= = 6 2 2. 2
Mostre que a equao

Exemplo 2.28.

x2 + bx + 17 = 0

no possui

raiz inteira positiva, se

um inteiro no negativo.

Soluo: Suponhamos que a equao possui alguma raiz inteira


sitiva e seja

m a outra raiz (podendo ser m = n). Ento, onde m = n b dever ser necessariamente um nmero inteiro. Por outro lado, m e n so nmeros inteiros tais que m n = 17, o que s possvel se m = 1 ou n = 1, o que nos daria em qualquer um dos casos que 1 + b + 17 = 0 (b = 18), sendo isto uma contradio com o fato de b ser inteiro no negativo.

n pon + m = b,

Exemplo 2.29.

Numa reunio havia pelo menos 12 pessoas e todos

os presentes apertaram as mos entre si. Descubra quantas pessoas estavam presentes na festa, sabendo que houve menos que 75 apertos de mo. Soluo: Vamos denotar por
as pessoas com os nmeros aperto de mo associaremos apertou a mo da pessoa pessoa

a o nmero de apertos de mo e enumerar do conjunto P = {1, 2, . . . , n}. A cada um par (i, j ), signicando que a pessoa i
Assim, os apertos de mo envolvendo a

j.

foram

A1 = (1, 2), (1, 3), . . . , (1, n) .

58

Equaes e Inequaes

Do mesmo modo, denimos os apertos de mo envolvendo a pessoa que no envolvem a pessoa

1,

como

A2 = (2, 3), (2, 4), . . . , (2, n) .


Note que o aperto

(2, 1)

o mesmo que o aperto

(1, 2),

j que se

aperta a mo de 2, ento

aperta a mo de 1. Analogamente, para

Ai = (i, i + 1), (i, i + 2), . . . , (i, n) ,


Note que

1 i n 1. A1 An1

apertos aparecem em um dos conjuntos contm todos os apertos de mo. elementos do conjunto

Ai Aj =

para

i = j.

Observe tambm que todos os

Ai .

Assim,

Logo, se

o nmero de apertos de mo, temos

|X |

denota o nmero de

|(A1 A2 An1 )| = |A1 | + |A2 | + + |An1 | =


Portanto,

= (n 1) + (n 2) + + 2 + 1 (n 1)n = a. 2

ou igual a 12. Deste modo, basta descobrirmos para que valores de

n2 n 2a = 0 deve admitir admite uma raiz inteira, maior

a < 75 a equao acima admite alguma raiz inteira n 12. Denotemos as razes da equao por n1 e n2 e suponhamos que n1 12. Das
relaes

n1 n2 = 2a, n1 + n2 = 1, n2 = n1 1 11. n2 n1 11 12 = 132, pois n2 11


conclumos que ento Assim, podemos deduzir que e

Observe que o mesmo raciocnio nos leva a concluir que se

n1 12.

n2 n1 = 2a 12 13 = 156,

o que nos daria

n1 13 a 78, sendo

2.3

Equao do Segundo Grau

59

isto impossvel pois

a < 75.

Assim, a raiz positiva para tal equao

no pode ser maior ou igual que 13, restando somente soluo. De fato, essa soluo possvel, se

n1 = 12 como considerarmos a = 66.

Logo, haviam 12 pessoas na festa.

2.3.3 Equaes Biquadradas


A deduo da soluo da equao do segundo grau nos permite resolver equaes de grau mais alto, desde que elas se apresentem numa forma peculiar, que nos permita reduzi-las a uma equao do segundo grau. Por exemplo,

Exemplo 2.30.

Resolva a equao

x4 2x2 + 1 = 0.

(2.16)

Apesar da equao acima ser de grau quatro, podemos solucion-la utilizando o que aprendemos at agora. O truque ser denotar por o valor

Soluo: Denote por

y = x2 .
Assim,

Neste caso, temos que

(y 1)2 . Logo, y = 1. ou x = 1.

x2 = y = 1,

de onde

0 = y 2 2y + 1 = segue-se que x = 1

De modo geral consideremos a equao

ax2k + bxk + c = 0,
e faamos a mudana seguinte;

k N,

(2.17)

y = xk .

Ento, a equao se transforma na

ay 2 + by + c = 0,

(2.18)

60

Equaes e Inequaes

a qual j sabemos resolver. Logo, se (2.18) no possui soluo ento (2.17) tambm no ter soluo e no caso em que

y =

seja uma

raiz de (2.18) ento as solues para (2.17), correspondentes raiz podem ser encontradas resolvendo a equao simples

xk = ,
a qual tem as seguintes possibilidades:

uma nica soluo:

x=

se

mpar;

nenhuma soluo: se duas solues:

<0
se

par; e

x=k

>0

par.

2.3.4 O Mtodo de Viti


A maneira que Franois Viti (1540-1603) descobriu para resolver a equao do segundo grau baseia-se em relacionar a equao

ax2 + bx + c = 0
como uma equao do tipo

(2.19)

Ay 2 + B = 0,
onde

(2.20)

A e B so nmeros que dependem de a, b, c, de modo que qualquer

soluo da equao (2.20) determinar uma soluo da equao (2.19). Note que a ltima equao possui solues

y1 =

B A

y2 =

B , A

se

B 0. A

2.3

Equao do Segundo Grau

61

Para fazer isso, usamos o seguinte truque: escrevendo a soma de duas novas variveis

x = u+v

como

u e v , a equao (2.19) se escreve como

a(u + v )2 + b(u + v ) + c = 0,
a qual, desenvolvendo o quadrado, equivale a

au2 + 2auv + av 2 + bu + bv + c = 0.
Agrupando convenientemente, podemos escrever a expresso acima como uma equao na varivel

v,

isto ,

av 2 + (2au + b)v + au2 + bu + c = 0.


Assim, podemos obter uma equao do tipo da equao (2.20), escolhendo o valor de lhendo

de modo que o termo

(2au + b)v

se anule. Esco-

u = b/2a
2

temos que

av + a

b 2a

b b2 b2 + c = 0 av 2 + + c = 0, 2a 4a 2a b2 + 4ac = 0. 4a

o que equivalente a

av 2 +

Observando que a equao assumiu a forma da equao (2.20), temos que suas solues so

v1 =

b2 4ac 4a2

v2 =
e que

b2 4ac , 4a2 x=u+v

se

= b2 4ac 0.

Lembrando que

equao (2.19) so

u = b/2a x1 =

temos que as solues da

b + v1 2a

x2 =

b + v2 , 2a

como j obtivemos anteriormente.

62

Equaes e Inequaes

2.4

Inequaes

Inequaes aparecem de maneira natural em vrias situaes dentro do contexto matemtico, assim como no prprio dia a dia.

Exemplo 2.31.

Numa loja de esportes as bolas de tnis Welson en-

traram em promoo, passando a custar cada uma trs reais. Pedro que um assduo jogador de tnis quer aproveitar ao mximo a oferta da loja, mas ele s dispe de cem reais. Qual a maior quantidade possvel de bolas que Pedro pode comprar?

Soluo. Se denotamos por

o nmero de bolas que Pedro compra,

ento devemos achar o maior valor possvel de

tal que

3x 100.

(2.21)

Notemos que o problema se reduz a encontrar o maior mltiplo positivo de 3 que seja menor ou igual a 100. Observe que o maior mltiplo de 3 menor ou igual a 100, pois

e Pedro no teria oramento para efetuar a compra. Logo, a soluo

99 = 3 33 3 34 = 102 > 100

x = 33,

ou seja, Pedro poder comprar 33 bolas.

Observemos que no exemplo anterior o que zemos foi achar o maior valor inteiro de nmero

real menor

caso particular de resoluo de uma inequao, chamada inequao do primeiro grau.

x tal que 3x 100 < 0; porm note que qualquer que 100/3 satisfaz que 3x 100 < 0. Isto um

2.5

Inequao do Primeiro Grau

63

2.5

Inequao do Primeiro Grau

Uma inequao do primeiro grau uma relao de uma das formas abaixo

ax + b < 0, ax + b > 0, ax + b 0, ax + b 0,
onde

(2.22)

O conjunto soluo de uma inequao do primeiro grau o conjunto

a, b R S

a = 0.

de nmeros reais que satisfazem a inequao, isto , o conjunto

de nmeros que quando substitudos na inequao tornam a desigualdade verdadeira. Para achar tal conjunto ser de vital importncia

tomar em conta as seguintes propriedades das desigualdades entre dois nmeros

Invarincia do sinal por adio de nmeros reais:


a
e

sejam para

nmeros reais tais que

qualquer nmero real tipo:

c.

O mesmo vale com as desigualdades do

a b,

ento

a+c b+c

<,

ou

>.

Invarincia do sinal por multiplicao de nmeros reais positivos: sejam a e b nmeros reais tais que a b, ento
logos valem para as desigualdades do tipo:

ac bc

para qualquer nmero real positivo

c. Resultados <, ou >.

an-

Mudana do sinal por multiplicao de nmeros reais negativos: sejam a e b nmeros reais tais que a b, ento ac
bc
para qualquer nmero real negativo valem para as desigualdades do tipo:

c. Resultados <, ou >.

anlogos

64

Equaes e Inequaes

Vejamos como solucionar as inequaes estritas

ax + b < 0

ax + b > 0.

Para isto, dividimos a anlise em dois casos.

Caso 1:

a>0
neste caso, dividindo por

Inequao ax + b < 0:
que

obtemos desta

x + b/a < 0

e somando

ltima inequao, temos S

b/a, em ambos os membros que x < b/a. Portanto,

= {x R; x < b/a},

o qual representamos no seguinte desenho:

b/a

Inequao ax + b > 0:
S

procedendo do mesmo modo que o

caso anterior, obtemos que o conjunto soluo vem dado por

= {x R; x > b/a},

representado no desenho abaixo:

b/a

Caso 2:

a<0
neste caso, quando dividimos por

Inequao ax + b < 0:
logo temos que

sinal da inequao se inverte, obtendo assim que

ao x + b/a > 0,

x > b/a
S

e, consequentemente,

= {x R; x > b/a},

cuja representao na reta a seguinte:

2.5

Inequao do Primeiro Grau

65

b/a

Inequao ax + b > 0:
dado por S

similarmente, o conjunto soluo vem

= {x R; x < b/a},

cuja representao a seguinte:

b/a

Observao 2.32.
ax + b 0 b/a.
e

Notemos que se queremos resolver as inequaes ento o conjunto soluo S em cada um

dos casos acima continua o mesmo acrescentado apenas do ponto

ax + b 0,

x=

Vejamos agora um exemplo simples.

Exemplo 2.33.

Para resolver a inequao

dividimos por 8 a inequao (prevalecendo o sinal da desigualdade) e imediatamente adicionamos para obter

8x 4 0,

primeiramente

1/2 em ambos x 4/8 + 1/2 1/2, ou seja,


S

os membros da mesma,

= {x R; x 1/2}.

A seguir damos alguns exemplos que podem ser resolvidos usando inequaes lineares.

Exemplo 2.34.

Sem fazer os clculos, diga qual dos nmeros

a = 3456784 3456786 + 3456785 e b = 34567852 3456788


maior?

66

Equaes e Inequaes

x ao nmero 3456784 ento das denies 2 de a e b temos que a = x (x + 2) + (x + 1) e b = (x + 1) (x + 4). 2 2 Logo, a = x + 3x + 1 e b = x + x 3. Se supomos que a b, ento
Soluo. Se chamamos de

x2 + 3x + 1 x2 + x 3,
e somando

x2 x+3 a ambos os membros desta desigualdade obtemos 2x + 4 0.

A soluo desta inequao do primeiro grau o conjunto dos tais que

nossa suposio inicial de

x 2,

mas isto falso, desde que

x = 3456784. Logo, a ser menor ou igual a b falsa, sendo ento

xR

a > b.
O prximo exemplo j foi tratado antes (ver Problema 2.7), porm apresentamos a seguir uma soluo diferente usando inequaes do primeiro grau.

Exemplo 2.35.

Um quadrado mgico

dividido em 9 quadradinhos de lado 1 de forma tal que os nmeros de 1 at 9 so colocados um a um em cada quadradinho com a propriedade de que a soma dos elementos de qualquer linha, coluna ou diagonal sempre a mesma. Provar que no quadradinho do centro de tal quadrado mgico dever aparecer, obrigatoriamente, o nmero 5. Soluo. Primeiramente observamos que a soma

33

um quadrado de lado 3

45,

logo como h trs linhas e em cada uma destas guram nmeros

1+2+3++9 =

diferentes temos que a soma dos elementos de cada linha 15. Logo, a soma dos elementos de cada coluna ou diagonal tambm 15. Chamemos de

o nmero que aparece no centro do quadrado

mgico, como mostra o desenho a seguir.

2.5

Inequao do Primeiro Grau

67

Agora fazemos as seguintes observaes:

O nmero

no pode ser 9, pois nesse caso em alguma linha,

coluna ou diagonal que contm o quadrado central aparecer o nmero 8, que somado com 9 d acontecer.

17 > 15

e isto no pode

O nmero

x no pode ser 1, y,
ento

pois nesse caso formaria uma linha,

coluna ou diagonal com o nmero 2 e um outro nmero que chamamos de impossvel.

1 + 2 + y = 15 y = 12,

o qual

Feitas as observaes anteriores, temos ento que o nmero

x forma

uma linha, coluna ou diagonal com o nmero 9 e algum outro nmero que chamamos de

z,

logo

z = 15 (x + 9) 1 6 x 1,
de onde segue que

Exemplo 2.36.
de comprimento

x aparece numa linha, coluna ou diagonal com o nmero 1 e algum outro nmero que chamamos de s, logo s = 15 (x + 1) = 14 x 9, de onde temos que x 5. Finalmente, como 5 x 5 segue-se que x = 5.
Por outro lado, o nmero

x 5.

Num tringulo com lados de comprimento

a, b

traamos perpendiculares desde um ponto arbitrrio

P,

sobre o lado

c,

at cada um dos lados restantes (ver a Figura 2.1).

Se estas perpendiculares medem

a > b,

ento

68

Equaes e Inequaes

(a) Qual a posio onde deve ser colocado seja mnimo? (b) Qual a posio onde deve ser colocado seja mximo?

de maneira que

= x+y

de maneira que

= x+y

C a x P c y b A
P
so perpen-

Figura 2.1: No desenho, os segmentos que partem do ponto

diculares aos lados

AC

BC

Soluo. Denotemos por

a rea do tringulo e notemos que divi-

dindo este em dois tringulos menores: um com base outro com base

e altura

e altura

y , temos que ax by + = S, 2 2 ax = 2S by 2S by x= . a

de onde se segue que

Somando

em ambos os lados da ltima igualdade, obtemos

x+y =

2S by +y a 2S by + ay = a 2S a b = + y, a a

2.6

Inequao do Segundo Grau

69

logo

= + y, ab 2S e = . a a Agora notemos que 0 y hb , onde hb denota a altura relativa ao lado de comprimento b no tringulo dado. Como positivo, por ser a > b, temos ento que 0 y hb e, portanto, + y + hb , de onde = 0 + hb .
Resumindo, o valor mnimo de atingido quando onde

y = 0,

portanto

P deve y = hb ,

ser colocado no vrtice portanto

A,

e o valor mximo obtido quando

deve ser colocado no vrtice

B.

2.6

Inequao do Segundo Grau

Agora passamos a discutir a soluo das inequaes do segundo grau, que possuem um maior grau de diculdade quando comparadas com as inequaes do primeiro grau. Ser de vital importncia o uso das propriedades da funo quadrtica anterior. Uma inequao do segundo grau uma relao de uma das formas abaixo

ax2 + bx + c, estudadas no captulo

ax2 + bx + c < 0, ax2 + bx + c > 0, ax2 + bx + c 0, ax2 + bx + c 0,

(2.23)

a, b, c R e a = 0. Por simplicidade, chamaremos o nmero a de 2 coeciente lder da funo quadrtica ax + bx + c.


onde

70

Equaes e Inequaes

Por exemplo, para resolver a inequao o trinmio usando que as razes da isto ,

x2 3x + 2 > 0 fatoramos 2 equao x 3x + 2 = 0 so 1 e 2,

x2 3x + 2 = (x 1)(x 2).
O trinmio toma valores positivos quando o produto positivo, ou seja, quando os fatores sinal:

(x 1)

(x 2)

tenham o mesmo

(x 1)(x 2) for

Ambos positivos:

x1>0x>1
e

x 2 > 0 x > 2,
logo

x > 2.

Ambos negativos:

x1<0x<1
e

x 2 < 0 x < 2,
logo

x < 1. x2 3x + 2 > 0
se, e somente se,

Portanto,

x<1

ou

x > 2.

A seguir explicamos como podemos resolver a inequao do segundo grau de forma geral. Suponhamos primeiramente que queremos resolver a inequao

ax2 + bx + c > 0.

(2.24)

2.6

Inequao do Segundo Grau

71

Notemos que valem as seguintes igualdades:

b ax2 + bx + c = a x2 + x + a b = a x2 + x + a b = a x2 + x + a b =a x+ 2a
onde

c a b2 b2 c + 2 2 4a 4a a 2 b c b2 a 2 2 4a 4a a , 4a

(2.25)

casos:

= b2 4ac.

Considerando esta igualdade, dividimos em vrios

em conta o sinal de

Caso 1: = b2 4ac > 0.


a.

Nesta situao procedemos tomando

(a > 0).

Usando (2.25) notamos que basta resolver a inequao

a x+

b 2a

> 0. 4a
em ambos os membros da

Como

a > 0,

multiplicando por

1/a

desigualdade anterior o sinal desta no muda, obtendo-se ento

x+ >0

b 2a

> 0. 4a2

Agora usamos que

para obtermos que

72

Equaes e Inequaes

b x+ 2a

2 = 4a = =

b x+ 2a

2a

b+ x+ 2a b x 2a

b x+ 2a b + x 2a

onde

b e 2a

= (x )(x ) > 0, = (x )(x ) > 0

b+ so as razes de 2a

ax2 + bx + c = 0. (x )
e

Agora notamos que

se os fatores

(x )

so ambos positivos ou ambos negativos. No primeiro

caso (ambos positivos) temos que

x >

x > ,

mas como

< , ento x > . No segundo caso (ambos negativos), temos que x < e x < , logo x < , novamente por ser < .
Resumindo, a soluo da inequao vem dada pelo conjunto S

= {x R; x <

ou

x > },

com a seguinte representao na reta:

(a < 0).

Esta situao bem similar anterior, a nica dife-

rena que ao multiplicar por que resolver a inequao

1/a o sinal se inverte tendo ento


2

b x+ 2a

< 0, 4a2

2.6

Inequao do Segundo Grau

73

a qual equivalente a provar (seguindo os mesmos passos do caso anterior) que

com

b e 2a

(x )(x ) < 0, =

b+ razes de 2a

ax2 + bx + c = 0.

Notemos que a desigualdade acima vlida sempre que os sinais

(x ) e (x ) forem diferentes. Por exemplo, se x > 0 e x < 0 temos ento que x deve satisfazer a desigualdade < x < , mas isso impossvel considerando que neste caso > , por ser a < 0. No caso restante, se x < 0 e x > 0 temos ento que < x < , o que possvel.
dos fatores Portanto, o conjunto soluo, neste caso, dado por S

= {x R ; < x < },

cuja representao na reta :

resolver a inequao

Caso 2: = b2 4ac = 0.
a x+

Usando novamente (2.25), devemos

b 2a

> 0,
se

a qual vlida para qualquer

a < 0.
os valores de

x = 2ba ,

a > 0

e sempre falsa, se

Caso 3: = b2 4ac < 0.


x
2

Neste caso, quando

positivo todos

reais so soluo para (2.24), pois a desigualdade

b ax + bx + c = a x + 2a

> 0, 4a

74

Equaes e Inequaes

sempre satisfeita, dado que

no temos nenhuma soluo possvel para a inequao (2.24) j que

4 > 0. a

Por outro lado, se

a negativo

ax2 + bx + c = a x +
sempre negativo, dado que

b 2a

4a

4 < 0. a

Observao 2.37.

Para a desigualdade do tipo

ax2 + bx + c < 0
so obtidos resultados similares, seguindo o mesmo processo descrito anteriormente. Alm disso, para as inequaes

ax2 + bx + c 0
ou

ax2 + bx + c 0 ,

os resultados so os mesmos, acrescentados apenas dos pontos

b/2a,

dependendo do caso. Provar que a soma de um nmero positivo com seu

Exemplo 2.38.
Soluo. Seja

inverso sempre maior ou igual que 2.

x > 0,

ento devemos provar que

x+

1 2. x

Partimos da seguinte desigualdade, que sabemos vale para qualquer

x R:
logo

(x 1)2 0 x2 2x + 1 0 x2 + 1 2x.

2.6

Inequao do Segundo Grau

75

Se

positivo, podemos dividir ambos os membros da ltima desi-

gualdade sem alterar o sinal da mesma, ou seja,

x+
conforme queramos provar.

1 2, x

2.6.1 Mximos e Mnimos das Funes Quadrticas


A funo quadrtica

f (x) = ax2 + bx + c,

como j foi observado ante-

riormente, satisfaz a identidade

b ax + bx + c = a x + 2a
2
onde

, 4a f (x)

(2.26)

f ( x)

o menor (maior) valor possvel que pode assumir

= b2 4ac. x

O valor mnimo (mximo ) da funo quadrtica quando

fazemos

percorrer o conjunto dos reais.

Da igualdade (2.26) segue-se que, quando do trinmio obtido quando Similarmente, quando quando

a > 0

o valor mnimo

x =

a < 0 a, b

o valor mximo do trinmio obtido

2ba e este vale

f ( 2ba ) = 4 . a

x=

Exemplo 2.39.
que

2ba , valendo tambm


Sejam

f ( 2ba ) = 4 a

reais positivos tais que

a + b = 1.

Provar

ab 1/4.

ab = a(1 a) = a2 + a. Denindo f (a) = a2 + a, basta provar que f (a) 1/4 para qualquer 0 < a < 1. Completando o quadrado a funo f (a), obtemos
Soluo. Notemos que

f (a) = (a2 a) = (a2 a + 1/4 1/4) = (a 1/2)2 + 1/4,


logo este assume seu valor mximo igual a

1/4,

quando

a = 1/2.

76

Equaes e Inequaes

Alguns problemas de mximos ou mnimos no parecem que possam ser resolvidos achando o mximo ou mnimo de funes quadrticas. Porm, estes problemas podem ser reformulados de forma tal que isto seja possvel. Vejamos um exemplo.

Exemplo 2.40.

Na gura abaixo

ABCD

um retngulo inscrito den-

tro do crculo de raio

r.

Encontre as dimenses que nos do a maior

rea possvel do retngulo

ABCD. D C r y x B

Soluo. A rea do retngulo vem dada pela frmula

A = 2x 2y = 4xy.
Usando o teorema de Pitgoras, temos que

y=

r 2 x2 ,

(2.27)

logo, substituindo esta ltima igualdade na frmula de rea anterior, obtemos

A = 4x r2 x2 . ABCD,
so as mesmas

No muito difcil nos convencermos de que as dimenses, que nos do a maior rea possvel para o retngulo

que nos do o mximo para o quadrado desta rea, ou seja, basta encontrar as dimenses que maximizam

A2 .

A vantagem que tem esta

2.7

Miscelnea

77

reformulao do problema que dada por

A2

tem uma expresso mais simples,

A2 = 16x2 (r2 x2 ) = 16r2 x2 16x4 .


Agora fazemos a mudana

z = x2 ,

para obter

A2 = 16z 2 + 16r2 z = 16 z
de onde segue que o menor valor de portanto quando

r2 2

+ 4r 4 , z =
r2 e 2

A2

obtido quando

x=

r . Usando agora a igualdade (2.27) temos que 2

y= = r 2.

r2

r2 r = . 2 2

Ento, o retngulo de maior rea possvel o quadrado de lado

2r 2

2.7

Miscelnea

Nesta seo combinamos a teoria desenvolvida nos tpicos anteriores para resolver outros tipos de equaes com um nvel de complexidade maior.

2.7.1 Equaes Modulares


Uma equao modular aquela na qual a varivel incgnita aparece sob o sinal de mdulo. Por exemplo, so equaes modulares (a) (b)

|2x 5| = 3; |2x 3| = 1 3x;

78

Equaes e Inequaes

(c)

|3 x| |x + 1| = 4.

Para resolver equaes modulares se usam basicamente trs mtodos: (1) eliminao do mdulo pela denio; (2) elevao ao quadrado de ambos os membros da equao; (3) partio em intervalos. Ilustramos a seguir estes mtodos com os exemplos dados em (a), (b) e (c).

Exemplo 2.41.

Resolver a equao

|2x 5| = 3.

Soluo: O mtodo (1) pode ser utilizado para resolver esta equao.
Para isto, usamos a denio de mdulo:

a |a| = a
de onde segue-se a propriedade: seja

se se

a 0, a < 0.

b um nmero no negativo, ento


ou

|a| = b a = b
Logo,

a = b. x
satisfaz uma das

soluo da equao se, e somente se,

equaes de primeiro grau a seguir:

2x 5 = 3
a soluo

ou

2x 5 = 3. x1 = 4
e da segunda obtemos

Da primeira equao obtemos a soluo

x2 = 1.

2.7

Miscelnea

79

Exemplo 2.42.

Resolver a equao

|2x 3| = 1 3x

Soluo: Resolveremos esta equao pelos mtodos (1) e (2).

Mtodo (1): Aplicando a denio de mdulo temos que resolver a


equao equivalente a resolver os sistemas mistos

(a)

2x 3 0, 2x 3 = 1 3x,

ou

(b)

2x 3 < 0, (2x 3) = 1 3x.

O sistema (a) no tem soluo visto que a soluo da equao do primeiro grau

2x 3 = 1 3x 5x = 4 x = 4/5 a qual no satisfaz a desigualdade 2x 3 0. Por outro lado, no sistema (b) a soluo da equao (2x 3) = 1 3x tem por soluo x = 2 a qual satisfaz a inequao 2x 3 < 0. Logo, a nica soluo da equao x = 2.
Mtodo (2): Observemos que a equao equivalente ao sistema
misto

1 3x 0,

(2x 3)2 = (1 3x)2 .


Resolvendo agora a equao de segundo grau qual equivalente a so

x1 = 4/5 e 1 3x1 0. Assim, a soluo do sistema misto e, portanto, da equao modular apenas x2 = 2.

5x + 6x 8 = 0, temos que as possveis solues x2 = 2, mas x1 descartada pois no satisfaz que

(2x 3)2 = (1 3x)2

Exemplo 2.43. |3 x| |x + 1| = 4.
Soluo. Neste caso usaremos o mtodo de partio em intervalos que
consiste no seguinte: marcamos na reta real os valores onde e

|x + 1|

se anulam, neste caso,

x1 = 3

x2 = 1.

Com isto a reta

|3 x|

80

Equaes e Inequaes

numrica dividida em 3 intervalos

Agora analisamos a equao em cada intervalo:

x < 1, 1 x 3

x > 3.

Intervalo x < 1: Neste caso a equao modular toma a forma

3 x (x 1) = 4 4 = 4,
Portanto, todo o intervalo

Intervalo 1 x 3: Neste caso a equao modular toma a forma

x < 1

soluo.

3 x (x + 1) = 4 2 2x = 4,
de onde segue-se que

x = 1.

x = 1.

Portanto, neste intervalo a soluo

Intervalo x > 3: Neste caso a equao modular toma a forma

3 + x (x + 1) = 4 4 = 4,
o que uma contradio. Portanto, neste intervalo no temos soluo. Em resumo, a soluo da equao modular o intervalo

x 1.

2.7.2 Um Sistema de Equaes No lineares


O seguinte exemplo nos mostra como podemos combinar a tcnica de resoluo de sistemas lineares e de equaes de segundo grau para resolver sistemas mais complicados.

Exemplo 2.44.

Resolva o sistema de equaes:

x2 + 3x (x2 2)3 = 3, x2 + 3x + (x2 2)3 = 1.

2.8

Exerccios

81

Soluo. Propomos a seguinte mudana de variveis:

u=

x2 + 3 x

v = (x2 2)3 .

Assim, o sistema se converte no sistema de equaes do primeiro grau

u v = 3, u + v = 1,
o qual tem como soluo

u=2

x2 + 3x = 2 x2 + 3x = 4,
sendo

v = 1.

Verique! Assim,

x=1

Por outro lado

x = 4

as solues desta equao do segundo grau.

(x2 2)3 = 1, x2 2 = 1, sendo x = 1 e x = 1 as solues desta equao. Logo, a soluo do sistema x = 1, que a nica que satisfaz u = 2 e v = 1 simultaneamente.
de onde

2.8

Exerccios

1. Observe as multiplicaes a seguir: (a) (b) (c)

12.345.679 18 = 222.222.222 12.345.679 27 = 333.333.333 12.345.679 54 = 666.666.666 999.999.999 devemos multiplicar 12.345.679 por quan-

Para obter to?

82

Equaes e Inequaes

2. Outro

dia

ganhei

250

reais, incluindo o pagamento de horas

extras. O salrio (sem horas extras) excede em

200

reais o que

recebi pelas horas extras. Qual o meu salrio sem horas extras? 3. Uma torneira

A enche sozinha um tanque em 10 h, uma torneira

enche o mesmo tanque sozinha em 15 h. Em quantas horas

as duas torneiras juntas enchero o tanque? 4. O dobro de um nmero, mais a sua tera parte, mais a sua quarta parte somam 31. Determine o nmero. 5. Uma certa importncia deve ser dividida entre 10 pessoas em partes iguais. Se a partilha fosse feita somente entre 8 dessas

pessoas, cada uma destas receberia R$5.000,00 a mais. Calcule a importncia. 6. Roberto disse a Valria: Pense um nmero, dobre esse nmero, some 12 ao resultado, divida o novo resultado por 2. Quanto

deu?  Valria disse 15 ao Roberto, que imediatamente revelou o nmero original que Valria havia pensado. Calcule esse nmero. 7. Por

2/3

de um lote de peas iguais, um comerciante pagou

R$8.000,00 a mais do que pagaria pelos o preo do lote todo? 8. Determine um nmero real sejam iguais. 9. Se voc multiplicar um nmero real

2/5 do mesmo lote.

Qual

a para que as expresses

3a+6 2a+10 e 8 6

por ele mesmo e do resul-

tado subtrair 14, voc vai obter o quntuplo do nmero esse nmero?

x.

Qual

2.8

Exerccios

83

10. Eu tenho o dobro da idade que tu tinhas quando eu tinha a tua idade. Quando tu tiveres a minha idade, a soma das nossas idades ser de 45 anos. Quais so as nossas idades? 11. Um homem gastou tudo o que tinha no bolso em trs lojas. Em cada uma gastou 1 real a mais do que a metade do que tinha ao entrar. Quanto o homem tinha ao entrar na primeira loja? 12. Com os algarismos rismos

x, y

formam-se os nmeros de dois alga o nmero de trs algarismos

xy

Quanto

yx, cuja soma valem x, y e z ?


e

zxz .

13. Quantos so os nmeros inteiros de 2 algarismos que so iguais ao dobro do produto de seus algarismos? 14. Obter dois nmeros consecutivos inteiros cuja soma seja igual a 57. 15. Qual o nmero que, adicionado ao triplo do seu quadrado, vale 14? 16. O produto de um nmero positivo pela sua tera parte igual a 12. Qual esse nmero? 17. Determine dois nmeros consecutivos mpares cujo produto seja 195. 18. A diferena entre as idades de dois irmos 3 anos e o produto de suas idades 270. Qual a idade de cada um? 19. Calcule as dimenses de um retngulo de 16 cm de permetro e

15 cm2

de rea.

84

Equaes e Inequaes

20. A diferena de um nmero e o seu inverso nmero?

8 . 3

Qual esse

21. A soma de dois nmeros 12 e a soma de seus quadrados 74. Determine os dois nmeros.

22. Um pai tinha 30 anos quando seu lho nasceu. Se multiplicarmos as idades que possuem hoje, obtm-se um produto que igual a trs vezes o quadrado da idade do lho. idades? Quais so as suas

23. Os elefantes de um zoolgico esto de dieta juntos. Num perodo de 10 dias devem comer uma quantidade de cenouras igual ao quadrado da quantidade que um coelho come em 30 dias. Em um dia os elefantes e o coelho comem juntos 1.444 kg de cenoura. Quantos kilos de cenoura os elefantes comem em 1 dia?

24. Sejam

as razes do polinmio

Calcule as seguintes expresses, em

ax2 + bx + c, com a = 0. funo de a, b e c:

(a) (b) (c)

1 + 2 ; 2 1 + 2 ; 4 + 4 . 1 2 3
a raiz da equao

25. O nmero

condies, determine o valor do

x2 7x 2c = 0. coeciente c.

Nessas

26. Encontre o polinmio a equao

p(x) = 2x4 + bx3 + cx2 + dx + e que satisfaz p(x) = p(1 x).

2.8

Exerccios

85

27. (OBM) Dois meninos jogam o seguinte jogo. O primeiro escolhe dois nmeros inteiros diferentes de zero e o segundo monta uma equao do segundo grau usando como coecientes os dois nmeros escolhidos pelo primeiro jogador e 1.998, na ordem que quiser (ou seja, se o primeiro jogador escolhe gundo jogador pode montar a equao

a e b o se1.998x2 + ax + b = 0 ou

ax2 + 1.998x + b = 0 etc.)

O primeiro jogador considerado ven-

cedor se a equao tiver duas razes racionais diferentes. Mostre que o primeiro jogador pode ganhar sempre. 28. (OBM) Mostre que a equao solues onde

x2 + y 2 + z 2 = 3xyz

tem innitas

x, y, z

so nmeros inteiros.

29. (Gazeta Matemtica, Romnia) Considere a equao

a2 x2 (b2 2ac)x + c2 = 0, a, b e c so nmeros inteiros positivos. Se n N p(n) = 0, mostre que n um quadrado perfeito.
onde 30. (Gazeta Matemtica, Romnia) Sejam equao tal que

a, b Z.

Sabendo que a

(ax b)2 + (bx a)2 = x,


tem uma raiz inteira, encontre os valores de suas razes. 31. (Gazeta Matemtica, Romnia) Resolva a equao:

2x2 = x. x2 + 1
Obs.:

[x]

o menor inteiro maior ou igual a

x.

86

Equaes e Inequaes

32. Demonstrar que: (a)

n4 + 4 n > 1.

no primo se

n > 1; n4 + 4n
no primo, para todo

(b) generalize, mostrando que

33. Para fazer 12 bolinhos, preciso exatamente de 100 g de acar, 50 g de manteiga, meio litro de leite e 400 g de farinha. Qual a maior quantidade desses bolinhos que serei capaz de fazer com 500 g de acar, 300 g de manteiga, 4 litros de leite e 5 kg de farinha ? 34. Dadas as fraes

966666555557 966666555558
qual maior?

966666555558 , 966666555559

35. Achar o maior valor inteiro positivo de

tal que

n200 < 5300 .


36. Achar o menor valor inteiro positivo de
1 2 3 n

tal que

10 11 10 11 10 11 10 11 > 100000.
37. Nove cpias de certas notas custam menos de R$ 10,00 e dez cpias das mesmas notas (com o mesmo preo) custam mais de R$ 11,00. Quanto custa uma cpia das notas? 38. Se enumeram de 1 at

n as pginas de um livro.

Ao somar estes

nmeros, por engano um deles somado duas vezes, obtendo-se o resultado incorreto: 1.986. Qual o nmero da pgina que foi somado duas vezes?

2.8

Exerccios

87

39. Determine os valores de

para os quais a funo quadrtica

ax ax + 12

sempre positiva.

40. Ache os valores de presses positiva:

para os quais cada uma das seguintes ex-

(a)

x 2 x +9

(b)

x3 x+1

(c)

x2 1 x2 3x

41. Resolver a equao:

[x]{x} + x = 2{x} + 10,


onde

[x] denota a parte inteira de x. Por exemplo, [2, 46] = 2 [5, 83] = 5. O nmero {x} chamado parte fracionria de x e denido por {x} = x [x].
maior rea um quadrado.

42. Mostre que entre os retngulos com um mesmo permetro, o de

43. Entre todos os tringulos issceles com permetro

xado, ache

as dimenses dos lados daquele que possui a maior rea. 44. (OBM Jnior 1993) dada uma equao do segundo grau razes inteiras

x2 + ax + b = 0,

com

a1 e b1 . Consideramos a equao do segundo grau x + a1 x + b1 = 0. Se a equao x2 + a1 x + b1 = 0 tem razes 2 inteiras a2 e b2 , consideramos a equao x + a2 x + b2 = 0. Se a 2 equao x + a2 x + b2 = 0 tem razes inteiras a3 e b3 , consideramos 2 a equao x + a3 x + b3 = 0. E assim por diante. Se encontramos uma equao com < 0 ou com razes que no sejam inteiros,
2
encerramos o processo.

88

Equaes e Inequaes

Por exemplo, se comeamos com a equao

x2 = 0

podemos

continuar o processo indenidamente. Pede-se:

(a) Determine uma outra equao que, como

x2 = 0,

nos per-

mita continuar o processo indenidamente; (b) Determine todas as equaes do segundo grau completas a partir das quais possamos continuar o processo indenidamente.

Referncias Bibliogrcas
[1] AIGNER, M. e ZIEGLER, G. (2002). As Provas esto

no Livro. Edgard Blcher.


[2] GARCIA, A. e LEQUAIN, I. (2003). Elementos de l-

gebra. Projeto Euclides, IMPA.


[3] LIMA, E. L.; CARVALHO, P. C. P.; WAGNER, E. e MORGADO, A.C. (2004). A Matemtica do Ensino M-

dio. Volume 1. Sociedade Brasileira de Matemtica.


[4] LIMA, E.L.; CARVALHO, P. C. P.; WAGNER, E. e MORGADO, A.C. (2004). A Matemtica do Ensino M-

dio. Volume 2. Sociedade Brasileira de Matemtica.


[5] LIMA,E.L.; CARVALHO,P. C. P.; WAGNER,E. e

MORGADO,A.C. (2004). A Matemtica do Ensino M-

dio. Volume 3. Sociedade Brasileira de Matemtica.


[6] LIMA, E.L.; CARVALHO, P. C. P.; WAGNER,E. e MORGADO, A.C. (2001). Temas e Problemas. Sociedade Brasileira de Matemtica. [7] LIMA, E.L. (2001). lgebra Linear. Sociedade Brasileira de Matemtica.

285

286

REFERNCIAS BIBLIOGRFICAS

[8] MORAIS FILHO, D. C. (2007). Um Convite Matem-

tica. EDUFCG.
[9] MORGADO, A.; CARVALHO, J.; CARVALHO, P.;

FERNANDEZ, P. (1991). Anlise Combinatria e Pro-

babilidade . Sociedade Brasileira de Matemtica.


[10] RIBENBOIM, P. (2001). Nmeros Primos: Mistrios e

Recordes. Sociedade Brasileira de Matemtica.


[11] SANTOS, J. P. O. (1993) Introduo Teoria dos N-

meros. IMPA.
[12] SANTOS, J. P. O.; MELLO, M. P. e MURARI, I. T. C. (2006). Introduo Anlise Combinatria. Editora Unicamp. [13] SOARES, M. G. (2005). Clculo em uma Varivel Com-

plexa. Sociedade Brasileira de Matemtica.

em Matemtica em Rede Nacional

Mestrado Profissional

Iniciao Matemtica

Autores: Krerley Oliveira Adn J. Corcho

Unidade II: Captulos III e IV

3
Divisibilidade

A teoria dos nmeros o ramo da Matemtica que estuda os mistrios dos nmeros e teve sua origem na antiga Grcia. Os belssimos problemas ligados a esta rea constituem, at hoje, uma das principais fontes inspiradoras dos amantes da Matemtica. Alm disso, essa rea possui vrias aplicaes teis a humanidade, como por exemplo, o processo de criptograa usado em transaes pela Internet. Alguns problemas em teoria dos nmeros demoram sculos para

ltimo teorema de Fermat , que arma que no existe nenhum conjunto de inteiros positivos x, y, z e n com n maior que 2 que satisfaa xn + y n = z n . Esse problema foi obserem resolvidos, como por exemplo o jeto de fervorosas pesquisas durante mais de 300 anos e foi nalmente demonstrado em 1995 pelo matemtico Andrew Wiles. Ainda hoje persistem muitas questes naturais e simples sem resposta. Por exemplo, ningum sabe mostrar (apesar de todo mundo

89

90
acreditar que verdade!) que

Divisibilidade

todo natural par soma de dois primos. Essa a famosa conjectura de Goldbach . Essa simplicidade de se
anunciar problemas e a extrema diculdade em resolv-los faz desta rea um grande atrativo para os matemticos do mundo todo. Este captulo ser dedicado ao estudo de algumas propriedades bsicas relativas aos nmeros inteiros.

3.1

Conceitos Fundamentais e Diviso Euclidiana

Denotamos por

o conjunto dos nmeros inteiros formado pelo con-

junto dos nmeros naturais nmeros negativos. Ou seja,

Comeamos observando que a soma, diferena e produto de nmeros inteiros tambm sero nmeros inteiros. Entretanto, o quociente de dois inteiros pode ser um inteiro ou no. Uma das propriedades fundamentais dos nmeros naturais que utilizaremos ao longo do texto o conhecido que arma o seguinte:

N = {1, 2, 3, . . .} munido do zero e Z = {. . . , 3, 2, 1, 0, 1, 2, 3, . . .}.

dos

princpio da boa ordenao ,

Princpio da Boa Ordenao: todo subconjunto no vazio A N


possui um elemento menor que todos os outros elementos deste, ou seja, existe a A tal que a n para todo n A.
Por exemplo, se

o conjunto dos nmeros pares, o menor elePor outro lado, observamos que o conjunto

mento de

A o nmero 2.

dos nmeros inteiros no goza da boa ordenao. Apesar do princpio da boa ordenao parecer inocente e natural, muitos resultados importantes a respeito dos nmeros naturais decor-

3.1

Conceitos Fundamentais e Diviso Euclidiana

91

rem do mesmo, como veremos ao longo de todo este captulo.

Denio 3.1. Sejam a e b inteiros. Dizemos que a divide b se existe


um inteiro q tal que b = aq . Tambm usaremos as frases a de b ou b mltiplo de a para signicar esta situao.
Usaremos a notao divisor

valentes ditas anteriormente. Se veremos

a | b

para representar todas as frases equi-

no for divisor de

b,

ento escre-

a b.

8 pois considerando o conjunto M = {3m, m N} = {3, 6, 9, 12, . . .} dos

Exemplo 3.2. 7 | 21 pois 21 = 7 3. Por outro lado 3

mltiplos positivos de 3 vemos que 8 no pertence ao mesmo.

A seguinte proposio um bom exerccio para entender os conceitos enunciados acima.

Proposio 3.3. Sejam a, b e c nmeros inteiros. Ento,


(a) se a | b e b | c ento a | c; (b) se a | b e a | c ento a | (b + c) e a | (b c); (c) se a e b so positivos e a | b ento 0 < a b; (d) se a | b e b | a ento a = b ou a = b. Demonstrao.
Se

a|b

b|c

ento existem inteiros

q1

q2

tais que

b = aq1
e

(3.1)

c = bq2 .

(3.2)

92
Substituindo (3.1) em (3.2) temos que

Divisibilidade

c = aq1 q2 = aq,

onde

q = q1 q2 Z, a | b
e

(3.3)

provando isto a armao feita em (a). Agora provaremos (b). igualdades Com efeito, se

a | c

valem as

b = aq1 ,
e

q1 Z q2 Z.

(3.4)

c = aq2 ,

(3.5)

Operando com os ambos lados das igualdades (3.4) e (3.5) temos que

b + c = a(q1 + q2 )
r Z

b c = a(q1 q2 ),
sZ

obtendo assim o resultado desejado. Continuamos agora com a prova de (c). De fato, se ambos positivos, ento

b = aq

com

a | b,

sendo

q 1.
Logo, multiplicando por tivo) que

(3.6)

a ambos lados de (3.6) temos (como a posib = aq a > 0,

como espervamos. Finalmente, provaremos (d). Com este propsito observamos que se

(c) temos

a|b

b|a

|a| = |b|

|a| divide |b| e |b| divide |a|. Portanto, pelo item que |a| |b| e |b| |a|, ou seja, |a| |b| |a|. Logo, consequentemente a = b ou a = b.
ento

3.1

Conceitos Fundamentais e Diviso Euclidiana

93

5.

Exemplo 3.4. Prove que o nmero N = 545362 7 no divisvel por


Vamos mostrar isso utilizando o mtodo do absurdo. Se

Soluo.
5
45362

este nmero fosse divisvel por 5, ento

5q ,

ou seja,

seria divisvel por

5 5, o

45362

que um absurdo.

7 = 5q .

Logo,

7 =

O prximo passo de nossa discusso ver o que acontece quando um nmero no divisvel por outro. Por exemplo, analisemos se 31 divisvel por 7 e para isto listaremos a diferena entre 31 e os mltiplos positivos de 7, isto :

r1 = 31 7 1 = 24, r2 = 31 7 2 = 17, r3 = 31 7 3 = 10, r5 = 31 7 5 = 4,


. . .

r4 = 31 7 4 = 3,

r6 = 31 7 6 = 11,

Claramente 31 no divisvel por 7, pois caso contrrio teramos que alguma das diferenas acima seria igual a zero, o que impossvel pois as diferenas e com

diferenas positivas a nica que menor que 7 corresponde ao caso

q 5

so todas negativas. Entretanto, notamos que entre as

rq = 31 7q

com

1 q 4

so todas positivas

q = 4.

O resultado seguinte nos diz o que acontece no caso geral da

diviso de um inteiro

por um inteiro positivo

a.

94

Divisibilidade

a positivo, existem nicos inteiros q e r tais que b = aq + r, 0 r < a.

Teorema 3.5

(Diviso Euclidiana)

. Dados dois inteiros a e b, sendo

Se a b, ento r satisfaz a desigualdade estrita 0 < r < a. Demonstrao.


a,
basta tomar Por simplicidade, suporemos que

positivo. Se

b = a, ento tomamos q = 1 e Assim, assumiremos tambm que b > a > 0. Consideremos o conjunto
e Se

q=0

r = b.

b< r = 0.

R = {b aq Z; b aq 0} N {0}
Notemos que o conjunto

(3.7) j que

no vazio, pois

b a > 0. Deste modo, pelo princpio da boa ordenao temos que R admite um menor elemento, que denotaremos por r. Claramente r = b aq 0, para algum q 0. Alm disso, r < a pois caso
contrrio

b a R,

r = b aq a b a(q + 1) 0.
Por outro lado,

(3.8)

a > 0 b a(q + 1) < b aq.


Das desigualdades (3.8) e (3.9) segue que

(3.9)

0 b a(q + 1) < b aq,


contradizendo o fato de que de

R.

r = b aq o menor elemento no negativo r


e

Agora provaremos que de fato

q,

escolhidos desta forma, so

nicos. Com efeito, suponhamos que existem outros inteiros tais que

r1

q1

b = aq1 + r1 ,

0 r1 < a.

3.1

Conceitos Fundamentais e Diviso Euclidiana

95

Ento resulta que

aq + r = aq1 + r1 .

Logo,

(r r1 ) = (q1 q )a;
sendo assim,

(3.10)

rr1 mltiplo de a. Mas, em virtude de a < rr1 < a, o nico valor que r r1 pode tomar, sendo este mltiplo de a, r r1 = 0. Portanto, r = r1 , de onde se deduz diretamente de (3.10) que q = q1 .
Os nmeros

no enunciado do teorema acima so chamados, e

respectivamente, de

quociente

resto

da diviso de

por

a.

Um resultado imediato da diviso euclidiana o seguinte.

existe um nmero natural n tal que

Corolrio 3.6. Dados dois nmeros naturais a e b com 1 < a b,


na b < (n + 1)a.

Demonstrao.
0r<a

Pela diviso euclidiana, existem nicos

tais que

b = aq + r.

Assim

q, r N

com

aq b = aq + r < aq + a = a(q + 1).


Basta agora tomar

q=n

para obtermos o resultado.

Os exemplos a seguir apresentam a utilidade do Teorema 3.5.

Exemplo 3.7. Se a um natural com a 3, ento a2 deixa resto 1


na diviso por a 1. Consequentemente, a 1 divide a2 1.
Usando a identidade com

Soluo.

a2 = (a 1)(a + 1) + 1

a2 1 = (a 1)(a + 1) temos que 1 < a 1, de onde segue o resultado.

96

Divisibilidade

O prximo exemplo, como veremos, motiva a procura de caminhos ecientes para encontrar o resto que deixa um nmero quando dividido por outro.

Exemplo 3.8. Um turista brasileiro chega a Cuba e troca parte de


seu dinheiro na casa de cmbio, recebendo 175 notas de 50 pesos e 213 notas de 20 pesos. Ele decide trocar este dinheiro pela maior quantidade possvel das famosas moedas de 3 pesos cubanos, porque elas tm gravada a imagem do guerrilheiro Che Guevara. Quanto sobrou do dinheiro depois de fazer a troca pelas moedas? Soluo.
nmero

Para resolver este problema basta achar o resto que deixa o quando dividido por 3. Entretanto,

queremos destacar que no preciso fazer os produtos e a soma envolvidos no nmero que aparece em assim um novo

n = 175 50 + 213 20 n.

Em lugar de fazer isto substitumos cada nmero

n pelo resto que este deixa na diviso por 3, formando nmero n1 , ou seja, n1 = 1 2 + 0 2 = 2 .

Agora procuramos o resto que

n1

deixa na diviso por 3, que obvi-

amente 2. A surpresa que este resto o mesmo que deixa diviso por 3. Logo, sobraram 2 pesos depois de fazer a troca.

na

A soluo do exemplo anterior uma aplicao particular do seguinte lema que de muita utilidade na resoluo de problemas.

nmeros naturais deixa o mesmo resto que a soma e o produto dos seus restos, na diviso por um inteiro positivo a.

Lema 3.9

(Lema dos Restos)

. A soma e o produto de quaisquer dois

3.1

Conceitos Fundamentais e Diviso Euclidiana


Sejam

97

Demonstrao.

ambos os nmeros por

n1 , n2 Z. a temos que
e

Fazendo a diviso com resto de

n1 = aq1 + r1
com

n2 = aq2 + r2 ,

0 r1 , r2 < a.

Ento,

n1 n2 = (aq1 + r1 )(aq2 + r2 ) = a2 q1 q2 + aq1 r2 + aq2 r1 + r1 r2 = a(aq1 q2 + q1 r2 + q2 r1 ) + r1 r2 = aq + r1 r2 ,


onde (3.11)

q = aq1 q2 + q1 r2 + q2 r1 .

Agora dividimos

r1 r2 por a para obtermos


(3.12)

r1 r2 = ap + r,

p Z,

0 r < a.

Das igualdades (3.11) e (3.12) segue que

n1 n2 = aq + ap + r = a(p + q ) + r,

0 r < a.

(3.13)

Portanto, de (3.12) e (3.13) conclumos que os restos que deixam e

n1 n2

r1 r2

na diviso por

so iguais, cando provado o resultado para o

produto. A prova para a soma anloga.

Observao 3.10. A vantagem do lema que em certos problemas


que envolvem nmeros muito grandes podemos substituir estes por nmeros muito menores e mais confortveis para trabalhar.
seguir.

Vejamos como aplicar o lema dos restos nos seguintes exemplos a

Exemplo 3.11. Prove que o produto de dois nmeros naturais consecutivos sempre divisvel por 2.

98

Divisibilidade
por

Soluo.
2.

Se

Quando fazemos a diviso de

n N temos r=0
ou

que provar que

an = n(n + 1) divisvel

por 2 temos duas possibilidades

para o resto:

r = 1.

Analisemos os dois casos por separado.

[r = 0]
2.

Neste caso o resto que deixa

an

na diviso por 2 o

mesmo que o resto que deixa 0(0+1)=0, logo

an

divisvel por

[r = 1]

Neste caso podemos substituir

an

por 1(1+1)=2 e o

resto que este ltimo deixa quando dividido por 2 0, logo tambm divisvel por 2.

an

Mostraremos agora como utilizar o exemplo anterior pra resolver um dos problemas da

1a

Olimpada Brasileira de Matemtica.

8.

Exemplo 3.12. Prove que se n mpar, ento n2 1 mltiplo de


Como

Soluo.
k Z.

mpar, podemos escrever

n = 2m + 1,

para algum

Logo

n2 1 = (2m + 1)2 1 = 4m2 + 4m + 1 1 = 4m2 + 4m.


Assim,

n2 1 = 4m(m + 1).
Observe que de acordo com o exemplo 3.11, 2. Portanto,

m(m + 1) = 2q

para algum

m(m + 1) q Z, de aonde

mltiplo de

n2 1 = 4m(m + 1) = 4 2q = 8q,
como queramos demonstrar.

3.2

Bases Numricas

99

inteiros, pelo menos um deles mltiplo de 3. Soluo.

Exemplo 3.13. Prove que em qualquer tringulo retngulo com lados


Comecemos analisando quais so os restos possveis para a

diviso por 3 de um nmero que quadrado. De acordo com o lema dos restos temos a seguinte tabela para os restos de por 3:

n e n2 ,

na diviso

n n2
0 1 2 0 1 1

Resumindo, se um nmero no mltiplo de 3 ento o resto da diviso de seu quadrado por 3 deve ser igual a 1. Agora denotemos por

os catetos e por

a hipotenusa. Supo-

nhamos que nenhum deles divisvel por 3. Ento 1 na diviso por 3. Logo,

a2 + b 2

deixa resto

a2 e b2 deixam resto 12 + 12 = 2 na diviso

por 3; mas isto uma contradio pois, pelo Teorema de Pitgoras,

a2 + b 2 = c 2

c2

deixa resto 1 quando dividido por 3.

3.2

Bases Numricas

Comeamos esta seo com uma brincadeira interessante. Joo, ao sair da aula de matemtica do professor Peitgoras, encontrou Pedro e lhe props a seguinte brincadeira:  Pense numa pea de domin, Pedro. Vou adivinhar que pea essa usando uma frmula mgica.  Ok, Joo. Pode comear, j pensei.

100

Divisibilidade

Figura 3.1: Pea de Domin

- Escolha um dos nmeros na pea e multiplique por 5.

Depois

disso some trs a esse resultado. Multiplique agora o nmero que voc obteve por dois. Some isto com o outro nmero da pea. Qual foi o resultado?  Foi 40.  Ento a pea que voc escolheu foi a 3 com 4!  Como voc acertou? Me ensina! Claro que de mgico Joo no tinha nada e decidiu contar seu segredo a Pedro. O jogo funciona assim: cada parte da pea de domin pode ser considerada como um dos dgitos de um nmero de 2 algarismos, o qual denotamos por

n = xy = 10x + y

(veja a Figura 3.1). Acompanhando

os passos de Joo, temos que:

(5x + 3)2 + y = 40 10x + y = 34,


que claramente, tem por solues: tao de 34 na base decimal.

(3.14)

x=3

y = 4,

usando a represen-

No sistema de numerao decimal, tambm conhecido como sistema numrico na base

10,

todo nmero pode ser representado como

uma sequncia de 10 smbolos, constitudos pelo rismos

(zero) e os alga-

1, 2, 3, . . . , 9.

Por exemplo,

345

escreve-se na base decimal da

3.2

Bases Numricas

101

seguinte forma

345 = 300 + 40 + 5 = 3 102 + 4 10 + 5,


assim como

2768

se escreve da forma

2768 = 2000 + 700 + 60 + 8 = 2 103 + 7 102 + 6 10 + 8.


De modo geral, se denotamos por positivo formado pelos ento

a = an an1 . . . a1 a0 o nmero inteiro algarismos an , an1 , . . . , a1 e a0 , nessa ordem,

se escreve na base decimal da forma

a = an 10n + an1 10n1 + . . . + a1 10 + a0

(3.15)

Antes de provar alguns dos critrios de divisibilidade mais populares do sistema de numerao decimal, provamos uma identidade muito til.

Lema 3.14. Sejam a, b, n N. Temos que


an bn = (a b)(an1 + an2 b + + abn2 + bn1 ).

Consequentemente, se 0 < b < a, ento a b divide an bn . Demonstrao.


1.
Primeiro provaremos que a propriedade vale para

b=

Com efeito, considerando a soma geomtrica

s = 1 + a + a2 + + an1
e multiplicando

por

temos que

as = (a + a2 + + an1 ) + an = s 1 + an .

102
Assim,

Divisibilidade

(a 1)s = as s = an 1,

de onde se segue que (3.16)

an 1 = (a 1)(an1 + an2 + + a + 1).


Da temos a validade para Para

b = 1. an b n = b n ( a )n 1 . b
Usando

esta expresso e (3.16) tem-se

bN

qualquer, observe que

an b n = b n ( a 1) ( a )n1 + ( a )n2 + + ( a )+1 b b b b

= (a b)(an1 + an2 b + + abn2 + bn1 ),


obtendo-se a igualdade clamada.

= (a b)bn1 ( a )n1 + ( a )n2 + + ( a )+1 b b b

(3.17)

um inteiro positivo, ento

Proposio 3.15

(Critrios de Divisibilidade)

. Seja a = an . . . a1 a0

(a) a divisvel por 10 se, e somente se, a0 for igual a 0; (b) a divisvel por 3 ou por 9 se, e somente se, a soma dos seus dgitos divisvel por 3 ou por 9, respectivamente; (c) a divisvel por 5 se, e somente se, a0 for igual a 0 ou 5. Demonstrao.
Utilizando a representao decimal (3.15) temos que

a = 10(an 10n1 + an1 10n2 + + a1 ) + a0 .


Ento, pela Proposio 3.3-(b) tem-se que

10 | a0 ,

prondose-se assim o critrio (a).

10 | a

se, e somente se,

Para provar (b) observemos que

a = an 10n + an1 10n1 + + 10a1 + a0

= an (10n 1) + an1 (10n1 1) + + (10 1)a1

(3.18)

+ an + an1 + + a1 + a0 .

3.2

Bases Numricas

103

Pelo Lema 3.14 temos que segue-se

10j 1 = 9qj

para todo

1 j n,

da

a = 9(an qn + an1 qn1 + + a1 ) + an + an1 + + a1 + a0 .


Ento, aplicando novamente o item (b) da Proposio 3.3 temos que

9|a

se, e somente se,

9 | (an + an1 + + a1 + a0 ).

A prova para o caso da divisibilidade por 3 segue de maneira idntica, logo ca provado o item (b). A prova do item (c) segue de maneira muito semelhante e deixamos a mesma a cargo do leitor.

Exemplo 3.16. Prove sem fazer muitas contas que o nmero


N = 13424136 + 1234567890

divisvel por 3. Soluo.


Note que no precisamos fazer a soma dos nmeros ante-

riores. Para mostrar isso, basta aplicar o item (b) da Proposio 3.3 e o item (b) da Proposio 3.15, observando que cada um dos nmeros acima divisvel por

3,

pois a soma de seus dgitos um mltiplo de

3.
Finalizamos esta seo com uma aplicao da diviso euclidiana que nos mostra que, analogamente representao decimal, qualquer nmero admite uma representao nica em qualquer outra base numrica.

104

Divisibilidade

Teorema 3.17

(Bases Numricas)

tem nicos nmeros naturais r0 , r1 , . . . , rn tais que 0 ri b 1, 0 i n, e satisfazendo


a = rn bn + rn1 bn1 + + r1 b + r0 .

. Dados a, b N, com b > 1, exis-

A representao acima dita representao de a na base b e usaremos a notao


a = (rn cn1 . . . r1 r0 )b ,

para fazer referncia a esta. Demonstrao.


segue:

Apliquemos sucessivamente a diviso euclidiana como

a = bq0 + r0 , q0 = bq1 + r1 , q1 = bq2 + r2 ,


. . . . . . . . .

r0 < b, r1 < b, r2 < b,


. . .

qj 1 = bqj + rj ,

rj < b,

a > q0 > q1 > q2 > > qj 1 , para algum j = n deveremos ter que qn1 < b. Logo, qj = 0 para todo j n, assim como rj = 0 para todo j n + 1. Das igualdades acima, para
e assim por diante. Como

3.2

Bases Numricas
tem-se

105

1 j n,

a = bq0 + r0 , bq0 = b2 q1 + br1 , b2 q1 = b3 q2 + b2 r2 ,


. . . . . . . . . (3.19)

bn1 qn2 = bn qn + bn1 rn1 bn qn1 = bn+1 0 + bn rn .


Efetuando a soma de todas as igualdades em (3.19) obtemos

a = rn bn + rn1 bn1 + + r1 b + r0 .
A unicidade dos nmeros euclidiana.

ri

vem da unicidade dos restos na diviso

Observao 3.18. O sistema de numerao na base 2 tambm conhecido como sistema binrio e o sistema habitualmente utilizado no funcionamento dos computadores.

Exemplo 3.19. Se deseja pesar qualquer nmero inteiro de gramas de

ouro, entre 1g e 100g , numa balana de dois pratos, onde os pesos s podem ser usados no prato esquerdo da balana. Mostre que a escolha adequada de 7 pesos diferentes suciente para realizar esta tarefa. Demonstrao.
nica como Usando o sistema de numerao em base

temos que

qualquer nmero

tal que

1 a 100

pode ser expressado de forma

a = r6 26 + r5 25 + r4 24 + r3 23 + r2 22 + r1 2 + r0 1,

106
com

Divisibilidade

estas potncias no so consideradas. notemos tambm que o fato de cada

ri {0, 1}, 0 i 1. ri
ser

Observe que

2n 128,

com

n 7,

logo

ou

nos diz que no precisamos repetir nenhum dos

pesos na realizao de qualquer pesada. Logo, os pesos

1, 22 , 23 , 24 , 25 , 26
so sucientes para realizar as pesadas de gramas de ouro entre

1g

100g .

3.3

Mximo Divisor Comum e Mnimo Mltiplo Comum

Nesta seo estudaremos dois conceitos fundamentais, que aparecem naturalmente em vrios problemas de divisibilidade, assim como a relao existente entre eles.

3.3.1

Mximo Divisor Comum

O primeiro destes conceitos est relacionado com os inteiros positivos que dividem simultaneamente a dois inteiros prexados e denominado

mximo divisor comum .

Daqui por diante s consideraremos os divisores positivos dos nmeros.

Denio 3.20 (Mximo Divisor Comum) . Sejam a e b inteiros diferentes de zero. O mximo divisor comum, resumidamente mdc, entre a e b o nmero d que satisfaz as seguintes condies: (a) d um divisor comum de a e b, isto , d | a e d | b;

3.3

Mximo Divisor Comum e Mnimo Mltiplo Comum

107

(b) d o maior inteiro positivo com a propriedade (a). Neste caso, denotamos o mdc entre a e b por d = mdc(a, b) ou por d = (a, b). Se (a, b) = 1, ento dizemos que a e b so primos entre si.

Exemplo 3.21. Observando que 12 = 6 2, 18 = 6 3 temos que

mdc.(12, 18) = 6. Por outro lado, mdc. (4, 15) = 1, logo os nmeros 4 e 15 so primos entre si.
Vejamos agora algumas das propriedades mais importantes dos divisores comuns de dois inteiros.

Proposio 3.22. Sejam a e b dois inteiros. Ento valem as seguintes


armaes. (a) Se a mltiplo de b, ento (a, b) = b. (b) Se a = bq + c, c = 0, ento o conjunto dos divisores comuns dos nmeros a e b coincide com o conjunto dos divisores comuns dos nmeros b e c. Particularmente, (a, b) = (b, c). Demonstrao.
Comeamos com a prova de (a). Com efeito, todo

a e b um divisor de b. Reciprocamente, usando que a mltiplo de b, todo divisor de b tambm um divisor de a, ou seja, um divisor comum dos nmeros a e b. Portanto, o conjunto dos divisores comuns dos nmeros a e b igual ao conjunto dos divisores de b. Como o maior divisor de b ele mesmo, resulta que (a, b) = b.
divisor comum dos nmeros Vejamos (b). Usando o item (b) da Proposio 3.3 temos que todo divisor comum de um divisor de

tambm divide

e, consequentemente,

b e c. Pela mesma razo todo divisor comum de b e c tambm divide a e, consequentemente, um divisor de a e b. Portanto

108
os divisores comuns de de

Divisibilidade

so os mesmos que os divisores comuns

c.

Particularmente, tambm coincidem os maiores divisores

comuns, ou seja,

(a, b) = (b, c).

O teorema a seguir uma das ferramentas bsicas na resoluo de problemas que envolvem o mdc entre dois nmeros. O resultado foi provado pela primeira vez por Claude-Gaspard Bachet de Mziriac (1581-1638) e mais tarde generalizado para polinmios por tienne Bzout (1730-1783). Frequentemente, na literatura se enuncia este

resultado como teorema (ou identidade) de Bzout, esquecendo-se o nome de Bachet.

ento existem nmeros inteiros x0 e y0 tais que d = (a, b) = ax0 + by0 . Demonstrao.
Considere a combinao linear

Teorema 3.23

(Teorema de Bachet-Bzout)

. Se d o mdc de a e b,
ax + by ,
onde

percorrem todos os inteiros. Este conjunto de inteiros, denotado por

Ca,b = {ax + by ; x, y Z},


inclui valores positivos e negativos. Alm disso, escolhendo vemos que

x = y = 0, y0
tais que

Ca,b

tambm contm o zero.

Pelo princpio da boa ordenao, podemos escolher

x0

= ax0 + by0 seja o menor nmero inteiro positivo contido no conjunto Ca,b .
Agora mostraremos que outro segue analogamente. Usaremos para este propsito o mtodo de reduo ao absurdo, ou seja, vamos supor que contradio.

|a

| b.

Provaremos que

|a

e o

e obteremos uma

3.3

Mximo Divisor Comum e Mnimo Mltiplo Comum


Usando a diviso euclidiana, de

109
existem inteiros

tais que

a = q + r

com

a segue que 0 < r < . Portanto,

r = a q = a q (ax0 + by0 ) = a(1 qx0 ) + b(qy0 )


e assim

est no conjunto

Ca,b ,

o que contradiz a hiptese de

ser o

menor elemento positivo contido em Uma vez que desde que

Ca,b .
efeito,

divide a e b s resta provar que = d. Com d = (a, b), podemos escrever a = da1 , b = db1 e = ax0 + by0 = d(a1 x0 + b1 y0 ).

Assim

d . Agora d < d = = ax0 + by0 .

d | .

Logo pela parte (c) da Proposio 3.3, conclumos que impossvel pois

d =

mdc(a, b), e portanto

A seguinte proposio resume algumas consequncias importantes da demonstrao dada ao teorema de Bzout.

Proposio 3.24. Sejam d, N e a, b, c Z. Ento valem as


seguintes armaes: (a) Se d | a e d | b, ento d | (a, b). (b) O mdc.(a, b) o menor valor positivo de ax + by , onde x e y percorrem todos os nmeros inteiros. (c) (a, b) = (a, b).
1 (d) Se d | a e d | b, ento ( a , b) = d (a, b). Consequentemente, d d

a b , (a, b) (a, b)

= 1.

110

Divisibilidade

(e) Se (a, c) = (b, c) = 1, ento (ab, c) = 1. (f) Se c | ab e (b, c) = 1, ento c | a. Demonstrao.


A prova de (a) consequncia imediata da igualdade anunciada no teorema de Bzout; assim como (b)

(a, b) = ax0 + by0

segue diretamente da demonstrao dada a este teorema. Para provar (c), primeiro observamos que

(a)x + (b)y = (ax + by )


Usando o item (a) e o fato de que

onde

x, y Z.

ser positivo, da igualdade acima segue

(a, b) = min (a)x + (b)y > 0; = min ax + by ; x, y Z = (a, b).

x, y Z

A armao feita em (d) segue diretamente de (c), observando que

(a, b) =

a b d ,d d d

=d

a b , d d

.
temos que

Continuamos com a prova de (e). De existem inteiros

(a, c) = (b, c) = 1,

xj

yj , j = 1, 2,

tais que

ax1 + cy1 = 1, bx2 + cy2 = 1.


Multiplicando lado a lado as igualdades obtemos

(x1 x2 )ab + (ax1 y2 + y1 bx2 + cy1 y2 )c = 1.


x y

3.3

Mximo Divisor Comum e Mnimo Mltiplo Comum

111

Ento, usando o item (b) e a igualdade acima resulta que

(ab, c) = 1.

Finalmente, provaremos (f ). Das hipteses temos que existem inteiros

x0

y0

tais que

bx0 + cy0 = 1.
Multiplicamos a igualdade acima por

a em ambos lados para obtermos

abx0 + acy0 = a.
Por outro lado,

ab = cq

para algum inteiro

q.

Usando esta condio

na ltima igualdade temos que

cqx0 + acy0 = c(qx0 + ay0 ) = a,


logo

c | a.
Algoritmo de Euclides

3.3.2

Apesar de conhecermos propriedades tericas do mdc entre dois inteiros, encontr-lo de fato pode ser uma tarefa complicada, sem auxlio das ferramentas corretas. Lembrando o seu signicado, o leitor talvez pudesse pensar que devemos calcular todos os divisores de os divisores de conjuntos. Para achar o mdc se faz uso de um importante mtodo denominado

a,

todos

e descobrir qual o maior elemento comum aos dois

algoritmo de Euclides .

Teorema 3.25 (Algoritmo de Euclides) . Dados dois inteiros positivos,


a e b, aplicamos sucessivamente a diviso euclidiana para obter a se-

112

Divisibilidade

guinte sequncia de igualdades


b = aq1 + r1 , a = r1 q2 + r2 , r = r q + r , 1 2 3 3 rn2 = rn1 qn + rn , r n1 = rn qn+1 , 0 r1 < a, 0 r2 < r1 , 0 r3 < r2 , 0 rn < rn1 ,
(3.20)

at algum rn dividir rn1 . Assim, o mdc.(a, b) = rn , ou seja, o ltimo resto no-nulo no processo de diviso anterior.

Observao 3.26. Quando lidamos com nmeros pequenos achar o


mdc uma tarefa fcil pois podemos calcular o mdc valendo-nos das fatoraes dos nmeros envolvidos. No entanto, quando estamos trabalhando com nmeros grandes o algoritmo de Euclides, em geral, mais fcil que a fatorao, podendo ser esta ltima bem difcil. Demonstrao do algoritmo de Euclides.
ros inteiros

Comeamos observando que

o processo de diviso (3.20) nito. Com efeito, a sequncia de nme-

rk estritamente decrescente e est contida no conjunto {r Z, 0 r < a}, portanto no pode conter mais do que a inteiros positivos. Examinando as igualdades (3.20) de cima para baixo e usando a Proposio 3.22 temos que

(a, b) = (a, r1 ) = (r1 , r2 ) = = (rn1 , rn ) = rn .

3.3

Mximo Divisor Comum e Mnimo Mltiplo Comum

113

ser obtido como consequncia do processo de diviso (3.20). Com efeito, podemos escrever
rn = rn2 rn1 qn rn1 = rn3 rn2 qn1 rn = rn2 (rn3 rn2 qn1 )qn .

Observao 3.27. Notemos que o teorema de Bzout tambm pode

Logo, conseguimos escrever rn em termos de rn2 e rn3 . Utilizando a expresso rn2 = rn4 rn3 qn2 podemos escrever rn como combinao de rn3 e rn4 . Repetindo este processo vrias vezes, conclumos que existem x, y Z tais que
d = rn = xr1 + yr2 .

Ora, como r1 = b aq1 e r2 = a r1 q2 = a(1 + q1 q2 ) bq2 , ento, substituindo estes valores na ltima igualdade obtemos o Teorema de Bzout.

Exemplo 3.28. Achar o mximo divisor comum dos nmeros 471 e


1.176. Soluo.
Aplicando o algoritmo de Euclides obtemos a seguinte sequn-

cia de divises com resto:

1176 = 471 2 + 234, 471 = 234 2 + 3, 234 = 78 3,

ento o mdc(471, 1176)

= 3. 2n + 8 irredutvel para todo 4n + 15

Exemplo 3.29. Provar que a frao


nmero natural n.

114

Divisibilidade

Soluo.

Usando o algoritmo de Euclides temos que

4n + 15 = (2n + 8) 1 + 2n + 7, 2n + 8 = (2n + 7) 1 + 1, 2n + 7 = (2n + 7) 1.

Ento o mdc. (4n

+ 15, 2n + 8) = 1

e portanto

4n + 15

2n + 8

so

primos entre si para qualquer valor de

n.
60 vezes

Exemplo 3.30. Achar o mdc.(111 . . . 111, 11 . . . 11)


100 vezes

Soluo.

Primeiro escrevemos os nmeros na base decimal, isto ,

111 . . . 111 = 1099 + 1098 + + 1,


100 vezes

11 . . . 11 = 1059 + 1058 + + 1.
60 vezes
Aplicamos agora o algoritmo de Euclides para obter as seguintes igualdades

111 . . . 111 = (1059 + 1058 + + 1)1040 + 1039 + 1038 + + 1,


100 vezes

1059 + 1058 + + 1 = (1039 + 1038 + + 1)1020 + 1039 + 1038 + + 1 = (1019 + 1018 + + 1)1020 + + 1019 + 1018 + + 1.
Disso resulta que mdc.(111 . . . 111, 11 . . . 11)

+ 1019 + 1018 + + 1,

100 vezes

60 vezes

= 1019 + 1018 + + 1 = 11 . . . 11 .
20 vezes

3.3

Mximo Divisor Comum e Mnimo Mltiplo Comum


Mnimo Mltiplo Comum

115

3.3.3

Agora passamos ao segundo conceito importante desta seo. O mesmo est relacionado com os inteiros positivos que so simultaneamente mltiplos de dois inteiros prexados e denominado

mnimo mltiplo

comum.

Denio 3.31

(Mnimo Mltiplo Comum)

diferentes de zero. O mnimo mltiplo comum, resumidamente mmc, entre a e b o inteiro positivo m que satisfaz as seguintes condies: (a) m um mltiplo comum de a e b, isto , a | m e b | m; (b) m o menor inteiro positivo com a propriedade (a).

. Sejam a e b inteiros

Neste caso, denotamos o mmc entre a e b por m = mmc(a, b) ou por m = [a, b].
Resumimos a seguir algumas das propriedades fundamentais do mmc de dois inteiros.

guintes armaes:

Proposio 3.32. Sejam a, b, c Z e Z. Ento valem as se(a) se c mltiplo comum de a e b, ento [a, b] | c; (b) [a, b] = [a, b]; (c) |ab| = [a, b] (a, b).

Demonstrao.
de

Comeamos com a prova de (a). A diviso com resto

por

[a, b]

nos d

c = [a, b]q + r,

0 r < [a, b].

(3.21)

116
Da igualdade anterior, basta provar que desejado. Suponhamos, pelo contrrio, que tanto

Divisibilidade
resultado Notemos

r = 0 para obter o que 0 < r < [a, b]. a


e

como

dividem

[a, b].
e

Logo, pelo item (b) da Pro-

posio 3.3 e a igualdade (3.21), temos que ou seja,

tambm dividem

mltiplo comum de

e no pode ser menor que

r, [a, b],

contradizendo nossa suposio. Prosseguimos com a prova de (b). Observemos que plo comum de

[a, b]

mlti-

b,

logo pelo item (i) vale que

[a, b] [a, b].


Por outro lado, [a, b] = q1 a = q2 b, para alguns inteiros [a,b] logo, um mltiplo comum de a e b. Portanto,

(3.22)

q1

q2 ;

[a, b]

[a, b] [a, b] [a, b].

(3.23)

Das igualdades (3.22) e (3.23) segue que

[a, b] [a, b] [a, b],


de onde vem diretamente o resultado. Para provar (c) podemos supor sem perda de generalidade que

so positivos devido s igualdades

[a, b] = [a, b] = [a, b] = [a, b].


Dividiremos a prova em dois casos:

Caso 1: (a, b) = 1.
b | [a, b] e [a, b] = qa, para algum q N. Ento b | qa e alm disso (a, b) = 1. Logo, pelo item (v) da Proposio 3.24 temos que b | q . Portanto, b q e consequentemente
Sabemos que

ab aq = [a, b].

(3.24)

3.3

Mximo Divisor Comum e Mnimo Mltiplo Comum

117

Entretanto, da denio de

[a, b]

vale que (3.25)

[a, b] ab.
Das desigualdades (3.24) e (3.25) segue que

ab = [a, b] = [a, b] 1 = [a, b] (a, b).

ab [a, b] ab.

Assim,

Caso 2: (a, b) > 1.


Da parte (c) da Proposio 3.24 sabemos que Aplicando o caso anterior vale que

a , b (a,b) (a,b)

= 1.

a a b b = , (a, b) (a, b) (a, b) (a, b)


Multiplicamos esta ltima igualdade por

a b , (a, b) (a, b) (a, b)2

e usamos o item (b)

provado anteriormente, assim como a parte (d) da Proposio 3.24 para obter

ab = (a, b)

a b , (a, b) (a, b) (a, b)

a b , (a, b) (a, b)

= [a, b] (a, b).

Exemplo 3.33. Dois amigos passeiam de bicicleta, na mesma direo, em torno a uma pista circular. Para dar uma volta completa um deles demora 15 minutos e o outro demora 18 minutos. Eles partem juntos e combinam interromper o passeio quando os dois se encontrarem pela primeira vez no ponto de partida. Quantas voltas deu cada um? Soluo.
Denotemos por

n1 T

n2 , respectivamente, o nmero de voltas

que d cada um dos amigos. Notemos que o tempo total da corrida o menor valor positivo de que satisfaz as igualdades

T = 15n1 = 18n2 ,

118
ou seja

Divisibilidade

T = [15, 18] =
Portanto,

15 18 = 90. 3

n1 = 6

n2 = 5.

Finalizamos esta seo com um exemplo que nos fornece uma bela interpretao geomtrica do mnimo mltiplo comum. proposto na Olimpada Brasileira de Matemtica. O mesmo foi

Exemplo 3.34. Um retngulo de lados inteiros AB = m e CD = n,


dividido em quadrados de lado 1. Em cada um dos vrtices ele possui um pequeno orifcio. Um raio de luz entra no retngulo por um dos vrtices, na direo da bissetriz do ngulo reto, e reetido sucessivamente nos lados do retngulo. Quantos quadrados so atravessados pelo raio de luz?
D C

Figura 3.2: Interpretao geomtrica do mmc

Soluo.
n,

Se zermos alguns testes preliminares dando valores a

veremos que em cada caso a resposta coincidir com o

me mmc( m,n).

Provemos que isto de fato vale para

quaisquer. Para realizar a

prova nos auxiliaremos da Figura 3.2.

3.3

Mximo Divisor Comum e Mnimo Mltiplo Comum

119

Primeiramente, notemos que cada vez que o raio de luz atravessa um quadrado ele avana uma unidade tanto na direo horizontal como na direo vertical. Usando este fato fazemos as observaes a seguir.

Se o raio entra pelo vrtice at chegar ao lado

A,

ter que atravessar

quadrados

BC , imediatamente mais m para chegar ao lado AD , depois mais m para chegar novamente ao lado BC , e assim sucessivamente. Alm disso, depois do raio percorrer pm quadrados, com p N, estar batendo no lado BC ou no lado AD.
Analogamente o raio bater no lado somente se, atravessar

AB

ou no lado

DC

se, e

qn

quadrados, com

q N.

Somente nos vrtices

B, C

do retngulo pode acontecer que

o raio incidente saia do retngulo, terminando assim o processo de reexo.

Usando as observaes acima fcil ver que o raio chegar a um vrtice quando chegar simultaneamente a dois lados perpendiculares do retngulo. Portanto, deve ter atravessado um nmero dos tal que de

x de quadrao

x = pm = qn,

ou seja,

dever ser um mltiplo comum

n. claro que a primeira vez que o raio chega a um vrtice nmero x o menor mltiplo comum de m e n, isto , x = [m, n].
e duas vezes no percurso do raio de

Finalmente, observamos que nenhum dos quadrados atravessado

A at bater no primeiro vrtice, pois

como vemos na gura numa das direes os quadrados atravessados sero todos cinzas e na outra direo, sero todos brancos.

120
3.3.4 Equaes Diofantinas Lineares

Divisibilidade

Consideremos a equao

ax + by = c,
onde

(3.26)

A equao (3.26) chamada de

a, b, c Z,

com

a=0

b = 0.

equao diofantina linear


(x, y )

e uma

soluo desta qualquer par de inteiros

que satisfaam (3.26).

conhecido que todos os pontos do plano, com coordenadas

(x, y ),

que satisfazem a igualdade (3.26) representam, geometricamente, uma reta. Logo, as solues de uma equao diofantina linear so os pontos de coordenadas inteiras do plano cartesiano, que esto dispostos sobre a reta que esta representa. Por exemplo, os pontos so solues da equao diofantina
3 2 1 0 -1 -2 -3 -3 -2

3x 2y = 1, y x

veja a Figura 3.3.

(1, 2)

(1, 1)

-1 0 1 2

Figura 3.3: A equao da reta

3x 2y = 1.

Naturalmente nos perguntamos: sempre possvel achar solues para uma equao diofantina linear? A resposta no; o prximo

resultado nos diz quando isto possvel. Alm disso, se uma equao diofantina linear tem uma soluo na verdade ela tem uma innidade de solues.

3.3

Mximo Divisor Comum e Mnimo Mltiplo Comum

121

Proposio 3.35. A equao diofantina linear


ax + by = c, a, b, c Z, com a = 0 e b = 0,
(3.27)

tem soluo se, e somente se, d | c, onde d = (a, b). Alm disso, se (x0 , y0 ) uma soluo, ento o conjunto de solues de (3.27) constitudo por todos os pares de inteiros (x, y ) da forma:
b x = x0 + t d

e y = y0 t a , d

t Z.

(3.28)

Demonstrao.
de (3.27), logo existem inteiros

Primeiramente suponhamos que

(x0 , y0 ) uma soluo ax0 + by0 = c. Usando que d = (a, b) sabemos que q1 e q2 , tais que dq1 = a e dq2 = b. Portanto, se dq1 x0 + dq2 y0 = d(q1 x0 + q2 y0 ) = c,

verica a igualdade

de onde segue obviamente que

Reciprocamente, suponhamos que

d | c.

inteiro. O teorema de Bzout nos garante a existncia de dois inteiros,

d|c

e portanto

c = qd

com

x0

y0 ,

ltima

ax0 + by0 = d. Multiplicando igualdade por q temos que


tais que

ambos os lados desta

ax0 q + by0 q = c,
logo o par diofantina. Resta provar agora que temos innitas solues da forma (3.28). Com efeito, sendo

(x1 , y1 ),

com

x1 = x 0 q

y1 = y0 q ,

soluo da equao

(x, y ) uma outra soluo qualquer alm de (x0 , y0 ), vale que ax0 + by0 = c = ax + by , de onde ax0 + by0 = ax + by . Desta igualdade obtemos a(x x0 ) = b(y0 y ) e dividimos esta ltima por d para obtermos a b (x x0 ) = (y0 y ). d d

122
Como existe

Divisibilidade
Logo,

(a , b ) = 1, ento d d inteiro t tal que

temos que

a d

| (y0 y )

b d

| (x x0 ).

b x = x0 + t d

y = y0 t a . d t as expresses

Por outro lado, fcil vericar que para qualquer inteiro achadas acima para

resolvem a equao diofantina.

A seguir damos um exemplo de como proceder para resolver equaes diofantinas.

Exemplo 3.36. Achar todas as solues inteiras da equao


12x + 33y = 27.

Soluo.

Observemos que

(12, 33) = 3

e que

o tem innitas solues. e teremos as restantes.

Como sabemos, basta achar uma delas

3 | 27,

logo a equa-

Para achar esta soluo particular podemos

trabalhar de duas maneiras, que descrevemos a seguir:

Alternativa 1:

reduzimos a equao forma equivalente

4x + 11y = 9,
e por tentativa e erro vemos que

x0 = 5

mesma. Ento pela Proposio 3.35 temos que

y0 = 1 t Z,

solucionam a

x = 5 + 11t

y = 4t 1,

esgotam todas as solues que procuramos.

Alternativa 2:
mdc

aplicamos o algoritmo de Euclides para achar o

(12, 33),

obtendo os seguintes resultados:

33 = 12 2 + 9, 12 = 9 1 + 3, 9 = 3 3 + 0.

3.4

Nmeros Primos e Compostos

123

Da segunda e primeira igualdades temos, respectivamente, que

3 = 12 9 1
Usando estas duas obtemos

9 = 33 12 2.

3 = 12 (33 12 2) 1 = 12 33 + 12 2 = 3 12 1 33,

x0 = 3 e y0 = 1, garantidos pelo teorema de Bzout, que validam 3 = 12x0 +33y0 . Multiplicamos por 9 esta ltima
ou seja, achamos igualdade para obter

27 = 12(9x0 ) + 33(9y0 ).
Portanto,

x0 = 9x0 = 27

a equao diofantina.

Analogamente, como na alternativa anterior,

y0 = 9y0 = 9 resolvem, particularmente,

podemos escrever a soluo geral da forma:

x = 27 + 11s

y = 4s 9,

s Z.

3.4

Nmeros Primos e Compostos

Ao longo da histria da Matemtica, os nmeros primos foram protagonistas de clebres problemas que motivaram o desenvolvimento de teorias e tcnicas pelas mentes mais frteis, como Fermat, Euler e Gauss. At hoje muitos desses problemas, simples de enunciar, que

envolvem nmeros primos so desaos intelectuais para toda a humanidade.

124

Divisibilidade

Esta seo ser dedicada ao estudo de propriedades bsicas dos nmeros primos. Todo nmero natural

maior do que 1 tem pelo

menos 2 divisores, claramente 1 e

n.

Isto motiva a seguinte denio.

Denio 3.37 (Nmeros Primos e Compostos) . Um inteiro positivo


n 2 dito primo se os nicos divisores que ele tem so 1 e ele

prprio; caso contrrio, dito composto. primo nem composto.

Observao 3.38. De modo geral o nmero 1 no considerado nem Exemplo 3.39. Os nmeros 2, 3, 5, 7, e 11 so primos e os nmeros
10, 15, 35 e 348 so compostos.

Exemplo 3.40. O nmero n = 220 254 composto.


Soluo.
Escrevemos

de outra forma, com o objetivo de facilitar

nosso trabalho. Com efeito, observemos que

n = (210 )2 (252 )2 = 10242 6252 ,


logo composto por ser diferena de quadrados. Alm disso,

n = 10242 6252 , = 399 1649,


Portanto, podemos concluir que

= (1024 625)(1024 + 625),

(3.29)

= 3 133 1649. 3 | n.

Proposio 3.41. Seja n > 1 um nmero inteiro. Ento


(a) o menor divisor de n diferente de 1 um nmero primo;

3.4

Nmeros Primos e Compostos

125

(b) se n composto, o seu menor divisor diferente de 1 no maior que n. Em outras palavras, se n no possui divisores diferentes de 1, menores ou igual que n, ento n primo. Demonstrao.
n, diferente de 1 < q < p; mas
Comeamos provando (a). Seja 1. Se

o menor divisor de

fosse composto teria algum divisor

tal que

q|p
o que nos diz que

p | n, p.
o menor divisor de

Para provar (b) denotamos por de 1. Portanto,

q | n,

e isto contradiz a hiptese levantada sobre

desigualdade

n = pq com por p obtemos

p q p.

n,

diferente

Multiplicando ambos lados da

e consequentemente vale

n p.

n = pq p2 ,

Agora vamos enunciar um dos resultados mais clssicos da Matemtica, que garante a existncia de innitos nmeros primos. At

onde se conhece, a demonstrao a seguir foi a primeira demonstrao escrita utilizando o mtodo de reduo ao absurdo e devida a Euclides cerca de 300 a.C. Para outras seis provas, incluindo a moderna prova de Fustenberg, recomendamos os livros [1] e [10].

Teorema 3.42 (Teorema de Euclides). A quantidade de nmeros primos innita. Demonstrao.


por Faremos a prova por reduo ao absurdo. Suponha

que existe uma quantidade nita de nmeros primos e denotemos estes

p1 , p2 , p3 , . . . , pk .

126
Consideremos o nmero

Divisibilidade

n = p1 p2 p 3 pk + 1
e chamemos de

o seu menor divisor primo. Obviamente

no coin-

cide com nenhum dos nmeros como ele divide

n,

teria que dividir 1, o que impossvel. Logo, te-

pi , 1 i k ,

pois caso contrrio,

mos uma contradio hiptese de termos uma quantidade nita de primos. Os nmeros primos tambm podem ser caracterizados da seguinte maneira:

Proposio 3.43. Um inteiro positivo p primo se, e somente se,


satisfaz a seguinte propriedade:
p | ab = p | a ou p|b
(3.30)

onde a, b Z. Demonstrao.
logo Primeiramente, suponhamos que

p primo e que p b,

(p, b) = 1.

Ento, pelo item (f ) da Proposio 3.24 temos que

p | a.

Reciprocamente, suponhamos que, a propriedade 3.30 vlida e

alm disso vamos supor, pelo absurdo, que

no primo. Ento, (3.31)

p = d1 d2 ,
De (3.30) segue que

com ou

1 < d1 < p, 1 < d2 < p.


consequentemente

p | d1

p | d2 ;
ou

p d1 ,

p d2 ,

(3.32)

contradizendo isto o armado em (3.31).

3.5

Procurando Primos

127

3.5

Procurando Primos

Os nmeros primos alm de belos e desaadores do ponto de vista matemtico, so extremamente importantes para as atividades usuais de nosso dia a dia. Por exemplo, nenhuma transao bancria ou pela internet estaria segura sem o uso de nmeros primos muito grandes. Assim, surge naturalmente a pergunta de como podemos produzi-los em grandes quantidades. Essa pergunta sempre intrigou os matemticos e continua sem soluo at os dias atuais. Apesar deles serem

abundantes, em quantidade innita de acordo com o Teorema 3.42, no existe nenhum mtodo razovel de produo de nmeros primos, mesmo tendo em mos a alta tecnologia de hoje em dia. Porm, ao longo do tempo algumas frmulas e algoritmos se mostraram teis para a descoberta de nmeros primos.

3.5.1

O Crivo de Eratstenes
um algoritmo que nos permite achar todos

O crivo de Eratstenes

os nmeros primos que so menores ou iguais que um natural

dado.

Segundo a tradio, este mtodo foi criado pelo matemtico grego Eratstenes (285-194 a.C.). O mtodo consiste nos seguintes passos: escrevemos os nmeros de forma ordenada a partir de 2, isto ,

2, 3, 4, 5, 6, 7, 8, 9, 10, 11, 12, 13, 14, 15, 16, 17, . . . , n

(3.33)

Observamos que o primeiro primo que aparece em (3.33) 2 e imediatamente apagamos da lista (3.33) todos os mltiplos de 2 maiores que ele, por serem compostos; resta assim a seguinte

128
lista

Divisibilidade

2, 3, 5, 7, 9, 11, 13, 15, 17 . . .


O primeiro nmero no apagado que aparece na lista restante 3, que tambm primo. Imediatamente apagamos da lista todos os mltiplos de 3 maiores que ele, por serem compostos; resta agora a lista

2, 3, 5, 7, 11, 13, 17, . . .


O primeiro nmero no apagado que aparece na lista que restou do passo anterior 5, que tambm primo. Imediatamente

apagamos da lista todos os mltiplos de 5 maiores que ele, por serem compostos.

Repetimos este processo at que o primeiro nmero no apagado da lista em questo seja maior que

n, pois graas Proposio

3.41-(b) a partir desse momento todos os nmeros restantes so os primos menores ou iguais que Por exemplo, se aplicando o mtodo: 2 11 21 31 12 22 32 3 13 23 33 4 14 24 34 5 15 25 35 6 16 26 36 7 17 27 37 8 18 28 38 9 19 29 39 10 20 30 40

n..

n = 40,

temos que

40 = 6, 324555.

Ento,

Passo 1: ordenamos os nmeros

2
11 21 31

3 13 23 33

5 15 25 35

7 17 27 37

9 19 29 39

3.5

Procurando Primos
Passo 2: tiramos os mltiplos de 2

129

2 11

3
13 23

7 17 19 29 37

25 35

31

Passo 3: tiramos os mltiplos de 3

2 11

3 13 23

7 17 19 29 37

31

Passo 4: tiramos os mltiplos de 5

Como

72 = 49 > 40,

paramos agora.

nmero primo p podemos comear a apagar a partir de p2 , pois se supomos que existe um nmero composto m no apagado menor que p2 , temos que m = p1 q1 , sendo p1 seu menor divisor primo. Ento, pelo item (b) da Proposio 3.41, p1 < m < p, logo m deveria ter sido apagado pois mltiplo de um primo menor que p.

Observao 3.44. Note que ao comear a apagar os mltiplos de um

3.5.2

Primos de Mersenne

Marin Mersenne (1588-1648) foi um monge francs que nasceu na cidade de Maine e foi um dos grandes inuenciadores da Matemtica

130
2 31 73 127 179 233 283 353 419 467 3 37 79 131 181 239 293 359 421 479 5 41 83 137 191 241 307 367 431 487 7 43 89 139 193 251 311 373 433 491 11 47 97 149 197 257 313 379 439 499 13 53 101 151 199 263 317 383 443 503 17 59 103 157 211 269 331 389 449 509 19 61 107 163 223 271 337 397 457 521

Divisibilidade
23 67 29 71 113 173 229 281 349 409 463 541

109 167 227 277 347 401 461 523

Tabela 3.1: Os primeiros 100 nmeros primos

francesa nos sculos XVI e XVII. Apaixonado pelos nmeros, teve entre seus correspondentes Descartes, Fermat, Pascal e Galileu. Entre suas vrias descobertas, ele estudou os nmeros da forma:

Mn = 2n 1.
Observe que vale o seguinte fato a respeito desses nmeros:

Proposio 3.45. Se Mn primo, ento n primo.


Demonstrao.
n = a.b,
com Provar essa proposio equivale a mostrar que a sua forma contrarrecproca vale. Ou seja, que se

composto, digamos De fato,

usando o Lema 3.14, podemos decomp-lo do seguinte modo:

a b > 1,

ento

Mn

tambm composto.

Ma.b = 2ab 1 = 2a(b1) 2a(b2) + + 2a + 1 2b 1 .

3.5

Procurando Primos

131

Porm, no verdade a recproca da armao acima. Por exemplo, Hudalricus Regius mostrou em 1536 que no primo, j que

Em 1643, Mersenne armou que para

2.047 = 23 89.

M11 = 211 1 = 2.047

n = 2, 3, 5, 7, 13, 17, 19, 31, 67, 127 e 257,


os valores de menores que

Mn so todos primos 257, Mn composto. n onde Mn primo:

e para todos os outros valores de

Hoje sabemos que Mersenne errou na sua armao, esquecendo trs valores de 61, 89 e 107 e incluindo

M67 e M257

como nmeros primos. Para mais informaes, sugerimos a pgina http://primes.utm.edu/mersenne/index.html.

web

Finalizamos esta seo, com um critrio interessante, devido matemtica francesa Sophie Germain (1776-1831), que nos permite saber quando um nmero no primo.

Proposio 3.46
vale a igualdade

(Identidade de Sophie Germain)

. Dados a, b R,

a4 + 4b4 = (a2 + 2b2 + 2ab)(a2 + 2b2 2ab).

Demonstrao.

A prova segue das seguintes igualdades:

a4 + 4b4 = a4 + 4a2 b2 + 4b4 4a2 b2 = (a2 + 2b2 )2 4a2 b2 = (a2 + 2b2 + 2ab)(a2 + 2b2 2ab).

Como aplicao desta identidade vejamos os seguintes exemplos.

132

Divisibilidade

Exemplo 3.47. qn = n4 + 4n composto, para todo n N.


Soluo.
O conjunto dos nmeros naturais particionado em duas classes disjuntas:o conjunto dos nmeros pares e o conjunto dos nmeros mpares. Estudaremos cada classe por separado. Assim,

n um nmero par, ento n = 2m para algum inteiro positivo m 1. Deste modo,


se

n4 + 4n = (2m)4 + 42m = 16m4 + 24m , = 2 8m4 + 24m1 .


Portanto, neste caso,

nmero inteiro positivo par temos que primo;

n4 + 4n 2.

Logo, se

n>1

qualquer

n +4

no um nmero

se

um nmero mpar, ento

n = 2m + 1

para algum inteiro

positivo

m 1.

Assim,

n4 + 4n = (2m + 1)4 + 42m+1 = (2m + 1)4 + 4 42m


Logo, tomando

= (2m + 1)4 + 4 24m = (2m + 1)4 + 4 (2m )4 . a = 2m + 1


e

b = 2m ,

o resultado uma con-

sequncia direta da identidade de Sophie Germain.

Exemplo 3.48. 520 + 230 um nmero composto.


Soluo.
Escrevemos

520 + 230 = 554 + 22 228 = 55 b=2


7
para comprovar que o nmero

+ 4 27

, a = 55
e

de onde podemos usar a Identidade de Sophie Germain com

5 +2

20

30

composto.

3.5

Procurando Primos
O Teorema Fundamental da Aritmtica

133

3.5.3

Os nmeros primos so as

clulas

dos nmeros naturais, no sentido

de que qualquer nmero natural produto de nmeros primos. Por exemplo,

560 = 56 10 = 7 8 5 2 = 7 2 2 2 5 2,
onde cada um dos fatores que aparecem no produto so nmeros primos. Perguntamo-nos, o que acontece se comeamos com uma outra fatorao inicial de 560, por exemplo,

560 = 28 20.

Vejamos:

560 = 28 20 = 14 2 10 2 = 7 2 2 5 2 2.
Surpreendentemente chegamos mesma representao anterior, salvo a ordem dos fatores.

Figura 3.4: O nmero 560 composto de 4 clulas do tipo 2, uma clula

do tipo 7 e uma clula do tipo 5.

O fato observado acima vale para qualquer nmero natural maior que 1. Especicamente, temos o seguinte resultado conhecido como

teorema fundamental da aritmtica .

134

Divisibilidade

Teorema 3.49 (Teorema Fundamental da Aritmtica) . Todo nmero


natural n maior que 1 pode ser escrito como um produto
m 1 2 3 n = p 1 p2 p3 pm ,
(3.34)

onde m 1 um nmero natural, i N e pi primo para todo 1 i m . Alm disso, a fatorao em (3.34) nica se exigirmos que p1 < p2 < < pm . Demonstrao.
Seja

um inteiro maior que 1. Denotando por

p1

seu

menor divisor primo tem-se que

n = p1 1 ,
Se

1 1 < n.

1 = 1,

contrrio,

N1 = p1 e a fatorao desejada obtida. Caso denotando por p2 o menor divisor primo de 1 tem-se que
ento

n = p1 p2 2 ,
Se

1 2 < 1 .

2 = 1, ento n = p1 p2 e novamente chegamos fatorao desejada. Caso contrrio, denotando por p3 o menor divisor primo de 2 tem-se
que

n = p1 p2 p3 3 ,

1 3 < 2 . n ,
ou seja,

Continuando este processo sucessivamente obtemos ento uma sequncia estritamente decrescente de nmeros naturais

n > 1 > 2 > 2 > > n > n+1 > 1,


Ento, pelo princpio da boa ordem, s pode existir uma quantidade nita de ndices onde segue que

tais que

n > 1

e consequentemente

n+1 = 1,

de

n = p1 p2 pn .

3.5

Procurando Primos

135

Notemos que na representao acima os

pi

podem-se repetir, resul-

tando nalmente a representao desejada em (3.34). Provaremos agora a unicidade de tal fatorao. Com efeito, suponha que existem duas fatoraes:

1 2 3 m s 1 2 3 p 1 p2 p 3 p m = n = q1 q2 q3 qs
Pela Proposio 3.43 temos que cada

pi

divide algum

qj ,

logo

pi =

qj ,

por serem primos.

Portanto, cada

pi

aparece no lado direito da

qj tambm aparece no lado esquerdo da igualdade. Ento, como os pi s e os qj s so diferentes dois a dois e organizados crescentemente, temos m = s
igualdade acima, e, um argumento anlogo nos d que cada e a igualdade se reduz a

1 2 3 m m 1 2 3 p 1 p 2 p3 pm = p1 p2 p3 pm .
Suponhamos agora que neralidade vamos supor

1 seja diferente de 1 ; que 1 < 1 . Portanto,

sem perda de ge-

1 1 2 3 3 m 2 m p p2 p3 p 2 p3 pm = p 1 m ,

1 1 > 0 ento, pela Proposio 3.43 temos que p1 divide algum pj , com j > 1, o que impossvel. Portanto, 1 = 1 . Similarmente provamos que i = i , com i = 1, . . . , n.
e como

ciado precisamente por Gauss (1777-1855). Seus antecessores, Fermat, Euler, Lagrange e Legendre, utilizavam este teorema sem a preocupao de t-lo enunciado ou demonstrado com preciso. Uma prova alternativa deste teorema ser apresentada no Captulo 6, usando o mtodo de induo.

Observao 3.50. O teorema fundamental da aritmtica foi enun-

136

Divisibilidade

nmero 2 aparece na fatorao de n em fatores primos. Soluo.


ento

Exemplo 3.51. Prove que um nmero n par se, e somente se, o


Obviamente, se 2 aparece na fatorao em primos de par. Ora, se

par temos que

n = 2q .

Por outro lado

N, qe

se fatoram, respectivamente, como

1 2 m q = q1 q2 qm
Logo,

s 1 2 n = p 1 p2 ps .

1 2 m s 1 2 2 q1 q2 q m = p 1 p2 ps .
Pela unicidade da fatorao, para algum respondente

i,

com

pi

deve ser igual a 2. Portanto, 2 aparece na fatorao de

1 i s,

o cor-

n.

Exemplo 3.52. Seja A = {1, 2, 3, 4, 5, 6, 7}. possvel decompor


o conjunto A em dois subconjuntos disjuntos tais que o produto dos elementos de um seja igual ao produto dos elementos do outro? Soluo.
Mostraremos que impossvel fazer esta decomposio. Com

efeito, suponha que existem tais conjuntos,

A2 = {q1 , q2 , . . . , qs }.

Ento

A1 = {p1 , p2 , . . . , pr }

p1 p2 pr = q1 q 2 qs

e alm disso, como os conjuntos nmero 5 aparece no produto

A1

A2

so disjuntos, temos que o

ou no produto

mas no em ambos

simultaneamente. Por outro lado, o Teorema 3.49 nos diz que a fatorao em primos de 5 deveria

igual fatorao em primos de , logo o nmero aparecer tanto no produto como no produto , contra-

dizendo isto o fato anterior. Portanto no existe uma decomposio com as condies exigidas.

3.5

Procurando Primos

137

Exemplo 3.53. Encontre todos os nmeros inteiros e positivos n com


a propriedade de que o conjunto

A = {n, n + 1, n + 2, n + 3, n + 4, n + 5}

pode ser particionado em dois subconjuntos tais que o produto dos elementos de um dos subconjuntos seja igual ao produto dos elementos do outro. Demonstrao.
gum Digamos que seja possvel essa decomposio para al-

n e vamos denotar os conjuntos que obtemos com a decomposio por A1 e A2 . Observando a decomposio dos elementos dos subconjuntos em fatores primos, temos que todo fator primo de A1 tambm dever pertencer a A2 . No conjunto dos seis nmeros s podemos ter um mltiplo de 7, por isso no podemos tomar n como mltiplo deste primo. Analogamente para primos maiores que 7. Analisando o primo 5, conclumos que n e n + 5 so mltiplos de 5, pois se no, cairamos na anlise anterior. Assim, os nmeros n + 1, n + 2, n + 3 e n + 4 so da forma 2 3 . Como entre eles existem dois mpares, logo teremos duas potncias de 3 cuja diferena 2, um absurdo. Assim, no existe n que satisfaz as condies do enunciado.

Finalizamos esta seo com um exemplo que mostra como podemos combinar os fatos estudados para resolver problemas mais difceis

algarismos da forma aabb e que sejam quadrados perfeitos.

Exemplo 3.54. Encontre todos os nmeros que so formados por 4

138

Divisibilidade

Soluo.

Como o nmero

aabb

um quadrado perfeito, signica que:

n2 =aabb n2 =103 a + 102 a + 10b + b = 103 + 102 a + (10 + 1) b n2 =1100 a + 11 b n2 =11 100a + b = 11 99a + a + b . 112 | N 2 . Segue-se ento que 11 | (99a + a + b). Portanto, 11 | (a + b). Como aabb tem 4 algarismos, segue-se que a = 0; portanto a {1, 2, 3, . . . , 9} e b {0, 1, 2, . . . , 9}. De onde a + b 18. Logo, necessariamente devemos ter a + b = 11. Podemos observar que a = 1, pois se a = 1 ento b = 10. Analogamente, b = 0, 1. Portanto,
Como 11 primo fcil ver, usando a Proposio 3.43, que

a {2, 3, 4, . . . , 9}
somente pode acabar em

b {2, 3, 4, . . . , 9}.
e

Como em todo nmero quadrado perfeito o algarismo das unidades

0, 1, 4, 5, 6

9.

Segue-se que

b {4, 5, 6, 9}.
Certamente ao

b = 5, pois todo nmero que acaba em 5 quando elevado quadrado sempre acaba em 25. Assim, b {4, 6, 9}.
Se

b = 4,

ento

a = 7.

Neste caso o nmero seria

7.744

que

um quadrado perfeito; Se

b = 6,

ento

a = 5.

Neste caso o nmero seria

5.566

que no

um quadrado perfeito;

3.6

Exerccios

139
ento

Se

b = 9,

a = 2.

Neste caso o nmero seria

2.299

que no

um quadrado perfeito.

Finalmente, a nica soluo possvel

aabb = 7.744 = 882 .

3.6

Exerccios

1. Encontre o resto que deixa (a) (b) (c)

2001 2002 2003 2004 + 20052 2100


quando dividido por 3;

quando dividido por 7;

(1237156 + 34)

28

quando dividido por 111.

2. Provar que o nmero nmero natural 3. Prove que se (a) (b) (c)

n5 + 4 n

divisvel por 5 para qualquer

n.
mpar

n3 n n2 1

divisvel por 24; divisvel por 8; divisvel por 12.

n2 + (n + 2)2 + (n + 4)2 + 1 21093 2

4. O nmero 5. Prove que 6. O nmero

divisvel por

10932

(999994)1234567890 1 N = 42005 + 20054

divisvel por

333331.

primo? composto.

7. Demonstre que o nmero

1 000 . . . 00 1
2006 zeros

140

Divisibilidade

8. Utilizando o fato de que o resto de um quadrado quando dividido por 4 s pode ser 0 ou 1, d uma outra soluo para o problema do Exemplo 3.54. 9. Dados trs inteiros, e

x, y, z ,

tais que

x2 + y 2 = z 2 ,

mostre que

no so ambos mpares e que

xy

mltiplo de 6.

10. Demonstre que o quadrado de um inteiro da forma ou

8n ou 8n +1

8n + 4. p, q
e

11. Trs nmeros primos

gresso aritmtica, ou seja, divisvel por 6.

r, maiores que 3, formam uma proq = p + d e r = p + 2d. Prove que d

12. Demonstrar que existem innitos nmeros primos da forma

4m +

e da forma

6m + 5,

onde

m Z.

13. Encontrar o ltimo dgito dos nmeros (a) (b) (c)

19892005 ; 777777 + 250 ; 1 + 22 + 32 + + 20052 .

14. Prove que a soma dos quadrados de cinco nmeros consecutivos no um quadrado perfeito. 15. Prove que

1 00 00 5 00 00 1
100zeros 100zeros

no um cubo perfeito.

16. Seja

um inteiro positivo. Enuncie e prove o critrio de divisi-

bilidade por

no sistema de numerao de base

b.

17. Prove que os nmeros

3.6

Exerccios
(a)

141

(b)

1 1 1 + + + , com n > 1, 2 3 n 1 1 1 n = + + + , com n > 0, 3 5 2n + 1 n = 1 +

no so inteiros.

18. Considere o polinmio

p(n) = am nm + am1 nm1 + + a0 de grau m 1 com coecientes inteiros e n N. Prove que p(n) um nmero composto para innitos valores de n.

Sugesto: Use o fato de que existe a N tal que = |p(a)| > 1 e mostre que divide a p(k + a), para todo k Z.
19. Dizemos que um conjunto

An

formado por

inteiros positivos

escritos no sistema binrio (base 2) regular se, para qualquer

inteiro no negativo a quantidade de nmeros de

An

que con-

templam

na representao binria par. Dizemos que

An

irregular se, pelo menos para algum

s, este nmero mpar.

De-

monstre que um sistema irregular pode se converter em regular excluindo-se apenas um nico elemento do mesmo, e, um sistema regular pode se converter em irregular excluindo-se qualquer um dos seus elementos.

20. Seja

n um inteiro positivo. n

Demonstrar que todos os coecientes

do desenvolvimento do binmio de Newton se, e somente se, da forma

(a + b)n

so mpares

2 1.

21. Prove que se

(x0 , y0 ) uma soluo da equao diofantina linear ax by = 1, ento a rea do tringulo cujos vrtices so (0, 0), (b, a) e (x0 , y0 ) 1/2.

142

Divisibilidade

22. Qual a menor distncia possvel entre dois pontos

(x1 , y1 )

(x2 , y2 ), com coordenadas inteiras, situados sobre a reta denida pela equao diofantina ax + by = c?

4
O Princpio da Casa dos Pombos

Um interessante instrumento elementar para tratar problemas matemticos relacionados existncia de elementos de conjuntos validando certas exigncias o chamado cido como

princpio de Dirichlet , tambm conheprincpio da casa dos pombos (PCP) . Este princpio foi

usado por Dirichlet (1805-1859) para resolver problemas na Teoria dos Nmeros, entretanto ele possui um grande nmero de aplicaes em diversos ramos da Matemtica como Combinatria e Geometria. A seguir enunciamos a verso mais simples do PCP.

pombos em N casas, ento alguma das casas contm dois ou mais pombos.
143

Proposio 4.1

(PCP  Verso Simples)

. Se distribumos N + 1

144

O Princpio da Casa dos Pombos

P1 C1

P2 C2

PN CN

PN +1
Figura 4.1: Em cada casa

denotado por

Pj .

O pombo restante, denotado por

Cj , 1 j N ,

coloca-se um nico pombo,

PN +1 ,

deve ir para

alguma das casas, juntando-se ao que j se encontrava contido nela

Demonstrao.

A prova deste princpio muito fcil e decorre de fa-

zer uma simples contagem dos pombos contidos em todas as casas depois de distribudos. Com efeito, suponhamos pelo contrrio que em cada casa no existe mais do que um pombo, ento contando todos os pombos contidos nas

casas no teremos mais do que

pombos,

contradizendo isto a hipteses de termos nas

N +1

pombos distribudos

casas (ver Figura 4.1).

No difcil detectar quando o princpio pode ser usado, mas a principal diculdade para aplic-lo reside em identicar, em cada problema, quem faz papel de pombos e quem faz papel de casas. Nas seguintes sees discutiremos vrios exemplos de diferentes naturezas onde o

princpio da casa dos pombos

aplicado com sucesso.

4.1

Primeiros Exemplos

145

4.1

Primeiros Exemplos

que uma jaqueira no contm mais do que 600 frutos. Prove que existem 2 jaqueiras na oresta que tm a mesma quantidade de frutos. Soluo.
0, 1, 2, 3, . . . , 600. k

Exemplo 4.2. Numa oresta crescem 1.000 jaqueiras. conhecido

Temos 1.000 jaqueiras, representando os pombos, e 601 casas O nmero associado

identicadas pelos nmeros

a cada casa signica que nela sero colocadas jaqueiras que tm exatamente

frutos. Como

1000 > 602 = 601 + 1,

PCP

nos garante

que existem duas jaqueiras com a mesma quantidade de frutos.

duas pessoas que conhecem exatamente o mesmo nmero de pessoas. Soluo.


Os pombos neste caso so as

Exemplo 4.3. Em uma reunio h n pessoas. Mostre que existem


n pessoas.
As casas so enume-

radas com os nmeros

sero colocadas pessoas que tm essa quantidade de conhecidos. Notemos que uma das casas enumeradas com

0, 1, 2, . . . , n 1, indicando estes que na mesma

0 ou n 1 permanece desocupada, pois a possibilidade de conhecer 0 e n 1 pessoas no acontece simultaneamente. Logo, nas n 1 casas restantes haver
uma ocupada por dois ou mais pombos, depois de serem distribudos. Portanto, existem no mnimo duas pessoas com o mesmo nmero de conhecidos.

deles cuja diferena divisvel por 7. Soluo.


7+1 7

Exemplo 4.4. Dados 8 nmeros inteiros mostre que existem dois


Consideramos os 8 nmeros como sendo os pombos e as casas possveis restos na diviso por 7. Como temos

como sendo os

8=

nmeros o

PCP

nos diz que existem dois nmeros dentro dos

146

O Princpio da Casa dos Pombos

8 dados que tm o mesmo resto quando divididos por 7. Finalmente, observamos que se dois nmeros deixam o mesmo resto na diviso por 7 ento a diferena entre eles divisvel por 7. Uma forma alternativa e muito til na qual pode-se apresentar o princpio da casa dos pombos a seguinte:

Proposio 4.5

(PCP  Verso Alternativa)

meros naturais igual S , ento existe pelo menos um deles que no maior que S/n, assim como existe pelo menos um deles que no menor que S/n.

. Se a soma de n n-

Exemplo 4.6. Numa famlia formada por 5 pessoas a soma das idades
de 245 anos. Prove que podem ser selecionados 3 membros da famlia cuja soma das idades no menor que 147. Soluo.
em Temos um total de

5 3

5! 3!2!

= 10

trios diferentes formados

por membros da famlia. Alm disso, cada pessoa aparece exatamente

4 2

4! 2!2!

=6

trios. Ento, denotando por

Ej

a soma das idades

dos membros de cada trio

Tj , j = 1, 2 . . . 10,

temos que

E1 + E2 + + E10 = 6 245 = 1470;


consequentemente existe algum trio

Tj

tal que

Ej

1470 10

= 147.

4.2

Uma Verso mais Geral

A seguinte verso mais geral do alguns problemas.

PCP

bastante til na resoluo de

bos em N casas, ento alguma das casas contm pelo menos k + 1 pombos.

Proposio 4.7 (PCP  Verso Geral). Se distribumos N k + 1 pom-

4.2

Uma Verso mais Geral

147

A prova deste enunciado mais geral similar anterior. Com efeito, suponhamos pelo contrrio que em cada casa no existe mais do que

pombos, ento contando todos os pombos contidos nas

casas no

teremos mais do que termos

Nk

pombos, contradizendo isto a hipteses de

Nk + 1

pombos distribudos nas

casas.

Notemos que se mais simples.

k = 1, esta verso mais geral coincide com a verso

Exemplo 4.8. Num colgio com 16 salas so distribudas canetas nas


cores preta, azul e vermelha para realizar uma prova de concurso. Se cada sala recebe canetas da mesma cor ento prove que existem pelo menos 6 salas que receberam canetas da mesma cor. Soluo.
3 5 + 1.
Fazendo a diviso com resto de 16 por 3 temos que

16 =

Consideramos as 16 salas como sendo os pombos e as trs Logo, podemos

cores, preto, azul e vermelho como sendo as casas. colocar cada sala em uma das trs cores. Assim, o e

PCP

com

N =3

k = 5 nos d que existe uma casa com pelo menos 6 pombos, ou seja,

existem no mnimo 6 salas que receberam canetas da mesma cor.

selecionada para representar o Brasil numa olimpada internacional. Mostre que necessariamente existem trs deles que se conhecem mutuamente, ou trs deles que no se conhecem mutuamente. Soluo.
aluno

Exemplo 4.9. Uma equipe formada por seis alunos de Matemtica

Resolveremos o problema com o auxlio da Figura 4.2. Cada com

Aj ,

j = 1, 2, . . . , 6,

representado por um dos vrtices de

um hexgono regular.

Quando dois alunos se conhecem traamos o

segmento de reta que liga os vrtices correspondentes com uma linha contnua; caso contrrio traamos este segmento com uma linha pontilhada. Logo, usando este esquema, o problema equivale a provar

148

O Princpio da Casa dos Pombos

que sempre existe um tringulo de lados contnuos ou um tringulo de lados pontilhados com vrtices no conjunto

A = {A1 , A2 , . . . , A6 }. A1 ,
cada um

Temos 5 segmentos (pombos) incidindo no vrtice

deles contnuo ou pontilhado (estes dois tipos de linhas so consideradas como as casas). Como

dos 5 segmentos so contnuos ou pontilhados. Suponhamos que 3 so contnuos (caso contrrio o argumento similar) e denotemos estes por

5 = 2 2 + 1,

pelo

PCP

temos que 3

A1 A3 , A1 A4 e A1 A6 A3 A4 , A3 A6 ou A4 A6 for se ligam com A1 formam

(ver Figura 4.2).

Se algum dos segmentos

contnuo ento este segmento junto aos que um tringulo de lados contnuos. Por outro

lado, se nenhum deles for contnuo, ento eles formam um tringulo de lados pontilhados, completando isto a demonstrao.

A3

A2

A4

A1

A5
Figura 4.2: O tringulo

A6
indica que os alunos

A1 A2 A5

A1 , A2

A5

no se

conhecem mutuamente e o tringulo e

A1 A4 A6

indica que os alunos

A1 , A4

A6

se conhecem mutuamente

4.3

Aplicaes na Teoria dos Nmeros

149

4.3

Aplicaes na Teoria dos Nmeros

Nesta seo apresentamos alguns exemplos de aplicaes do Teoria dos Nmeros. A primeira delas :

PCP

na

A = {m + 1, m + 2, . . . , m + n} possui algum divisor de n.

Exemplo 4.10. Se n e m so nmeros naturais, ento o conjunto


Soluo.
de Temos

n nmeros diferentes no conjunto acima. A

Vamos utili-

zar o mtodo de reduo ao absurdo. Se no existisse nenhum mltiplo

n,

quando dividssemos os nmeros do conjunto

por

n,

os res-

tos pertenceriam ao conjunto

B = {1, 2, . . . , n 1}, que possui n 1 elementos. Logo, devem existir dois nmeros m + i e m + j , com 1 i < j n tais que o resto da diviso de m + i por n o mesmo que o resto da diviso de m + j por n. Logo, m + j (m + i) um mltiplo de n, o que implica que n > j i 1 mltiplo de n menor que n (absurdo!). Logo, deve existir algum mltiplo de n no conjunto A.
Como consequncia desse exemplo, podemos resolver o prximo

problema.

Exemplo 4.11. Demonstrar que todo inteiro tem um mltiplo cuja


representao decimal comea com o bloco de dgitos 1234567890. Soluo.
Se

m e n so inteiros positivos, pelo exemplo anterior um dos nmero m + 1, m + 2, . . . , m + n mltiplo de n. Assim, dado n n+1 um inteiro qualquer, escolhe-se m = 1234567890 10 . Deste modo, todos os inteiros m + 1, m + 2, . . . , m + n comeam com 1234567890 e algum deles mltiplo de n.

150

O Princpio da Casa dos Pombos

Exemplo 4.12. Dado um nmero inteiro positivo n, mostre que existe


um mltiplo de n que se escreve com os algarismos 0 e 1 apenas. (Por exemplo, se n = 3, temos 111 ou 1.101 etc.) Soluo.
Consideramos os

n+1

nmeros

1, 11, 111, 1111, . . . , 111 1


como sendo os pombos e

(4.1)

n+1vezes

casas enumeradas com os nmeros

0, 1, 2, 3, . . . , n 1,
ou seja, com os possveis restos na diviso por

n.

Similarmente ao

exemplo anterior existem dois nmeros na lista (4.1) que deixam o mesmo resto na diviso por o menor mltiplo de

e, portanto, a diferena entre o maior e

n.

Obviamente a diferena entre dois nmeros

quaisquer da lista (4.1) resulta em um nmero formado apenas pelos algarismos 0 e 1.

Exemplo 4.13. Prove que entre n + 1 elementos escolhidos no conjunto {1,2,3, . . . , 2n} existem dois que so primos relativos. Soluo.
A escolha das casas e dos pombos neste exemplo no to b-

via. Os pombos representam os

{1, 2, . . . , 2n}

e as casas so

n + 1 nmeros escolhidos do conjunto escolhidas como sendo os n conjuntos:

Cj = {2j 1, 2j }, 1 j n.
Logo, pelo tos

PCP, quando distribumos os n + 1 nmeros nos n conjunou

seja, estes nmeros sero consecutivos e portanto primos entre si.

Cj , 1 j n, dois deles caro juntos em algum conjunto Cj ,

4.4

Aplicaes Geomtricas

151

Finalizaremos esta seo com uma outra prova do teorema de Bachet-Bzout, (veja o Teorema 3.23).

Exemplo 4.14. Seja d = (a, b) o mdc entre os nmeros naturais a e


b. Ento, existem x e y nmeros inteiros tais que ax + by = d.

Soluo.
b

Denotando por

m = a/d

n = b/d,

podemos supor que

so primos entre si. Realmente, se podemos escrever

mx + ny = 1
ento, substituindo os valores de

na equao acima, temos que

ax + by = d. Se (a, b) = 1, considere a sequncia A = {a, 2a, . . . , ba}. Armamos que existe algum nmero no conjunto A que deixa resto 1 quando dividido por b. De fato, se isso no ocorresse, teramos b nmeros em A deixando no mximo b 1 restos diferentes quando divididos por b. Logo, pelo PCP, dois deles, digamos ia e ja com b > j > i 1, devem deixar o mesmo resto quando divididos por b. assim, (j i)a divisvel por b. Como estamos supondo que (a, b) = 1, temos que b deve dividir j i > 0. Como b > j i, temos um absurdo. Assim, algum dos nmeros em a deixa resto 1 quando divididos por b. Digamos que esse nmero seja ax. Logo, ax 1 mltiplo de b, onde ax 1 = by , o que encerra nossa prova.

4.4

Aplicaes Geomtricas

Na geometria tambm encontramos belas aplicaes do os problemas a seguir para constatar isto.

PCP. Vejamos

152

O Princpio da Casa dos Pombos

Exemplo 4.15. Mostre que se tomamos cinco pontos quaisquer sobre


um quadrado de lado 1, ento pelo menos dois deles no distam mais que 2/2.
como mostra a gura. Logo, pelo

Soluo.
1/2,

Vamos dividir o quadrado em quatro quadradinhos de lado

PCP

pelo menos dois deles de-

vem estar no mesmo quadradinho, uma vez que temos 4 quadradinhos e 5 pontos. Logo, como a maior distncia num quadrado a diagonal, o Teorema de Pitgoras nos garante que a distncia desses dois pontos no mximo

2/2,

como queramos mostrar.

lado 4 so marcados 10 pontos no interior deste. Prove que existe ao menos um par destes pontos cuja distncia entre eles no maior que 3. Soluo.
equilteros menores de lado 1, conforme a Figura 4.3.

Exemplo 4.16. Na regio delimitada por um tringulo equiltero de

Dividimos o tringulo equiltero de lado 4 em 16 tringulos

Agora pintamos os tringulos nas cores branco e cinza de maneira que dois tringulos vizinhos, isto , com um lado comum, so pintados de cores diferentes. Se tivssemos dois pontos no mesmo tringulo a distncia mxima possvel entre eles seria 1 e o problema estaria resolvido. Se tivssemos pontos em tringulos vizinhos, a maior distncia possvel entre eles seria

e tambm isto resolveria o problema. Se

no tivssemos nenhum dos casos anteriores, no seria difcil ver que

4.5

Miscelnea

153

C A D

E B
3

Figura 4.3: O tringulo

DBE

equiltero de lado

os 10 pontos deveriam estar situados sobre os 10 tringulos brancos, contendo cada tringulo exatamente um ponto. Dividindo o tringulo

DBE

3/2 pelo PCP temos que pelo menos dois dos 6 pontos contidos em DBE esto num destes 4 tringulos, logo a distncia entre eles no maior que 3/2 < 3. Com
em 4 tringulos congruentes de lado

isto terminamos nossa prova.

4.5

Miscelnea

Os problemas que apresentamos a seguir usam o PCP combinado com outras idias que so muito empregadas nas suas solues.

um dos nmeros: -1, 0 ou 1. Prove que entre todas as somas das linhas, colunas e diagonais do tabuleiro h duas que so iguais. Por exemplo, no tabuleiro abaixo a soma da segunda linha 2, que coincide com a soma da terceira coluna.

Exemplo 4.17. Em cada quadradinho de um tabuleiro 3 3 colocado

154

O Princpio da Casa dos Pombos

-1 1 0

-1 0 -1

1 1 0

Soluo.

S = a1 + a2 + a3 , onde cada a1 , a2 e a3 podem tomar valores: 1, 0 e 1. Ento, temos 7 valores possveis para S (casas), que so: 3, 2, 1, 0, 1, 2, 3. O tabuleiro 3 3 tem 3 linhas, 3 colunas e 2 diagonais, portanto, ao
Seja somarmos os elementos de cada uma das linhas, colunas e diagonais, obteremos 8 nmeros (pombos). possveis para estes nmeros, pelo ser iguais. Como existem somente

valores

PCP

pelo menos dois deles devem

naturais escolhidos entre 1 e 99, inclusos, demonstre que existem dois subconjuntos disjuntos e no vazios de A tal que a soma dos seus respectivos elementos igual. Soluo:
conhecido que

Exemplo 4.18. Dado qualquer conjunto A formado por 10 nmeros

tem

vazios diferentes.

A soma dos elementos de cada um deles d uma

210 1 = 1.023

subconjuntos no-

quantidade menor do que 1.000, pois o subconjunto com no mximo 10 elementos de maior soma possvel o formado por e nesse caso

90, 91, . . . , 99,

como sendo os 1.023 subconjuntos distintos de

90 + 91 + + 99 = 945.

Agora consideramos os pombos

e as casas como

sendo as somas possveis dos elementos de cada um dos conjuntos. Logo, como o nmero de conjuntos maior que o nmero de somas possveis, devem existir dois conjuntos a soma dos elementos de

de

A,

de tal modo que

igual soma dos elementos de

C.

Se

4.5

Miscelnea

155

e e

dos seus elementos a mesma, pois retiramos de ambos a mesma quantidade.

C so disjuntos, acabou a prova. Se no, considere D = B B C E = C B C . Logo, os conjuntos D e E so disjuntos e a soma

buleiro de 8 8 que podem ser pintados de preto, de forma tal que qualquer arranjo de trs quadradinhos, como mostra a Figura 4.4, tenha pelo menos um dos quadradinhos no pintado de preto?

Exemplo 4.19. Qual o maior nmero de quadradinhos de um ta-

Figura 4.4: Tridomins

Soluo.

Primeiramente, pintamos o tabuleiro de

leiro de jogar xadrez, ou seja, 32 quadradinhos pintados de branco e 32 quadradinhos pintados de preto (ver Figura 4.5).

8 8 como um tabu-

Figura 4.5: Tabuleiro de xadrez

156

O Princpio da Casa dos Pombos

Notemos que uma vez pintado o tabuleiro desta forma satisfeita a exigncia do problema, pois nunca temos 2 quadradinhos vizinhos (quadradinhos com um lado comum) pintados de preto. Mostraremos agora que se pintamos 33 quadradinhos de preto ento a condio exigida no problema falha. tabuleiro em 16 quadrados de De fato, se dividimos o

dinhos de preto (pombos); ento, como geral do

22

(casas) e pintamos 33 quadra-

PCP

um dos 16 quadrados de

pintados de preto. Portanto, este ltimo contm um arranjo como na Figura 4.4 completamente pintado de preto.

33 = 16 2 + 1, pela verso 2 2 contm 3 quadradinhos

Resumindo, o nmero mximo de quadradinhos que podemos pintar de preto 32.

Exemplo 4.20. Dados sete nmeros reais arbitrrios, demonstre que


existem dois deles, digamos x e y , tais que 0 Soluo.
sar na frmula

1 xy 1 + xy 3
x y nos faz pen1+xy

Primeiramente observamos que a expresso

tan( ) =
Sejam

tan tan . 1 + tan tan

(4.2)

Lembramos que a funo tangente uma bijeo entre o intervalo

x1 , x2 , , x7

os sete nmeros selecionados arbitrariamente.

, ) e os nmeros reais R, logo para cada xi , 1 i 7, existe um ( 2 2 , ) tal que tan(i ) = xi . Dividimos o intervalo ( , ) em i ( 2 2 2 2
seis subintervalos de comprimento

, como mostra o desenho a seguir. 6

Pelo

PCP

dois dos nmeros

i
e

pertencem ao mesmo subintervalo.

Denotemos os mesmos por

i1

i2

e suponhamos, sem perda de

4.6

Exerccios

157

i1 2
generalidade, que

i2
6 2

i1 i2 .

Ento vale

0 i2 i1
(4.2) temos que

. 6

Usando o fato de que a tangente uma funo crescente e a frmula

tan(0) tan(i2 i1 ) tan( ). 6


Equivalentemente,

xi 2 xi 1 1 . 1 + xi 2 xi 1 3

4.6

Exerccios
C
um conjunto formado por cinco pontos de coordenadas

1. Seja

inteiras no plano. Prove que o ponto mdio de algum dos segmentos com extremos em

tem tambm coordenadas inteiras.

2. O conjunto dos dgitos 1, 2, ..., 9 dividido em trs grupos. Prove que o produto dos nmeros de algum dos grupos deve ser maior que 71. 3. Prove que se

mpar ento para qualquer bijeo

p : IN IN

158
do conjunto

O Princpio da Casa dos Pombos

IN = {1, 2, . . . , N } o produto P (p) = (1 p(1))(2 p(2)) (N p(N )) necessariamente par.


(Dica: O produto de vrios fatores par se, e somente se, um dos fatores par.)

4. Dado um conjunto de 25 pontos no plano tais que entre quaisquer 3 deles existe um par com distncia menor que 1. Prove que

existe um crculo de raio 1 que contm pelo menos 13 dos 25 pontos dados. 5. Prove que entre quaisquer 5 pontos escolhidos dentro de um tringulo equiltero de lado 1 sempre existe um par deles cuja distncia no maior que 0,5. 6. Marquemos todos os centros dos 64 quadradinhos de um tabuleiro de xadrez de

linhas retas que no passem pelos pontos marcados e de forma tal que cada pedao de recorte do tabuleiro tenha no mximo um ponto marcado? 7. Prove que existem duas potncias de 3 cuja diferena divisvel por 1.997. 8. So escolhidos 6 nmeros quaisquer pertencentes ao conjunto

8 8.

possvel cortar o tabuleiro com 13

A = {1, 2, 3, . . . , 10}.
Prove que existem dois desses seis nmeros cuja soma mpar. 9. Seja

um nmero real arbitrrio. Prove que entre os nmeros

x, 2x, 3x, . . . , 101x

4.6

Exerccios

159

existe um tal que sua diferena com certo nmero inteiro menor 0,011. 10. Mostre que entre nove nmeros que no possuem divisores primos maiores que cinco, existem dois cujo produto um quadrado. 11. Um disco fechado de raio um contm sete pontos, cujas distncias entre quaisquer dois deles maior ou igual a um. Prove que o centro do disco um destes pontos. 12. Na regio delimitada por um retngulo de largura quatro e altura trs so marcados seis pontos. Prove que existe ao menos um

par destes pontos cuja distncia entre eles no maior que 13. Seja

5.

um nmero irracional. Prove que existem innitos nme-

ros racionais

r = p/q

tais que

|a r| < 1/q 2 .

14. Suponha que cada ponto do reticulado plano pintado de vermelho ou azul. Mostre que existe algum retngulo com vrtices no reticulado e todos da mesma cor. 15. Um certo livreiro vende pelo menos um livro por dia. Sabendo que o livreiro vendeu 463 livros durante 305 dias consecutivos, mostre que em algum perodo de dias consecutivos o livreiro vendeu exatamente 144 livros.

Referncias Bibliogrcas
[1] AIGNER, M. e ZIEGLER, G. (2002).

As Provas esto Elementos de l-

no Livro. gebra.

Edgard Blcher.

[2] GARCIA, A. e LEQUAIN, I. (2003). Projeto Euclides, IMPA.

[3] LIMA, E. L.; CARVALHO, P. C. P.; WAGNER, E. e MORGADO, A.C. (2004).

A Matemtica do Ensino M-

dio. Volume 1.

Sociedade Brasileira de Matemtica.

[4] LIMA, E.L.; CARVALHO, P. C. P.; WAGNER, E. e MORGADO, A.C. (2004).

A Matemtica do Ensino M-

dio. Volume 2.
[5] LIMA,E.L.;

Sociedade Brasileira de Matemtica. C. P.; WAGNER,E. e

CARVALHO,P.

MORGADO,A.C. (2004).

A Matemtica do Ensino M-

dio. Volume 3.

Sociedade Brasileira de Matemtica.

[6] LIMA, E.L.; CARVALHO, P. C. P.; WAGNER,E. e MORGADO, A.C. (2001).

Temas e Problemas.

Socie-

dade Brasileira de Matemtica. [7] LIMA, E.L. (2001). de Matemtica.

lgebra Linear. Sociedade Brasileira

285

286

REFERNCIAS BIBLIOGRFICAS
[8] MORAIS FILHO, D. C. (2007).

Um Convite MatemCARVALHO, P.;

tica.

EDUFCG. CARVALHO, J.;

[9] MORGADO, A.;

FERNANDEZ, P. (1991).

Anlise Combinatria e ProNmeros Primos: Mistrios e Introduo Teoria dos N-

babilidade . Recordes. meros.

Sociedade Brasileira de Matemtica.

[10] RIBENBOIM, P. (2001).

Sociedade Brasileira de Matemtica.

[11] SANTOS, J. P. O. (1993) IMPA.

[12] SANTOS, J. P. O.; MELLO, M. P. e MURARI, I. T. C. (2006). Unicamp. [13] SOARES, M. G. (2005).

Introduo Anlise Combinatria.

Editora

Clculo em uma Varivel Com-

plexa.

Sociedade Brasileira de Matemtica.

em Matemtica em Rede Nacional

Mestrado Profissional

Iniciao Matemtica

Autores: Krerley Oliveira Adn J. Corcho

Unidade III: Captulos V e VI

160

5
Contagem

Neste captulo discutiremos problemas envolvendo a contagem de elementos de um conjunto nito dado. Por exemplo, responderemos perguntas do tipo: de quantos modos podemos distribuir 32 selees nacionais de futebol em seis grupos de quatro times cada? Para solucionar questes como esta, utilizaremos como ferramentas bsicas os princpios aditivo e multiplicativo da contagem. Veremos tambm que o uso simultneo destes princpios ser muito til para resolver problemas com certos nveis de complexidade. Alm disso, sero abordados os conceitos de permutaes, arranjos e combinaes, sendo estes de muita importncia por serem os alicerces de um ramo da matemtica denominado combinatria. Antes de prosseguirmos daremos algumas denies e notaes que sero teis ao longo de todo o captulo. Dado um conjunto A denotamos por |A| a quantidade de elementos que este possui. O produto cartesiano de n conjuntos A1 , A2 , . . . , An1 e An o conjunto denido 161

162

Contagem

por
A1 A2 An := (a1 , a2 , . . . , an ); ai Ai , i = 1, 2, . . . , n ,

onde cada elemento (a1 , a2 , . . . , an ) chamado de n-upla ordenada. Denotaremos o conjunto vazio com o smbolo . O leitor que deseja rever os conceitos bsicos da teoria de conjuntos, pode ach-los muito bem expostos em [3].
5.1 Princpio Aditivo da Contagem

O princpio aditivo nitos que no tm unio exatamente seja, se A1 e A2 so

da contagem garante que dados dois conjuntos elemento em comum, o nmero de elementos da a soma do nmero de elementos de cada um, ou disjuntos (isto , A1 A2 = ), ento
|A1 A2 | = |A1 | + |A2 |.

Apesar de sua simplicidade, muitos problemas podem ser resolvidos utilizando esse simples princpio. A seguir enunciamos uma extenso deste princpio para um nmero nito qualquer de conjuntos.

Princpio Aditivo da contagem: Dados os conjuntos nitos A1 ,


A2 , . . . , An dois a dois disjuntos (isto , Ai Aj = , i = j ),

temos que

|A1 A2 An | = |A1 | + |A2 | + + |An |.

Exemplo 5.1. Em Macei entraram em cartaz 4 lmes distintos e


2 peas de teatro. Se Pedro Vtor s tem dinheiro para assistir a um

lme ou a uma pea de teatro, diga quantos so os possveis programas de Pedro Vtor.

5.1

Princpio Aditivo da Contagem

163

Soluo. Denotemos por f1 , f2 , f3 e f4 os quatro lmes que esto em cartaz e por t1 e t2 as duas peas de teatro. Agora, representemos pelo par (i, j ), com 0 i 4 e 0 j 2, o programa que consiste em assistir ao lme fi e pea tj (caso i = 0 ou j = 0 isso signica que no ser assistido a nenhum lme ou a nenhuma pea, respectivamente). Pelas limitaes econmicas do Pedro Vtor temos que ele s pode escolher um programa dentro dos seguintes conjuntos disjuntos:
A1 = (1, 0), (2, 0), (3, 0), (4, 0)

e A2 = (0, 1), (0, 2) .

Logo, no total so |A1 A2 | = |A1 | + |A2 | = 6 programas distintos, entre os quais Pedro Vtor ter que escolher um.

todos os presentes apertaram as mos entre si. Sabendo-se que ao todo foram feitos 66 cumprimentos, calcule o nmero de pessoas presentes reunio.

Exemplo 5.2. Numa reunio havia um certo nmero de pessoas e

Soluo. Vamos enumerar as pessoas com os nmeros do conjunto P = {1, 2, . . . , n}. A cada aperto de mo associaremos um par (i, j ), signicando que a pessoa i apertou a mo da pessoa j . Assim, os apertos de mo envolvendo a pessoa 1 foram:
A1 = {(1, 2), (1, 3), . . . , (1, n)}.

Do mesmo modo, denimos os apertos de mo envolvendo a pessoa 2 que no envolvem a pessoa 1, como: Note que o aperto (2, 1) o mesmo que o aperto (1, 2), j que se 1 aperta a mo de 2, ento 2 aperta a mo de 1. Analogamente,
Ai = {(i, i + 1), (i, i + 2), . . . , (i, n)}, para 1 i n. A2 = {(2, 3), (2, 4), . . . , (2, n)}.

164

Contagem

Note que Ai Aj = para i = j . Observe tambm que todos os apertos aparecem em um dos conjuntos Ai . Assim, A1 An contm todos os apertos de mo. Logo, pelo princpio aditivo:
|A1 A2 An | = |A1 | + |A2 | + . . . |An | (n 1)n = 66. 2 Resolvendo em n, temos que n = 12. = = (n 1) + (n 2) + + 2 + 1

Vimos que o princpio aditivo nos fornece o nmero de elementos de qualquer unio de conjuntos dois a dois disjuntos. Discutiremos agora uma extenso do princpio para qualquer unio de conjuntos, no necessariamente dois a dois disjuntos.

Ento,

Proposio 5.3. Sejam A1 e A2 dois conjuntos nitos quaisquer.


|A1 A2 | = |A1 | + |A2 | |A1 A2 |. A1 A2 = (A1 A2 ) A2

Demonstrao. Observe que

onde a unio dois a dois disjunta. Pelo princpio aditivo, temos que
|A1 A2 | = |A1 A2 | + |A2 |.

(5.1)

Analogamente, aplicando novamente este princpio, temos que


|A1 | = |A1 A2 | + |A1 A2 |;

(5.2)

A proposio segue imediatamente combinando as igualdades (5.1) e (5.2).

5.1

Princpio Aditivo da Contagem

165

Para chegar a uma expresso anloga do princpio aditivo, vamos fazer mais um caso, considerando agora trs conjuntos.

Ento,

Corolrio 5.4. Sejam A1 , A2 e A3 trs conjuntos nitos quaisquer.


|A1 A2 A3 | =|A1 | + |A2 | + |A3 |

|A1 A2 | + |A1 A3 | + |A2 A3 | + |A1 A2 A3 |.

Demonstrao. Pela Proposio 5.3 temos que,


|A1 (A2 A3 )| = |A1 | + |A2 A3 | |A1 (A2 A3 )|,

de onde,
|A1 A2 A3 | = |A1 | + |A2 A3 | |(A1 A2 ) (A1 A3 )|.

Novamente, pela Proposio 5.3 temos que,


|A1 A2 A3 | = |A1 | + |A2 | + |A3 | |A2 A3 | |(A1 A2 ) (A1 A3 )|.

Aplicando mais uma vez a Proposio 5.3 temos que,


|(A1 A2 ) (A1 A3 )| = |A1 A2 | + |A1 A3 | |(A1 A2 ) (A1 A3 ).

Combinando as duas ltimas igualdades obtemos


|A1 A2 A3 | =|A1 | + |A2 | + |A3 |

|A1 A2 | + |A1 A3 | + |A2 A3 | + |A1 A2 A3 | ,

como desejvamos.

166

Contagem

Para facilitar nossa escrita, vamos denotar por A1 A2 . . . Ak o conjunto A1 A2 Ak . Assim, outra forma de enunciar o Corolrio 5.4 a seguinte:
3 3

Ai =
i=1 i=1

|Ai |

1i1 <i2 3

|Ai1 Ai2 | +

1i1 <i2 <i3 3

|Ai1 Ai2 Ai3 |.

De forma geral, dados os conjuntos nitos A1 , A2 , . . . , An , as expresses anteriores nos levam a denir os nmeros:
n

S1 =
i=1

|Ai | |Ai1 Ai2 |,

S2 =

. . .
Sk =

1i1 <i2 n

. . .

1i1 <i2 <<ik n

|Ai1 Ai2 . . . Aik |,

Sn = |A1 A2 . . . An |.

Assim, a verso mais geral do princpio aditivo, tambm conhecida como princpio de incluso e excluso , :

Princpio Aditivo - Verso Geral: Sejam A1 , A2 . . . , An


conjuntos nitos quaisquer. Ento,
n

i=1

Ai = S1 S2 + S3 S4 + + (1)n1 Sn .

No iremos provar essa verso, mas o leitor pode (e deve!) mostr-la como exerccio, repetindo os argumentos anteriores.

5.1

Princpio Aditivo da Contagem

167

tes. Destes, 32 estavam fazendo um curso de francs; 40 um curso de fsica; 30 um curso de matemtica; 23 um curso de histria; 19 francs e fsica; 13 francs e matemtica; 15 fsica e matemtica; 2 francs e histria; 15 fsica e histria; 14 matemtica e histria; 8 francs, fsica e matemtica; 8 francs, fsica e histria; 2 francs, matemtica e histria; 6 fsica, matemtica e histria e 2 estavam fazendo todos os quatro cursos. Quantos estudantes estavam fazendo pelo menos 1 curso nas 4 reas mencionadas? Soluo. Denotemos por A1 , A2 , A3 , e A4 os conjuntos dos estudantes que fazem francs, fsica, matemtica e histria, respectivamente. Observemos que as igualdades
|A1 | = 32, |A2 | = 40, |A3 | = 30, |A4 | = 23,
4

Exemplo 5.5. No Colgio Fantstico foram entrevistados 78 estudan-

nos do que S1 =
i=1

|Ai | = 125; as igualdades |A1 A2 | = 19, |A1 A3 | = 13, |A1 A4 | = 2, |A2 A3 | = 15, |A2 A4 | = 15, |A3 A4 | = 14,

168

Contagem

nos do que S2 =
1i1 <i2 4

|Ai1 Ai2 | = 78; as igualdades |A1 A2 A3 | = 8, |A1 A2 A4 | = 8, |A1 A3 A4 | = 2, |A2 A3 A4 | = 6,

nos do que S3 =
|A1 A2 A3 A4 | = 2. 2 = 69.
1i1 <i2 <i3 4

|Ai1 Ai2 Ai3 | = 24; assim como que S4 =


4

Segue-se ento, do princpio aditivo, que


i=1

Ai = 125 78 + 24

ao conjunto U como sendo um subconjunto de U dado por


Ac = x U ; x /A . U A

Denio 5.6. Denimos o complementar do conjunto A em relao

Figura 5.1: A rea branca corresponde a Ac e o conjunto U representado


por todo o retngulo

5.1

Princpio Aditivo da Contagem

169

Neste caso fcil vericar que os conjuntos A e Ac so disjuntos e que U = A Ac . Segue-se do princpio aditivo que |U| = |A| + |Ac |; portanto, Analogamente, dados dois conjuntos A1 U e A2 U , temos que A1 A2 e (A1 A2 )c so disjuntos e, alis, U = (A1 A2 ) (A1 A2 )c . Novamente, pelo princpio aditivo, vale que
|U| = |A1 A2 | + |(A1 A2 )c |; |Ac | = |U| |A|.

e consequentemente temos que


|(A1 A2 )c | = |U| (|A1 | + |A2 |) + |A1 A2 |.

Similarmente, dados trs conjuntos A1 U , A2 U e A3 U podemos demonstrar que


|(A1 A2 A3 )c | = |U| (|A1 | + |A2 | + |A3 |) |A1 A2 A3 |. + (|A1 A2 | + |A1 A3 | + |A2 A3 |)

Ento, usando a notao S0 = |U|, temos a seguinte proposio:


1, 2, . . . , n, vale a relao:
n c

Proposio 5.7. Para toda famlia de subconjuntos Ai U , i =


Ai
i=1

= S0 S1 S2 + S3 S4 + (1)n1 Sn = S0 S1 + S2 S3 + S4 + (1)n Sn ,
c n

ou resumidamente,
n

Ai
i=1

c |Ac 1 A2

Ac n|

=
j =0

(1)j Sj .

170

Contagem

Observao 5.8. Observemos que na ltima relao da proposio

usamos a conhecida Lei de DeMorgan: o complementar da unio de uma famlia nita de conjuntos, em relao a um conjunto U , a interseco dos complementares de cada um deles.
5.2 Princpio Multiplicativo de Contagem

Comeamos esta seo discutindo um problema relacionado com o apaixonante jogo de xadrez. Ele consiste no seguinte: queremos saber de quantas maneiras diferentes podemos colocar duas torres num tabuleiro de xadrez de forma tal que nenhuma ataque a outra. Uma situao como a que procuramos mostrada na Figura 5.2, pois lembramos que torres s se movimentam na direo horizontal ou na direo vertical do tabuleiro. Antes de prosseguir deixamos claro o seguinte: se na Figura 5.2 trocamos a posio da torre a com a torre b consideraremos isto como uma situao diferente.

Figura 5.2: Torres que no se atacam Notemos o seguinte: uma vez que coloquemos uma das torres numa

5.2

Princpio Multiplicativo de Contagem

171

casa do tabuleiro no podemos colocar a segunda torre na mesma linha ou coluna em que esta se encontra, pois ela seria ameaada. Como cada linha e cada coluna contm 8 casas do tabuleiro, sendo uma delas comum a ambas, ento temos 15 posies proibidas para colocar a segunda torre, ou seja, ela s pode ser colocada em 64 15 = 49 posies diferentes. Resumindo, por cada uma das 64 possveis posies para a torre a temos 49 possibilidades diferentes para colocar a torre b, totalizando 64 49 = 3.136 formas diferentes de colocar ambas as torres no tabuleiro sem que elas se ataquem. O exemplo acima traz a essncia do que chamado princpio multiplicativo da contagem : se um evento A1 pode ocorrer de m maneiras distintas e, se para cada uma dessas m maneiras possveis de A1 ocorrer, um outro evento A2 pode ocorrer de n maneiras distintas, ento o nmero de maneiras de ocorrerem sucessivamente os eventos A1 e A2 m n. Na linguagem matemtica: relembramos que dados dois conjuntos A1 e A2 , podemos construir um par ordenado (a1 , a2 ) tomando um elemento a1 A1 , denominado o primeiro elemento do par, e um elemento a2 A2 , denominado o segundo elemento do par. O conjunto A1 A2 constituido por todos os pares ordenados construdos dessa forma. Assim sendo, a verso mais simples do princpio multiplicativo nos garante que
|A1 A2 | = |A1 | |A2 |.

Uma extenso deste princpio para um nmero nito qualquer de conjuntos a seguinte:

princpio multiplicativo da contagem: Dados os conjuntos

172

Contagem

nitos A1 , A2 , . . . , An temos que


|A1 A2 An | = |A1 | |A2 | |An |.

Note que neste princpio, no necessria nenhuma hiptese adicional sobre os conjuntos Ai . Vamos agora dar alguns exemplos de como aplicar esse princpio.

Exemplo 5.9. Em Macei entraram em cartaz 4 lmes distintos e


2 peas de teatro. Se agora o Pedro Vtor tem dinheiro para assistir

exatamente a um lme e a uma pea de teatro, diga quantos so os possveis programas que Pedro Vtor pode fazer. Soluo. Denotemos por f1 , f2 , f3 e f4 os quatro lmes que esto em cartaz e por t1 e t2 as duas peas de teatro. Denamos os conjuntos
A1 = {f1 , f2 , f3 , f4 } e A2 = {t1 , t2 }.

Neste caso, as condies econmicas do Pedro Vtor permitem que ele escolha um elemento do conjunto A1 e outro elemento do conjunto A2 . Este tipo de escolha representa-se pelo conjunto
A1 A2 = (fi , tj ); 1 i 4 e 1 j 2 ,

onde cada par (fi , tj ) representa o programa que consiste em assistir ao lme fi e pea tj . Logo, no total so |A1 A2 | = |A1 | |A2 | = 8 programas distintos.

balas e 5 tipos distintos de chiclete, diga quantas escolhas podemos fazer para comprar somente uma bala e um chiclete.

Exemplo 5.10. Se numa loja de doces existem 9 tipos distintos de

5.2

Princpio Multiplicativo de Contagem

173

Soluo. Denotemos por b1 , b2 , b3 , b4 , b5 , b6 , b7 , b8 e b9 os nove tipos distintos de balas e por c1 , c2 , c3 , c4 e c5 os cinco tipos distintos de chicletes. Denamos os conjuntos
B = {b1 , b2 , b3 , b4 , b5 , b6 , b7 , b8 , b9 } e C = {c1 , c2 , c3 , c4 , c5 }.

Como precisamos comprar simultaneamente um elemento do conjunto B e um elemento do conjunto C , ento o conjunto B C me d o conjunto de todas as escolhas possveis. Logo, o nmero de escolhas possveis para comprar simultaneamente um tipo de bala e um tipo de chiclete |B C | = 9 5 = 45.

carros numa garagem com 10 vagas?

Exemplo 5.11. De quantas maneiras 2 pessoas podem estacionar seus

Soluo. Observando que a primeira pessoa pode estacionar seu carro de 10 formas distintas e que a segunda pessoa pode estacionar seu carro de 9 formas distintas, temos pelo princpio multiplicativo que existem 9 10 = 90 formas possveis nas quais duas pessoas podem estacionar seus carros numa garagem com 10 vagas.

Exemplo 5.12. Dado o nmero 720, diga


(a) quantos divisores inteiros e positivos ele possui; (b) entre seus divisores inteiros e positivos, quantos so pares; (c) entre seus divisores inteiros e positivos, quantos so mpares; (d) dos divisores acima, quantos so quadrados perfeitos.

174

Contagem

Soluo. Pelo teorema fundamental da aritmtica, todo nmero inteiro positivo primo ou produto de primos. Observe que a decomposio de 720 em fatores primos vem dada por:
720 = 24 32 51 .

(5.3)

Agora denamos os seguintes conjuntos:


B ={todos os divisores de 720 que so da forma 3m , onde m Z+ }, C ={todos os divisores de 720 que so da forma 5n , onde n Z+ }. A ={todos os divisores de 720 que so da forma 2k , onde k Z+ },

Observemos que 0 k 4, pois se k > 4 ento pelo menos a potncia 25 deveria estar presente em (5.3); como isto no acontece segue-se que 0 k 4, de modo que
A = 20 , 21 , 22 , 23 , 24 ,

seguindo o mesmo raciocnio, podemos demonstrar que 0 m 2 e que 0 n 1. Assim,


B = 30 , 31 , 32

e C = 50 , 51 .

(a) O conjunto de todos os possveis divisores de 720 pode ser identicado com o conjunto A B C . De onde o nmero de divisores inteiros e positivos de 720 |A B C |. Porm, o princpio multiplicativo nos garante que |A B C | = |A||B ||C |. Portanto, o nmero de divisores inteiros e positivos de 720 5 3 2 = 30, pois |A| = 5, |B | = 3 e |C | = 2. (b) Para obter o conjunto de todos os divisores pares de 720 devemos remover o elemento 20 do conjunto A. Assim, o conjunto de

5.2

Princpio Multiplicativo de Contagem

175

todos os divisores pares e positivos de 720 vem dado pelo conjunto A {20 } B C . O princpio multiplicativo nos garante que A {20 } B C = A {20 } |B | |C |. Portanto, o nmero de divisores pares e positivos de 720 4 3 2 = 24, pois A {20 } = 4, |B | = 3 e |C | = 2. (c) Para obter o conjunto de todos os divisores mpares de 720 devemos remover os elementos 21 , 22 , 23 e 24 do conjunto A. Assim, o conjunto de todos os divisores mpares e positivos de 720 vem dado pelo conjunto
A {21 , 22 , 23 , 24 } B C.

O princpio multiplicativo nos garante que


A {21 , 22 , 23 , 24 } B C = A {21 , 22 , 23 , 24 } |B | |C |.

Portanto, o nmero de divisores mpares e positivos de 720 1 3 2 = 6; pois A {21 , 22 , 23 , 24 } = 1, |B | = 3 e |C | = 2. (d) Para obter o conjunto de todos os divisores de 720 que so quadrados perfeitos devemos car com as potncias pares nos conjuntos A, B e C , respectivamente. Portanto, devemos remover os elementos 21 , 23 do conjunto A. Tambm devemos remover o elemento 31 do conjunto B . Finalmente do conjunto C devemos remover o elemento 51 . Logo, o conjunto de todos os divisores quadrados perfeitos e positivos de 720 vem dado pelo conjunto
D := A {21 , 23 } B {31 } C {51 } .

O princpio multiplicativo nos garante que


D = A {21 , 23 } B {31 } C {51 } .

176

Contagem

Portanto, o nmero de divisores quadrados perfeitos e positivos de 720 3 2 1 = 6; pois A {21 , 23 } = 3, B {31 } = 2 e C {31 } = 1. Observe que {1, 4, 9, 16, 36, 144} o conjunto dos divisores de 720 que so quadrados perfeitos.

Exemplo 5.13. Se um nmero natural n se fatora como


k2 kr 1 n = pk 1 p2 pr ,

(5.4)

onde os pi so nmeros primos distintos e cada ki Z+ , ento o nmero de divisores positivos de n, denotado por d(n)
d(n) = (k1 + 1)(k2 + 1) . . . (kr + 1).

Soluo. Dena o conjunto


+ 1 A1 ={todos os divisores de n que so da forma pm 1 , onde m Z },

e em geral, dena
+ i Ai ={ todos os divisores de n que so da forma pm i , onde t Z }.

Observemos que mi pi , pois se mi > pi , ento pelo menos a potncia i +1 pk deveria estar presente em (5.4);como isto no acontece segue-se i que mi pi , de modo que
ki 1 2 Ai = p0 , para i = 1, 2, 3, . . . , ki . i , pi , pi , . . . , pi

imediato ver que Ai = ki + 1.

5.2

Princpio Multiplicativo de Contagem

177

O conjunto de todos os possveis divisores de n vem dado pelo conjunto A1 A2 Ar , de onde se conclui que o nmero de divisores inteiros e positivos de n onde na ltima igualdade usamos o princpio multiplicativo. Portanto, o nmero de divisores inteiros e positivos de n
d(n) = (k1 + 1)(k2 + 1) (kr + 1). d(n) = |A1 A2 Ar | = |A1 | |A2 | |Ar |,

Exemplo 5.14. De quantas maneiras podemos escolher dois inteiros


de 1 a 20 de forma que a soma seja mpar? Soluo. Observemos que
a soma de dois nmeros inteiros pares um nmero par. Com efeito, para quaisquer a, b Z temos que 2a + 2b = 2(a + b); a soma de dois nmeros inteiros mpares um nmero par. Com efeito, para quaisquer a, b Z temos que (2a + 1) + (2b + 1) = 2(a + b + 1); a soma de um nmero inteiro par com qualquer outro inteiro

Isto nos sugere denir os conjuntos

mpar sempre um inteiro mpar. Com efeito, para quaisquer a, b Z temos que 2a + (2b + 1) = 2(a + b) + 1.
P = {2, 4, 6, 8, 10, 12, 14, 16, 18, 20},

onde P I so todas as formas possveis de somar um nmero inteiro par com outro mpar. O princpio multiplicativo nos garante que nossa resposta |P I | = |P | |I | = 100, pois |P | = |I | = 10.

I = {1, 3, 5, 7, 9, 11, 13, 15, 17, 19},

178

Contagem

5.3

Uso Simultneo dos Princpios Aditivo e Multiplicativo

Aproveitamos esta seo para apresentar problemas um pouco mais difceis que os tratados nas sees anteriores. Nestes problemas, precisaremos empregar simultaneamente o Princpio Aditivo e o princpio multiplicativo. Vamos ao primeiro deles:

Olimpada Alagoana de Matemtica existem 10 livros diferentes de lgebra, 7 livros diferentes de combinatria e 5 livros diferentes de geometria para homenagear os vencedores. Danielle a primeira a pegar o prmio que consiste em 2 livros, com a condio de que estes no podem ser da mesma matria. Diga quantas escolhas Danielle pode fazer para pegar seu prmio. Soluo. Denotemos por
A = {a1 , . . . , a10 }, C = {c1 , . . . , c7 } e G = {g1 , . . . , g5 },

Exemplo 5.15. Sabemos que no incio da premiao da 1a fase da

os conjuntos de livros de lgebra, combinatria e geometria, respectivamente. Observemos que |A| = 10, |C | = 7 e |G| = 5 e Danielle tem as seguintes possibilidades de escolha:
escolher um livro de A e um livro de C . Neste caso, Danielle tem |A C | = |A| |C | = 70 escolhas possveis (devido ao princpio

multiplicativo).

escolher um livro de A e um livro de G . Neste caso, Danielle tem |A G| = |A| |G| = 50 escolhas possveis (devido ao princpio

multiplicativo) ou

5.3

Uso Simultneo dos Princpios Aditivo e Multiplicativo

179

escolher um livro de C e um livro de G . Neste caso, Danielle tem |C G| = |C | |G| = 35 escolhas possveis (devido ao princpio

multiplicativo).

Agora o Princpio Aditivo nos garante que o nmero total de escolhas que Danielle pode fazer 70 + 50 + 35 = 155.

(3 moas e 2 rapazes) e os restantes no possuem parentesco. Diga quantos casamentos so possveis naquela turma (sabendo que irmos no se casam).

Exemplo 5.16. H 18 moas e 12 rapazes, onde 5 deles so irmos

Soluo. Observemos que 15, entre as 18 moas, no tm parentesco nenhum com os 12 rapazes, logo, pelo princpio multiplicativo temos que possvel efetuar 15 12 = 180 casamentos diferentes entre eles. Por outro lado, as 3 moas restantes podem efetuar casamento com 10 dos 12 rapazes, pois 2 deles so seus irmos. Novamente, pelo princpio multiplicativo possvel realizar 3 10 = 30 casamentos diferentes neste caso. Finalmente, o Princpio Aditivo nos d que podem ser realizados um total de 180 + 30 = 210 casamentos. com as letras , e de modo que em cada palavra no falte nenhuma dessas letras? Soluo. Denamos os seguintes conjuntos,
A ={palavras que esto em U e onde no aparece a letra }; A ={palavras que esto em U e onde no aparece a letra }; A ={palavras que esto em U e onde no aparece a letra }. U ={palavras de 5 caracteres s com as letras , e };

Exemplo 5.17. Quantas palavras de 5 caracteres podem ser formadas

180

Contagem

Por exemplo,
a palavra A A ; a palavra A ; a palavra A .

Primeiramente, notemos que cada caracter de U pode ser escolhido de 3 formas distintas. Segue-se ento do Princpio Multiplicativo que existem 35 formas de escrever uma palavra de 5 caracteres usando um alfabeto de 3 letras, isto ,
S0 = |U| = 35 = 243.

Calculemos agora |A |, isto , o nmero de palavras onde no aparece a letra . Para isto, observemos que cada caractere em A pode ser escolhido de 2 formas. Logo, o princpio multiplicativo nos garante que existem 25 palavras em A , ou seja, |A | = 25 . Analogamente, podemos mostrar que |A | = |A | = 25 . Portanto,
S1 = |A | + |A | + |A | = 25 + 25 + 25 = 96.

Prosseguimos com o clculo de |A A |, isto , do nmero de palavras onde no aparecem as letras e ; portanto, cada caractere em A A pode ser escolhido de 1 forma. Logo, o princpio multiplicativo nos garante que existe 15 = 1 palavra em A A , ou seja, |A A | = 1. Similarmente, podemos mostrar que |A A | = |A A | = 1. Portanto,
S2 = |A | + |A | + |A | = 3.

Por m, achamos |A A A |, que nos d o nmero de palavras onde no aparecem as letras , e ; mas cada palavra em A A A tem

5.4

Permutaes Simples

181

que usar pelo menos um dos caracteres proibidos. Logo,


S3 = |A A A | = 0.

Finalmente, observamos que o conjunto das palavras de 5 caracteres que podem ser formadas com as letras , e de modo que em cada palavra no falte nenhuma dessas letras exatamente o conjunto c c Ac A A . Usando a Proposio 5.7, temos:
c c |Ac A A | =S0 S1 + S2 S3

=150.

=243 96 + 3 0

5.4

Permutaes Simples

Denimos o fatorial n! de um inteiro positivo n


n! = n (n 1) (n 2) 2 1

se n > 0 e 0! = 1, por conveno. Observe que o fatorial cresce muito rapidamente quando n cresce. Por exemplo, para os 10 primeiros valores de n 1!=1 2!=2 3!=6 4!=24 5!=120 6!=720 7!=5.040 8!=40.320 9!=362.880 10!=3.628.800

Denio 5.18. Uma permutao simples de n objetos distintos


qualquer agrupamento ordenado desses n objetos. Denotaremos por Pn o nmero de todas as permutaes simples de n objetos dados.

182

Contagem

Por exemplo, todas as permutaes dos 3 elementos do conjunto A = {a1 , a2 , a3 } so:


1 = (a1 , a2 , a3 ), 2 = (a1 , a3 , a2 ), 3 = (a2 , a1 , a3 ), 4 = (a2 , a3 , a1 ), 5 = (a3 , a1 , a2 ), 6 = (a3 , a2 , a1 ).

Proposio 5.19. Seja n 1. O nmero total de permutaes simples de n objetos O = {o1 , o2 , . . . , on } dado por Pn = n! Demonstrao. claro que a frmula vale para n = 1. Vejamos agora que existe a seguinte relao entre Pn e Pn1 para n 2:
Pn = nPn1 .

(5.5)

Para comprovar isto, para cada i denamos Ai como sendo as permutaes dos n 1 objetos {o1 , . . . , oi1 , oi+1 , . . . , on }. Note que |Ai | = Pn1 , para cada i = 1, 2, . . . , n. Assim, para obtermos uma permutao dos n objetos, basta que xemos o objeto inicial oi e tomemos um elemento do conjunto Ai , que uma permutao dos n 1 objetos restantes. Pelo princpio aditivo, temos que:
Pn = |A1 | + |A2 | + + |An | = nPn1 .

Como a equao (5.5) vlida para todo n 2, podemos aplic-la para n 1, obtendo:
Pn1 = (n 1)Pn2 ,

5.4

Permutaes Simples

183

de onde vem que Repetindo este argumento, obtemos que


Pn = n(n 1)Pn2 .

Pn = n(n 1)(n 2) 3 2 1 = n!,

como queramos demonstrar.

Exemplo 5.20. De quantas maneiras podemos formar uma la com


4 pessoas?

Demonstrao. Observe que se enumeramos os lugares da la e enumeramos as pessoas, pa , pb , pc , pd , cada distribuio vai corresponder a uma permutao do conjunto {1, 2, 3, 4}. Por exemplo, a distribuio (pc , pa , pb , pd ) corresponde permutao (3, 1, 2, 4). Assim, o nmero de distribuies na la 4! = 24. formar pares para uma dana?

Exemplo 5.21. De quantas maneiras k moas e k rapazes podem

Soluo. Estando as moas em uma la e os rapazes em outra, podemos enumer-los com nmeros de 1, 2, . . . , k . A uma permutao desses nmeros, digamos (a1 , a2 , . . . , ak ) com ai {1, 2, . . . , k } faremos uma associao da mulher i com o rapaz ai . Por exemplo, a permutao (2, 1, 3, . . . , k ) signica que a moa 1 danar com o rapaz 2, a moa 2 com o rapaz 1, e a moa i com o rapaz i, para i 3. Observe que toda associao de k moas e k rapazes produz uma permutao, de modo que o nmero de associaes possveis das moas com os rapazes igual ao nmero de permutaes dos elementos do conjunto {1, 2, 3, . . . , k }. Pela Proposio 5.19 existem k ! modos diferentes de combinar as moas com os rapazes.

184

5
Arranjos Simples

Contagem

5.5

que 0 < p n. Um arranjo simples de classe p dos n objetos dados uma seleo de p objetos distintos dentre estes que diferem entre si pela ordem de colocao ou pela natureza de cada um, isto , o que importa quem participa ou o lugar que ocupa. Denotaremos por Ap n o nmero de arranjos simples de classe p de n objetos.
Por exemplo, dados os objetos o1 , o2 e o3 todos os arranjos possveis de classe 2 so: A1 = (o1 , o2 ), A2 = (o2 , o1 ), A3 = (o1 , o3 ), A4 = (o3 , o1 ), A5 = (o2 , o3 ) e A6 = (o3 , o2 ).

Denio 5.22. Consideremos n objetos e p um inteiro positivo tal

objetos dados no mais que uma permutao desses n objetos. Logo, Pn = An n = n!.

Observao 5.23. Notemos que um arranjo simples de classe n de n

Proposio 5.24. Seja n 1. O nmero total de arranjos simples


de classe p de n objetos O = {o1 , o2 , . . . , on } dado por Ap n =
n! . (np)!

Demonstrao. Para n = 1 a frmula obviamente vlida. Similarmente ao caso das permutaes, primeiramente provaremos que para n 2 vale a seguinte igualdade:
p1 Ap n = nAn1 .

(5.6)

Agora denimos os conjuntos Ai como sendo os arranjos simples de classe p 1 dos n 1 objetos {o1 , . . . , oi1 , oi+1 , . . . , on }. Note que 1 |Ai | = Ap n1 , para cada i = 1, 2, . . . , n. Assim, para obtermos um arranjo simples de classe p dos n objetos, basta que xemos o objeto inicial oi e tomemos um elemento do conjunto Ai , que uma arranjo

5.5

Arranjos Simples

185

de classe p 1 dos n 1 objetos restantes. Pelo princpio aditivo, temos que:


p1 Ap n = |A1 | + |A2 | + + |An | = nAn1 .

Como nossa equao (5.6) vlida para todo n 2, podemos aplic-la para n 1, obtendo:
1 p2 Ap n1 = (n 1)An2 ,

de onde vem que


p2 Ap n = n(n 1)An2 .

Repetindo este argumento sucessivamente, obtemos que


Ap n = n(n 1)(n 2) (n (p 2))An(p1) = n(n 1)(n 2) (n p + 2)A1 np+1 .
p(p1)

Notemos agora que A1 np+1 = n p + 1; logo, da igualdade anterior segue-se que


Ap n = n(n 1)(n 2) (n p + 2)(n p + 1) =

n(n 1)(n 2) (n p + 2)(n p + 1) (n p) 1 (n p) 1 n! = , (n p)!

como desejvamos. Agora vamos dar alguns exemplos de como aparecem problemas prticos que requerem fazer este tipo de clculo. O primeiro dele tem

186

Contagem

a ver com a formao de palavras diferentes com um conjunto dado de letras. Um anagrama de uma palavra uma permutao de letras dessa palavra para formar outra, a qual pode carecer de signicado. Por exemplo:
um anagrama de amor roma; um anagrama de celia alice; um anagrama de caterina natercia; um anagrama de elvis lives.

formar com um alfabeto de 23 letras, sendo p < 23?

Exemplo 5.25. Quantos anagramas de p letras distintas podemos

Soluo. Como as letras so diferentes, nosso problema consiste em achar todos os arranjos de classe p de 23 objetos dados, que neste caso so as 23 letras do alfabeto. Logo, este nmero
Ak 23 = 23! . (23 k )!

Exemplo 5.26. De quantos modos 2 pessoas podem se sentar em 5


cadeiras que esto em la? Soluo. Este problema equivalente a achar o nmero total de arranjos de classe 2 de 5 objetos, correspondendo as 5 cadeiras aos 5 objetos e as duas pessoas indicando a ordem do arranjo. Logo, este nmero dado por
A2 5 = 5! = 20. 3!

5.5

Arranjos Simples

187

Exemplo 5.27. Considere os dgitos 2, 3, 4, 5, 7 e 9. Supondo que a


repetio de dgitos no seja permitida, responda s seguintes perguntas: (a) Quantos nmeros de trs dgitos podem ser formados? (b) Entre os achados em (a) quantos so pares? (c) Entre os achados em (a) quantos so mpares? (d) Entre os achados em (a) quantos so mltiplos de 5? (e) Entre os achados em (a) quantos so menores do que 400? Soluo. Seja O = {2, 3, 4, 5, 7, 9} nosso conjunto de objetos.
(a) A quantidade de nmeros de trs dgitos que podemos formar sem repetio de algum deles claramente o nmero de arranjos de classe 3 dos 6 dgitos de O, isto ,
A3 6 = 6! = 120. 3!

(b) Sabemos que em todo nmero par o ltimo dgito um mltiplo de 2, isto , ele acaba em 0, 2, 4, 6 ou 8. Ento, em nosso caso as nicas possibilidades so que o nmero termine em 2 ou 4. Supondo que o ltimo dgito seja 2, temos que preencher as duas casas restantes com os dgitos pertencentes ao conjunto O {2}. 5! 2 Assim, existem A2 |O{2}| = A5 = 3! = 20 nmeros dos achados em (a) que nalizam em 2. De forma anloga, existem A2 |O{4}| = 5! 2 A5 = 3! = 20 nmeros dos achados em (a) que nalizam em 4. Logo, entre os nmeros achados em (a) existem 20 + 20 = 40 nmeros pares.

188

Contagem

(c) Todo conjunto de nmeros pode ser dividido em duas classes disjuntas: a classe dos nmeros pares e a classe dos nmeros mpares que pertencem ao mesmo. Segue-se que dentre os nmeros achados em (a) existem 120 40 = 80 nmeros mpares. (d) Todo nmero mltiplo de 5 acaba em 0 ou 5; no nosso caso temos que a nica possibilidade para o ltimo dgito 5. Assim o problema consiste em preencher as duas casas restantes com dgitos do conjunto O {5}. De onde se segue que a quantidade de nmeros mltiplos de 5 existentes em (a) vem dada por 5! 2 A2 |O{5}| = A5 = 3! = 20. (e) Para obter os nmeros menores do que 400 a casa das centenas s poder ser ocupada pelos dgitos 1, 2 ou 3. Como 1 / O, temos que as nicas possibilidades em nosso caso so 2 ou 3. Ento, supondo que o primeiro dgito do nmero seja 2, devemos preencher duas casas restantes com os dgitos pertencentes a 5! 2 O {2}. De forma anloga, existem A2 |O{3}| = A5 = 3! = 20 nmeros dos achados em (a) e que comeam com 3. Logo, dentre os nmeros achados em (a) existem 20 + 20 = 40 menores do que 400.

5.6

Combinaes Simples

O conceito de combinao simples surge naturalmente quando tentamos responder seguinte pergunta: de quantas formas diferentes podemos selecionar p objetos dentro de n objetos dados?

5.6

Combinaes Simples

189

Por exemplo, suponha que queremos enfeitar uma festa de aniversrio com bolas de dois tipos de cores e na loja onde as compraremos existem bolas nas cores azul, verde e vermelha. De quantas formas distintas podemos enfeitar nossa festa? claro que podemos enfeitar a festa de 3 formas diferentes: com bolas em azul e verde; com bolas em azul e vermelho ou com bolas em verde e vermelho. Notemos que, ao contrrio do caso em que trabalhamos com arranjos, quando fazemos uma seleo de duas cores no estamos interessados na ordem em que elas foram escolhidas.

que 0 < p n. Uma combinao simples de classe p dos n objetos dados uma seleo de p objetos distintos entre estes que diferem entre si apenas pela natureza de cada um, isto , o que importa simplesmente quem participa no grupo selecionado. Denotaremos por n o nmero de combinaes simples de classe p de n objetos. p

Denio 5.28. Consideremos n objetos e p um inteiro positivo tal

Proposio 5.29. Seja n 1. O nmero total de combinaes


n p

simples de classe p de n objetos O = {o1 , o2 , . . . , on } dado por


=
n! . p!(np)!

Demonstrao. Veremos a seguir que arranjos simples e combinaes simples de classe p esto estreitamente relacionados. Com efeito, para cada combinao simples formada por p objetos distintos de O podemos gerar todos os arranjos simples de classe p formados por estes p objetos. Basta para isto fazer todas as suas permutaes possveis. Obtm-se assim p ! arranjos simples diferentes com esses p objetos. Resumindo, para cada combinao simples de classe p formada com p objetos diferentes de O podemos fazer p ! arranjos simples diferentes de classe p com estes mesmos objetos; logo, no total, teremos a

190

Contagem

seguinte relao:
p!

n p

= Ap n =

de onde segue-se que


n p =

n! , (n p)!

n! . p!(n p)!

Exemplo 5.30. De quantas formas diferentes podemos construir uma


palavra de tamanho n com i letras a e n i letras b? Soluo. A soluo do problema equivale em escolher a posio das i letras a em questo, uma vez que a posio das (n i) letras b restantes estar determinada. Se enumeramos as posies das letras de 1 a n, uma palavra ser formada ao xarmos a posio das i letras a. Isso exatamente n , j que corresponde ao nmero de grupos i com i elementos (posies com letra a) tomados em um conjunto de n elementos (todas as posies), que diferem somente por sua natureza.

soas em um grupo de duas e outro de trs?

Exemplo 5.31. De quantas formas podemos dividir um grupo 5 pes-

! Soluo. Temos 5 = 25 = 10 formas diferentes de escolher duas 2 !3 ! pessoas do grupo. Por cada uma dessas escolhas o outro grupo de trs pessoas automaticamente determinado; logo, temos 10 possibilidades diferentes de fazer a diviso.

Exemplo 5.32. De quantos modos podemos dividir 6 pessoas em:


(a) Dois grupos de 3 pessoas cada?

5.6

Combinaes Simples

191

(b) Trs grupos de 2 pessoas cada? Soluo. Comeamos por (a). primeira vista, parece que a resposta deve ser n = 3!6!3! = 20, similarmente ao exemplo anterior. Porm, 3 aqui h um problema devido ao fato de estarmos dividindo em grupos que tm a mesma quantidade de pessoas e, portanto, as permutaes de cada dois grupos formados so consideradas divises iguais; logo, devemos dividir o resultado por 2 !, obtendo assim 10 formas diferentes de obter dois grupos com 3 pessoas cada. Para resolver o item (b) seguimos os seguintes passos:
Primeiramente calcularemos o nmero de formas possveis para

dividir 6 pessoas em um grupo de 2 e outro grupo de 4; esta quantidade vem dada por 6 = 4!6!2! . 2

Agora dividiremos as 4 pessoas restantes em um grupo de 2 e outro grupo de 2; esta quantidade vem dada por 4 = 2!4!2! . 2
4 6! Pelo princpio multiplicativo temos que existem 6 = (2!) 3 possi2 2 bilidades de dividir 6 pessoas em 3 grupos com duas pessoas cada. Igualmente ao caso anterior, aqui as permutaes possveis de cada 3 grupos formados so consideradas iguais; logo, devemos dividir este ltimo resultado por 3 !. Portanto, existem 15 formas diferentes de dividir 6 pessoas em trs grupos de 2 pessoas cada.

pode escolher dois ou mais deles para jantar?

Exemplo 5.33. Se voc possui 10 amigos, de quantas maneiras voc

Soluo. Esquematizamos a soluo da seguinte maneira:


Primeiramente, vamos encontrar a quantidade de maneiras pelas

quais voc pode jantar com 2 amigos; isto feito de diferentes.

10 2

formas

192

Contagem

Depois, vamos encontrar a quantidade de maneiras pelas quais

voc pode jantar com 3 amigos; isto feito de rentes.

10 3

formas dife-

Em seguida, encontramos a quantidade de maneiras pelas quais

voc pode jantar com 4 amigos; isto feito de rentes.

10 4

formas dife-

Em geral, o nmero de maneiras diferentes que voc tem de jantar com p amigos dado por 10 . p

Pelo princpio aditivo, temos que a quantidade de formas diferentes que voc tem de jantar com 2 ou mais de seus amigos, dada por
10 10 10 10 + + + + 2 3 9 10 = 1013,

sendo este o nmero procurado.

quantas comisses de 5 pessoas podem ser formadas de modo que pelo menos uma mulher faa parte?

Exemplo 5.34. De um grupo de 10 pessoas das quais 4 so mulheres,

Soluo. Sendo que o grupo tem 10 pessoas e 4 destas so mulheres, segue-se que no grupo temos 6 homens. Para formar um grupo de 5 pessoas com pelo menos uma mulher, temos as seguintes alternativas:
Nosso grupo composto por uma mulher e 4 homens; neste caso 6 poderemos formar 4 = 60 comisses de 5 pessoas. 1 4 Nosso grupo composto por 2 mulheres e 3 homens; neste caso 6 poderemos formar 4 = 120 comisses de 5 pessoas. 2 3

5.7

O Binmio de Newton

193

Nosso grupo composto por 3 mulheres e 2 homens; neste caso 6 poderemos formar 4 = 60 comisses de 5 pessoas. 3 2 Nosso grupo composto por 4 mulheres e um homem; neste caso 6 poderemos formar 4 = 6 comisses de 5 pessoas. 4 1

Pelo princpio aditivo temos que possvel formar 246 comisses de 5 pessoas de modo que pelo menos uma mulher faa parte.

5.7

O Binmio de Newton

Nesta seo, estudaremos uma frmula que generaliza a conhecida expresso


(a + b)2 = a2 + 2ab + b2 .

Essa frmula conhecida como o binmio de Newton ou frmula binomial de Newton, devido ao Matemtico Isaac Newton (1642-1727). A frmula binomial de Newton pode ser motivada pelas seguintes igualdades que so fceis de vericar:
1 1 a+ b, 0 1 2 2 2 2 2 (a + b)2 = a2 + 2ab + c2 = a + ab + b, 0 1 2 3 3 3 2 3 3 3 (a + b)3 = a3 + 3a2 b + 3ab2 + c3 = a + a b+ ab2 + b. 0 1 2 3 (a + b)1 = a + b =

Os casos particulares acima podem ser estendidos para qualquer potncia inteira positiva de a + b, ou seja, vale o seguinte resultado:

194

Contagem

Teorema 5.35 (Frmula Binomial de Newton) . Sejam a e b nmeros


reais e n N, ento
Os nmeros miais.
n i

(a+b)n =

n n n n1 1 n ni i n n n a + a b + + a b + + a1 bn1 + b . 0 1 i n1 n , 0 i n, so chamados tambm de coecientes bino-

Demonstrao. Expandimos o binmio no produto de seus n fatores, isto ,


(a + b)n = (a + b)(a + b) (a + b) .
nfatores

(5.7)

Se desenvolveremos o produto destes n fatores iguais acima obtemos uma soma nita de termos da forma a1 a2 an , onde cada aj , 1 j n, toma valor a ou b. Notemos que em cada termo se o nmero b aparece i vezes, ento o nmero a aparecer (n i) vezes, ou seja, quando cada termo for multiplicado dever tomar valor igual a ani bi , para algum 1 i n. Por exemplo, os n termos
abb b = abn , bab b = abn , ..., bbb ba = abn

tm o mesmo valor . Assim, para calcular o coeciente do termo ai bni que aparece na equao (5.7), basta responder seguinte pergunta: de quantos modos podemos formar uma palavra com i letras a e (n i) letras b? A resposta dessa pergunta foi estudada no Exemplo 5.30 e simplesmente n . Logo, a expresso na equao (5.7) i
(a + b)n = n n n n n n1 1 n a + a b + + a1 bn1 + b , 0 1 n1 n

o que prova o teorema.

5.8

Contagem e Probabilidades

195

A frmula binomial de Newton nos d algumas propriedades interessantes dos coecientes binomiais que resumimos na prxima proposio.

Proposio 5.36. Seja n N. As seguintes igualdades so vlidas:


(a) (b)
n 0 n 0

n 1 n 1

+ +

n i

+ +
n i

n n1

n n

= 2n ;
n n

+ + (1)i

+ + (1)n

= 0.

Demonstrao. Para a letra (a), basta tomar a = b = 1 e expanda 2n = (1 + 1)n no Binmio de Newton. Para a letra (b), tome a = 1 e b = 1 e expanda 0 = (1 1)n no binmio de Newton, observando que (1)n igual a 1 se n par, e igual a 1 se n mpar.
5.8 Contagem e Probabilidades

Uma das aplicaes interessantes da contagem de elementos de um conjunto quando desejamos estudar a probabilidade de eventos aleatrios. Por exemplo, se lanarmos um dado de seis faces, temos os seguintes resultados possveis:
= {1, 2, . . . , 6}.

Se desejamos saber qual a chance de que ocorra um nmero primo no lanamento, devemos contar quantos primos aparecem em {1, 2, 3, 4, 5, 6} e dividir por 6. Ou seja, a chance de ocorrer um nmero primo num lanamento de um dado de seis faces 3/6 = 0, 5. Denimos a probabilidade de um subconjunto A como o nmero
p(A) = |A| . ||

196

Contagem

Tambm chamamos o subconjunto de todos os resultados possveis de espao amostral e um subconjunto A de de evento. Por exemplo, podemos calcular a probabilidade de escolhermos um nmero par no conjunto 1, 2, 3, . . . , 15. Neste caso, o conjunto est claro e igual a = {1, 2, 3, . . . , 15}. O conjunto A A = {2, 4, 6, . . . , 14}. Logo,
p(A) = 7 |A| = . || 15

Assim, ca claro que a maior diculdade para calcular a probabilidade de um evento contar quantos elementos pertencem a este evento e quantos elementos pertencem ao espao amostral. A seguir, veremos um exemplo mais elaborado onde aplicamos a noo de arranjo simples.

de 44 pessoas, existam pelo menos duas que fazem aniversrio no mesmo dia do ano.

Exemplo 5.37. Calcular a probabilidade de que escolhendo um grupo

Soluo. Podemos reescrever isso do seguinte modo: num saco existem bolas enumeradas com os nmeros 1, 2, . . . , 365 (correspondentes aos dias do ano). Retiramos a bola b1 e anotamos o nmero que apareceu. Devolvemos a bola ao saco e efetuamos uma nova retirada, anotando novamente o nmero que aparece. Repetindo este processo 44 vezes, obtemos uma lista com 44 nmeros. Assim, a pergunta se transforma em: de quantos modos diferentes podemos escolher 44 bolas enumeradas com os nmeros 1, 2, 3, . . . , 365 com reposio, tal que existam pelo menos duas bolas com o mesmo nmero? A primeira coisa que devemos fazer calcular o espao amostral, de todas as possibilidades possveis de resultado. Como escolhemos 44 bolas enumeradas num saco, cada resultado possvel uma lista

5.9

Exerccios Propostos

197

(n1 , n2 , . . . , n44 ) com 44 nmeros. Observe que, pelo princpio multiplicativo, || = 36544 , pois temos 365 opes para escolher n1 , 365 opes para escolher n2 , etc.

A segunda pergunta trata-se de saber quantos resultados so favorveis, ou seja, quantas so as escolhas tais que existam pelo menos duas bolas com o mesmo nmero. Para isso mais fcil contar quantas escolhas existem tais que os 44 nmeros so diferentes. Neste caso, devemos escolher uma ordenao de 44 nmeros distintos entre 365. Isso corresponde quantidade de arranjos de classe 44 num grupo de 365 elementos. Assim, conclumos que a probabilidade de que este evento ocorra
36544 A44 365 =1 p= 36544
365! (365!44!) . 36544

Obter um valor aproximado para o nmero acima com o computador uma tarefa fcil nos dias atuais. Porm, aproximar expresses envolvendo fatoriais (sem o uso do computador) um fato conhecido h muito tempo pela humanidade, atravs da famosa frmula de Stirling.1 Com a ajuda desta frmula, obtemos que p aproximadamente p = 0.93, como havamos prometido no Captulo 1.

Alm dos exerccios abaixo, recomendamos a leitura de [9]. L, o leitor encontrar material adicional sobre anlise combinatria, bem como uma ampla variedade de problemas.
5.9 Exerccios Propostos

1 Grosseiramente,

1. De quantas maneiras podemos escolher trs nmeros distintos do conjunto I50 = {1, 2, 3, . . . , 49, 50} de modo que sua soma seja
n n

2nn e

a frmula de Stirling diz que o quociente entre n! e est prximo de 1, para valores de n grandes.

198

Contagem

a) um mltiplo de 3? b) um nmero par? 2. Considere o conjunto In = {1, 2, 3, . . . , n1, n}. Diga de quantos modos possvel formar subconjuntos de k elementos nos quais no haja nmeros consecutivos? 3. Considere as letras da palavra PERMUTA. Quantos anagramas de 4 letras podem ser formados, onde: a) no h restries quanto ao nmero de consoantes ou vogais? b) o anagrama comea e termina por vogal? c) a letra R aparece? d) a letra T aparece e o anagrama termina por vogal? 4. Calcular a soma de todos os nmeros de 5 algarismos distintos formados com os algarismos 1, 3, 5, 7 e 9. 5. Quantos nmeros podem ser formados pela multiplicao de alguns ou de todos os nmeros 2, 2, 3, 3, 3, 5, 5, 6, 8, 9, 9? 6. Entre todos os nmeros de sete dgitos, diga quantos possuem exatamente trs dgitos 9 e os quatro dgitos restantes todos diferentes? 7. De quantas maneiras podemos distribuir 22 livros diferentes entre 5 alunos se 2 deles recebem 5 livros cada e os outros 3 recebem 4 livros cada? 8. Quantos so os nmeros naturais de sete dgitos nos quais o dgito 4 gura exatamente 3 vezes e o dgito 8 gura exatamente 2 vezes?

5.9

Exerccios Propostos

199

9. De quantas maneiras uma comisso de 4 pessoas pode ser formada, de um grupo de 6 homens e 6 mulheres, se a mesma composta de um nmero maior de homens do que de mulheres? 10. O comprimento de uma palavra a quantidade de caracteres que ela possui. Encontre a quantidade de palavras de comprimento 5 que podemos formar fazendo uso de 10 caracteres distintos, de forma que no existam trs caracteres consecutivos idnticos em cada palavra. 11. Quantos nmeros inteiros existem entre 1 e 10.000 que no so divisveis por 3, 5 e 7? 12. Quantas so as permutaes da palavra PROPOR nas quais no existem letras consecutivas iguais? 13. De quantos modos 6 casais podem sentar-se ao redor de uma mesa circular de tal forma que marido e mulher no quem juntos? 14. Quantas so as permutaes das letras da palavra BRASIL em que o B ocupa o primeiro lugar, ou o R ocupa o segundo lugar, ou o L o sexto lugar? 15. De quantas formas podemos representar o nmero 15 como soma de vrios nmeros naturais? 16. Quantos quadrados perfeitos existem entre 40.000 e 640.000 que so mltiplos simultaneamente de 3, 4 e 5? 17. Oito amigos vo ao cinema assistir a um lme que custa um real. Quatro deles possuem uma nota de um real e quatro possuem

200

Contagem

uma nota de dois reais. Sabendo-se que o caixa do cinema no possui nenhum dinheiro, como eles podem organizar uma la para pagar o lme permitindo o troco pelo caixa? 18. Se considerarmos todas as conguraes do tabuleiro com duas torres que no se atacam, como no Exemplo 5.2, sem distinguir as torres, quantas conguraes obteremos? 19. Continuando o problema anterior, generalize-o para 3, 4, 5, . . . torres que no se atacam, encontrando tambm o nmero mximo de torres que podem ser colocadas no tabuleiro de modo que duas delas no se ataquem. 20. Tente fazer o problema anterior para cavalos de xadrez. 21. Mostre que em toda sequncia de n2 + 1 inteiros distintos possui uma subsequncia crescente de n + 1 elementos ou uma subsequncia decrescente de n + 1 elementos. 22. Encontre o nmero de zeros que termina o nmero 2010!. 23. O jogo do 7 consiste em lanar dois dados e somar o nmero obtido nas suas faces. Caso a soma seja 7, o jogador A ganha o dois reais do jogador B . Caso a soma no seja 7, o jogador B ganha um real de A. Pergunta-se: quem leva vantagem? 24. A funo de Euler associa a cada nmero natural n o valor (n) igual ao nmero de inteiros positivos menores ou iguais a n relativamente primos com n. Ou seja,
(n) = {1 m n; (m, n) = 1} .

5.9

Exerccios Propostos

201

Usando os princpios estudados, mostre que se n se decompe k em fatores primos como n = p1 1 p2 2 . . . p k , ento
(n) = n 1 1 p1 1 1 p2 ... 1 1 pk .

O leitor pode achar mais informaes sobre a funo de Euler nos livros [11] ou ainda [10].

202

Contagem

6
Induo Matemtica

Imagine uma la com innitos domins, um atrs do outro. Suponha que eles estejam de tal modo distribudos que, uma vez que um domin caia, o seu sucessor na la tambm cai. O que acontece quando derrubamos o primeiro domin? Apesar da simplicidade da pergunta acima ela traz em sua essncia toda a ideia usada no mtodo da induo nita . Muitas descobertas em Matemtica so feitas baseadas na realizao de testes que nos fornecem evidncias empricas. Tais evidncias so estudadas para efetivamente vericarmos se os resultados que elas insinuam so verdadeiros. O mtodo da induo nita constitui uma ferramenta muito til na hora de desvendar a veracidade de resultados provenientes deste tipo de estudo. Esse mtodo uma das grandes armas do matemtico moderno e tem utilidade na soluo de vrios problemas, como iremos ver ao longo deste captulo. 203

204

6
Formulao Matemtica

Induo Matemtica

6.1

No incio do sculo XX, o matemtico Giuseppe Peano (1858-1932) estabeleceu os axiomas necessrios que nos permitem hoje descrever com preciso o conjunto dos nmeros naturais. O ltimo dos seus axiomas diz o seguinte: seja A um subconjunto de N (A N). Se 1 A e se, alm disso, A contm todos os sucessores dos seus elementos, ento A = N. Este axioma conhecido como axioma de induo e serve como base do mtodo de demonstrao por induo, o qual de grande utilidade para estabelecer provas rigorosas em Matemtica. O princpio da boa ordenao dos naturais, enunciado no Captulo 3, e o axioma de induo no so independentes e sem nenhuma conexo. De fato, eles so equivalentes, ou seja, se consideramos o princpio da boa ordenao como sendo um postulado podemos deduzir o axioma de induo e, reciprocamente, se consideramos o axioma de induo como sendo um postulado podemos deduzir o princpio da boa ordenao. No resto do captulo, p(n) representa uma armao em relao ao natural n, podendo esta ser verdadeira ou falsa.

teiro no negativo. Suponhamos que, para cada inteiro n n0 , seja dada uma proposio p(n). Suponha que se pode vericar as seguintes propriedades: (a) p(n0 ) verdadeira; (b) se p(n) verdadeira ento p(n + 1) tambm verdadeira, para todo n n0 .

Teorema 6.1 (Princpio da Induo Finita) . Considere n0 um in-

6.1

Formulao Matemtica

205

Ento, p(n) verdadeira para qualquer n n0 .


A armao (a) chamada de base da induo e a (b) de passo indutivo. O fato de que p(n) verdadeira no item (b) chamado de hiptese da induo.

Demonstrao. Denamos o conjunto


V = {m inteiros no negativos ; m n0 e p(m) verdadeira} .

Notemos que V no vazio, pois a condio (a) nos assegura que n0 V . A prova do teorema equivalente a mostrarmos que
V = {n0 , n0 + 1, n0 + 2, n0 + 3, },

ou equivalentemente, a provarmos que o conjunto


F = {m inteiros no negativos ; m n0 e p(m) falsa}

vazio. Suponhamos que F no vazio. Pelo principio da boa ordenao existe um menor elemento m0 F , onde p(m0 ) falso. Observemos que,
m0 n0 + 1. De fato, m0 n0 , porm a possibilidade m0 = n0

contradiz a condio (a);

m0 1 V . Com efeito, p(m0 1) verdadeira pois, caso contrrio, m0 1 F e, alm disso, m0 1 < m0 , contradizendo isto a minimalidade de m0 .

Finalmente, como p(m0 1) verdadeira, segue da condio (b) que p(m0 ) tambm verdadeira, o que impossvel pela denio de m0 . Portanto, o conjunto F vazio, concluindo-se assim a prova.

206

Induo Matemtica

Para um pouco mais sobre a relao entre os princpios de induo e da boa ordenao, recomendamos o Apndice A da referncia [11].

Observao 6.2. Uma grande vantagem do princpio da induo nita poder provar que uma quantidade innita de armaes so verdadeiras, simplesmente vericando que uma quantidade nita destas armaes so verdadeiras. Deixaremos clara a utilidade deste mtodo resolvendo alguns problemas na prxima seo.
6.2 Aplicaes

Dentro da grande gama de problemas que podem ser abordados aplicando o mtodo de induo podemos distinguir trs importantes grupos:
demonstrao de identidades; demonstrao de desigualdades; demonstrao de problemas de divisibilidade.

A seguir damos vrios exemplos de como aplicar o mtodo em problemas referentes a cada um destes grupos.
6.2.1 Demonstrando Identidades

Comeamos com os seguintes problemas clssicos:

(P1) Determinar uma frmula para a soma dos n primeiros nmeros


pares, isto ,
sp (n) := 2 + 4 + 6 + + 2n.

6.2

Aplicaes

207

(P2) Determinar uma frmula para a soma dos n primeiros nmeros


mpares, isto ,
si (n) := 1 + 3 + 5 + + 2n 1.

Para induzir ambas as frmulas, primeiro fazemos os clculos para vrios valores de n, os quais apresentamos na seguinte tabela.
n 1 sp (n) 2 = 1 2 si (n) 1 = 12

2
6=23 4 = 22

3
12 = 3 4 9 = 32

4
20 = 4 5 16 = 42

30 = 5 6 25 = 52

Os resultados na tabela sugerem que sp (n) = n(n + 1) e que si (n) = n2 . Entretanto, isto no constitui por si s uma prova rigorosa destas frmulas, pois para poder garantir a veracidade delas utilizando a tabela teramos que vericar cada valor de n natural, sendo isto impossvel. Provaremos agora que, de fato, as frmulas induzidas so vlidas usando o mtodo de induo nita.

Exemplo 6.3. Demonstre que para qualquer n N vlida a igualdade:


2 + + 2n = n(n + 1).

Soluo. Denamos a proposio


p(n) : 2 + + 2n = n(n + 1)

e observemos que a mesma vale para n = 1 (base da induo); de fato


p(1) : 2 = 1(1 + 1).

Agora partimos para a prova do passo indutivo:

208

Induo Matemtica

Hiptese: suponhamos que p(k ) verdadeira para um certo k > 1, k N. Tese: devemos mostrar que p(k + 1) tambm verdadeira.

Com efeito, como


2 + + 2k = k (k + 1),

somando 2(k + 1) a ambos os lados desta igualdade, temos que


2 + + 2k + 2(k + 1) = k (k + 1) + 2(k + 1) = (k + 2)(k + 1).

Esta ltima igualdade arma que p(k + 1) tambm verdadeira. O Princpio de Induo nos garante que p(n) verdadeira para qualquer n N.

Exemplo 6.4. Demonstre que para qualquer n N vlida a igualdade:


1 + 3 + 5 + + 2 n 1 = n2 .

Soluo. Aqui denimos a proposio:


p(n) : 1 + 3 + 5 + + 2n 1 = n2

e notamos que a mesma vlida se tomarmos, por exemplo, n = 1. De fato, Agora s resta provar o passo indutivo:
p(1) : 1 = 2 1 1.

Hiptese: suponhamos que p(k ) seja verdadeira para um certo k > 1, k N.

6.2

Aplicaes

209

Tese: devemos mostrar que p(k + 1) tambm verdadeira.

Com efeito, como


1 + 3 + 5 + + 2k 1 = k 2 ,

somando 2k + 1 a ambos os lados desta igualdade, temos que


1 + 3 + 5 + + 2k 1 + 2 k + 1 = k 2 + 2k + 1 = (k + 1)2 .

O princpio de induo nos garante que p(n) verdadeira para qualquer n N. Uma consequncia imediata do Exemplo 6.3 a frmula para a soma dos n primeiros nmeros naturais, dada por
sn = 1 + 2 + 3 + + n = n(n + 1) . 2

(6.1)

Com efeito, como


2 + 4 + + 2n = n(n + 1),

ento dividindo por 2 ambos os membros da igualdade acima, obtemos a equao (6.1). Continuando com o mesmo raciocnio, natural nos perguntarmos se possvel obter uma frmula para a soma dos n primeiros quadrados perfeitos, ou seja, determinar qn onde:
qn = 12 + 22 + 32 + + n2 .

Para induzir a frmula, consideramos os valores de sn e qn numa tabela:

210

Induo Matemtica

n sn qn

1 2 3 4 5 6 1 3 6 10 15 21 1 5 14 30 55 91

Aparentemente no existe nenhuma relao entre sn e qn . Mas, se considerarmos o quociente qn /sn , vejamos o que acontece:
n qn /sn

1
3/3

2
5/3

3
7/3

4
9/3

5
11/3

6
13/3

Isso nos sugere que vale a relao


qn 2n + 1 = , sn 3

logo nosso candidato para valor de qn


qn = sn (2n + 1) n(n + 1)(2n + 1) = . 3 6

Convidamos o leitor a provar a veracidade da equao acima utilizando o Mtodo da Induo no Exerccio 1 no nal do captulo.
6.2.2 Demonstrando Desigualdades

Apresentamos agora alguns exemplos de como usar induo para provar desigualdades.

Exemplo 6.5. Prove que 3n1 < 2n2 para todo n N.


Soluo. Denotamos por p(n) a propriedade: 3n1 < 2n . claro que p(1) vlida, pois 1 < 2. Agora supondo que P (n) verdadeira temos que
2

3n = 3n1 3 < 2n 22n+1 = 2(n+1) ,

logo p(n + 1) tambm vale. Observamos que na desigualdade acima usamos o fato de que 3 < 22n+1 para qualquer n N.

6.2

Aplicaes

211

Exemplo 6.6. Mostre que para todo nmero n N, n > 3, vale que 2n < n!
Demonstrao. Para n = 4 a desigualdade vericada, pois 24 = 16 < 4! = 24. Vamos assumir como hiptese de induo que a desigualdade vlida para n 4. Ento, precisamos mostrar que a mesma vale tambm para n + 1. De fato, por hiptese de induo:
2n < n!

(6.2)

Como 2 < n + 1, podemos multiplicar o lado esquerdo da desigualdade em (6.2) por 2 e o lado direito por n +1, sem alterar o sinal de desigualdade. Logo, temos que:
2n .2 = 2n+1 < n!(n + 1) = (n + 1)!,

concluindo-se a demonstrao.

Exemplo 6.7. Prove que, para todo n N,


2+ 2+ 2 + + 2 < 2.

nradicais

Demonstrao. Claramente a desigualdade vale para n = 1, pois 2 < 2. Suponhamos que para certo n N a desigualdade acontece, ento
2+ 2+ 2 + + 2 < 2.

nradicais

Logo, adicionando 2 em ambos os lados desta desigualdade tem-se


2+ 2+ 2+ 2 + + 2 < 2 + 2.

nradicais

212

Induo Matemtica

Tomando raiz quadrada em ambos os lados desta ltima desigualdade obtemos


2+ 2+ 2+ 2 + + 2 < 2,

n+1radicais

como desejvamos.

6.2.3

Induo e Problemas de Divisibilidade

Agora damos alguns exemplos de problemas de divisibilidade que podem ser mostrados utilizando o mtodo da induo:

Exemplo 6.8. Mostre que para qualquer n N,


por 3.

n3 + 2n sempre divisvel

Soluo. Para n = 1 a armao vlida, pois 13 +2 1 = 3, que obviamente divisvel por 3. Assumamos como hiptese indutiva que a armao vale para algum k N, isto ,
Hiptese: k3 + 2k divisvel por 3. Devemos mostrar que a armao tambm verdadeira para k + 1, ou seja, temos que provar que Tese: (k + 1)3 + 2(k + 1) divisvel por 3. Para provar isto ltimo, usamos o fato de que
(k + 1)3 + 2(k + 1) = (k 3 + 3k 2 + 3k + 1) + (2k + 2);

6.2

Aplicaes

213

agrupando adequadamente,
(k + 1)3 + 2(k + 1) = (k 3 + 2k ) + (3k 2 + 3k + 3) = (k 3 + 2k ) + 3(k 2 + k + 1)
mltiplo de 3 mltiplo de 3

= mltiplo de 3,

concluindo assim a prova.

Exemplo 6.9. Mostre que a soma dos cubos de trs nmeros naturais consecutivos divisvel por 9. Soluo. Denamos a seguinte proposio:
p(n) : n3 + (n + 1)3 + (n + 2)3 um mltiplo de nove.

Notemos que P (1) vlida, pois


13 + 23 + 33 = 1 + 8 + 27 = 36 = 9 4.

Precisamos provar agora o passo indutivo, isto ,


Hiptese: P (k ) verdadeira para algum k N. Tese: P (k + 1) tambm verdadeira.

Para provar isto, observamos que

(k + 1)3 + (k + 2)3 + (k + 3)3 = (k + 1)3 + (k + 2)3 + (k 3 + 9k 2 + 27k + 27).

Ordenando adequadamente, temos que o lado direito da ltima igualdade se escreve como
k 3 + (k + 1)3 + (k + 2)3 + (9k 2 + 27k + 27) = k 3 + (k + 1)3 + (k + 2)3 + 9(k 2 + 3k + 3)
mltiplo de 9 mltiplo de 9

= mltiplo de 9,

completando assim nossa demonstrao.

214

Induo Matemtica

Muitas vezes, para conseguir mostrar que a hiptese p(n + 1) verdadeira, precisamos supor que p(k) verdadeira para todo n0 k n. Isto a base do princpio forte da induo nita que enunciamos a seguir:

Teorema 6.10 (Princpio Forte da Induo Finita). Considere n0 um inteiro no negativo. Suponhamos que, para cada inteiro n n0 seja dada uma proposio p(n) e que valem as propriedades (a) p(n0 ) verdadeira; (b) se para cada inteiro no negativo k, com n0 k n, temos que p(k) verdadeira, ento p(n + 1) tambm verdadeira. Ento, a proposio p(n) verdadeira para qualquer n n0 .
Utilizando o princpio forte da induo, vamos dar uma prova diferente do teorema fundamental da aritmtica da apresentada no Captulo 3.

Exemplo 6.11 (Teorema Fundamental da Aritmtica). Todo nmero natural N maior que 1 pode ser escrito como um produto
N = p1 p2 p3 pm ,

(6.3)

onde m 1 um nmero natural e os pi , 1 i m so nmeros primos. Alm disso, a fatorao em (6.3) nica se exigirmos que p1 p2 pm . Soluo. Para cada n N, n 2, denamos a proposio
p(n) : n escrito de modo nico como um produto de nmeros primos.

Notemos que p(2) verdadeira, pois 2 um nmero primo. Agora enunciemos o passo indutivo:
Hiptese indutiva: p(k ) verdade para cada inteiro k tal que 2 k n.

6.3

Induo na Geometria

215

Tese: p(n + 1) verdade. Em outras palavras, temos que mostrar que n + 1 escrito de modo nico como um produto de nmeros primos.

Faremos a prova dividindo em dois casos: (a) Se n + 1 um nmero primo, ento p(n + 1) verdade e isto acaba nossa demonstrao. (b) Se n + 1 no um nmero primo, ento existem , N com 2 n e 2 n tais que n + 1 = . Nossa hiptese indutiva vlida para e . Isto signica que se escreve de modo nico como um produto de nmeros primos e que se escreve de modo nico um produto de nmeros primos. Portanto, n + 1 = se escreve como um produto de nmeros primos.

Agora mostraremos que n + 1 se escreve de modo nico como produto de primos. Assuma que
p1 p2 . . . pk = q1 q2 . . . qm = n + 1,

(6.4)

com p1 p2 pk e q1 q2 qm todos primos. Vamos mostrar que necessariamente k = m e pi = qi . De fato, como p1 primo, ele divide algum qi . Logo, como qi primo, p1 = qi q1 . Analogamente, existe um j tal que q1 = pj p1 . Logo, p1 = q1 . Cancelando p1 em ambos os lados da equao (6.4), temos que (n + 1)/p1 = p2 . . . pk = q2 . . . qm n. Logo, por hiptese de induo, k = m e p2 = q2 , . . . , pm = qm , encerrando a demonstrao.

6.3

Induo na Geometria

Tratamos aqui alguns exemplos que mostram a utilidade do mtodo de induo na resoluo de problemas geomtricos. Vamos comear estudando

216

Induo Matemtica

duas propriedades importantes dos polgonos. A primeira delas trata da soma dos ngulos internos de um polgono convexo de n lados (n-gono). Um polgono convexo um polgono tal que qualquer segmento de reta que liga dois de seus pontos est contido no interior dele. No caso de polgonos, isto equivalente ao fato de que todo segmento que liga dois vrtices ou uma aresta ou est contido no interior do polgono. convexo de n lados (n 3) igual a (n 2) radianos.

Exemplo 6.12. Mostre que a soma dos ngulos internos de um polgono


Soluo. No caso de n = 3 a propriedade acima muito bem conhecida. Desde Tales de Mileto e Euclides se conhecia que a soma dos ngulos internos de um tringulo radianos. Faamos mais um caso, tomando n = 4. Neste caso, podemos dividir um quadriltero em dois tringulos, como mostra a Figura 6.1 (a). Assim, a soma dos ngulos internos de um quadriltero 2 radianos.
A4 A3 A5 A4 A3

A1

A2

A1

A2

Figura 6.1: Dividindo polgonos


Para elucidar o processo de induo e no deixar dvidas sobre o que iremos fazer, vamos considerar mais um polgono, o pentgono (n = 5). Neste caso, para mostrar que a soma dos seus ngulos internos (5 2) = 3 radianos, iremos dividir o pentgono A1 A2 A3 A4 A5 em um quadriltero A1 A2 A3 A4 e um tringulo A1 A4 A5 , como mostra a Figura 6.1 (b). Assim,

6.3

Induo na Geometria

217

a soma dos ngulos internos do pentgono A1 A2 A3 A4 A5 igual soma dos ngulos internos do tringulo A1 A4 A5 (igual a ) mais a soma dos ngulos internos do quadriltero A1 A2 A3 A4 (igual a 2 ), ou seja, igual a 3 . Finalmente, vamos assumir como hiptese de induo que para um certo n 3 mostramos que a soma dos ngulos internos do n-gono dada pela expresso (n 2) . Precisamos mostrar que a soma dos ngulos internos de um n + 1-gono [(n + 1) 2] = (n 1) . De fato, podemos repetir o processo anterior. Vamos denominar de A1 , A2 , . . . , An , An+1 os vrtices consecutivos do (n + 1)-gono. Podemos dividi-lo no n-gono A1 A2 . . . An e no tringulo A1 An+1 An . Logo, a soma dos ngulos internos do (n +1)-gono (n 2) + = (n 1) .

de n-lados igual a

Exemplo 6.13. Mostre que o nmero de diagonais de um polgono convexo


n(n3)

Soluo. Observe que para n = 3 temos que existem 0 = 3.(3 3)/2 diagonais num tringulo. Para n = 4, temos 2 = 4(4 3)/2 diagonais num quadriltero convexo (veja a Figura 6.2). Vamos agora assumir como hiptese de induo que se n um ngono convexo ento o seu nmero de diagonais n(n 3)/2 e vamos provar que a frmula vale para um (n + 1)-gono convexo. De fato, denote por A1 , A2 , . . . , An , An+1 os vrtices consecutivos do n + 1-gono. Podemos decomp-lo como a unio do n-gono A1 , A2 , . . . , An e do tringulo A1 , An , An+1 . Neste caso, para contarmos as diagonais do (n + 1)-gono devemos considerar os seguintes casos:
Diagonais do n-gono A1 , A2 , . . . , An ; por hiptese de induo, o nmero dessas diagonais n(n 3)/2.

Assim, o nmero total de diagonais do (n + 1)-gono

n 2 diagonais que partem do vrtice An+1 mais a diagonal A1 An .

n2 3n + 2n 2 n2 n 2 (n + 1)(n 2) n(n 3) + (n 2) + 1 = = = , 2 2 2 2

218

Induo Matemtica

como queramos demonstrar.


A4 A3 A5 A4 A3

A1

A2

A1

A2

Figura 6.2: Diagonais de polgonos

seguir traando 2n 2 segmentos de reta sem tirar o lpis do papel.

Exemplo 6.14. Mostre que podemos cobrir os

n2 pontos no reticulado a


nn

Figura 6.3: O problema de bar n n

6.3

Induo na Geometria

219

Soluo. O caso n = 3 j foi enunciado no Problema 1.12 do Captulo 1. A gura a seguir mostra a soluo, onde o caminho realizado com as 4 linhas o seguinte: A B C D B .
C A D B

Figura 6.4: Soluo do problema de bar 3 3


Daremos a prova do problema acima por induo. Para isso, veja que podemos resolver o caso n = 4 continuando a soluo do caso n = 3. Como paramos num dos vrtices do quadrado 3 3, acrescentamos mais uma linha e uma coluna para obter um reticulado 4 4. Assim, conseguimos cobrir os 16 pontos utilizando 4 + 2 = 6 linhas, sem tirar o lpis do papel e cobrindo dois lados do quadrado, como mostram as linhas descontnuas na Figura 6.5.
A D C B

Figura 6.5: Completando o reticulado

220

Induo Matemtica

Finalmente, vamos assumir como hiptese de induo que podemos cobrir n 2 um reticulado n n com 2n 2 linhas, sendo que a ltima delas cobre um dos lados do reticulado. Acrescentando 2n +1 pontos como mostra a Figura 6.5, obtemos um reticulado (n + 1) (n + 1) que pode ser coberto com 2n 2 + 2 = 2(n + 1) 2 pontos, como queramos demonstrar.
6.4 Miscelnea

Nesta seo discutiremos alguns exemplos interessantes de como podemos aplicar o mtodo da induo aos mais variados tipos de problemas. O primeiro deles uma generalizao do Problema 1.8.

Exemplo 6.15 (A Moeda Falsa). Um rei muito rico possui 3n moedas de

ouro. Porm, uma destas moedas falsa e seu peso menor que o peso das demais. Com uma balana de 2 pratos e sem nenhum peso, mostre que possvel encontrar a moeda falsa com apenas n pesagens.

Soluo. Para resolver este problema, vamos utilizar o Mtodo da Induo. De fato, se n = 1, procederemos da seguinte forma: pegamos duas moedas quaisquer e colocamos na balana, deixando uma do lado de fora. Caso a balana se equilibre, a moeda que est do lado de fora necessariamente a que tem menor peso. Caso a balana se desequilibre, a que tem menor peso est na balana, no prato mais alto. O caso n = 2 foi feito no Problema 1.8. Vamos agora assumir como hiptese de induo que dadas 3n moedas, podemos achar a moeda mais leve com n pesagens. Vamos mostrar que para 3n+1 moedas, suciente n + 1 pesagens. De fato, dividiremos as 3n+1 moedas em 3 grupos, A, B e C com 3n moedas cada. Colocamos na balana os grupos A e B . Caso os dois grupos se equilibrem, a moeda mais leve est no grupo C . Caso o grupo A esteja mais leve, a moeda mais leve se encontra no grupo A. De qualquer modo, com uma pesagem conseguimos

6.4

Miscelnea

221

determinar em qual grupo de 3n elementos a moeda mais leve se encontra. Por hiptese de induo, precisamos de mais n pesagens para encontrar a moeda mais leve, totalizando n + 1 pesagens. Desaamos o leitor a mostrar que no possvel realizar tal tarefa com menos de n pesagens.

Exemplo 6.16. Mostre que utilizando um balde com 5 litros de capacidade


e outro com 7 litros, possvel separar qualquer quantidade superior ou igual a 24 litros.

Soluo. Novamente, faremos a prova utilizando o Mtodo da Induo. Neste caso, comearemos o processo de induo a partir de 24. De fato, podemos separar 24 litros utilizando duas vezes o balde de 7 e duas vezes o balde de 5 litros. Note que o problema acima equivale a mostrar que Todo nmero maior ou igual a 24 pode ser escrito da forma 7x + 5y , onde x e y so nmeros inteiros maiores ou iguais a zero.
Neste caso, escrevemos 24 como 24 = 2 7 + 2 5. Por hiptese de induo, vamos supor que conseguimos escrever um nmero n 24 como n = 7x + 5y , com x e y nmeros inteiros maiores ou iguais a zero. Devemos mostrar que n + 1 se escreve deste modo tambm. Para isso, vamos dividir a anlise em dois casos:

Caso 1:

Logo, x 2 pois se isso no ocorresse, teramos 7x + 5y 22 < 24, o que impossvel. Assim, podemos escrever:
n + 1 = 7x + 5y + 1 = 7(x 2) + 5(y + 3),

y3

pois x 2 0.

Caso 2:

y4

222

Induo Matemtica

Neste caso, y 4 0. Logo, podemos escrever:


n + 1 = 7x + 5y + 1 = 7(x + 3) + 5(y 4),

nalizando a nossa prova por induo.


6.4.1 Cuidados ao Usar o Princpio da Induo

Observao 6.17. Quando aplicamos o princpio da induo devemos tomar certos cuidados. A seguir damos um exemplo de como o mtodo pode ser aplicado de forma errada. Vamos mostrar a seguinte armao: possuem a mesma cor.

Armao: Num conjunto qualquer de n bolas, todas as bolas

Observe que nossa proposio claramente falsa. Mas, mesmo assim, vamos dar uma prova por induo. Para n = 1, nossa proposio verdadeira pois em qualquer conjunto com uma bola, todas as bolas tm a mesma cor, pois s existe uma bola. Assuma por hiptese de induo que a proposio verdadeira para n e provemos que a proposio verdadeira para n+1. Ora, seja A = {b1 , . . . , bn , bn+1 } o conjunto com n + 1 bolas referido. Considere os subconjuntos de B e C de A com n elementos, construdos como:
B = {b1 , b2 , . . . , bn } e C = {b2 , . . . , bn+1 }

Observe que ambos os conjuntos tm n elementos. Assim, as bolas b1 , b2 , . . . , bn do conjunto B tm a mesma cor. Do mesmo modo, as bolas do conjunto C tm a mesma cor. Em particular, a bola bn tem a mesma cor da bola bn+1 . Assim, todas as bolas tm a mesma cor. Ache o erro no argumento! Se voc no conseguir, leia a nota de rodap. 1
1 Uma

dica da soluo encontra-se no nal do captulo.

6.5
6.5

Induo e Recorrncias

223

Induo e Recorrncias

Vamos comear esta seo discutindo um problema muito conhecido e interessante.

dos tempos, em um templo de Hani, foram colocados 64 discos perfurados de ouro puro e de dimetros diferentes ao redor de uma de trs hastes de diamante. Muitos sacerdotes moviam os discos, respeitando as seguintes regras: eles comeam empilhados em ordem crescente de acordo com seu tamanho (ver Figura 6.6). Os discos podem ser deslocados de uma coluna para qualquer outra, sendo que nunca pode ser colocado um disco maior em cima de um menor e a cada segundo os sacerdotes movem um disco. Quando os sacerdotes transportassem todos os discos de uma coluna para outra, o mundo se acabaria. Suponha que eles comearam esse processo no ano 2000 e que a lenda verdadeira, quanto tempo ainda resta para a Terra?

Exemplo 6.18 (As Torres de Hani2 ). Diz uma antiga lenda que na origem

Figura 6.6: Torre de Hani


Para responder esse problema, consideraremos o problema geral de descobrir quantos movimentos so necessrios para mover n anis de uma haste para outra. Argumentaremos do seguinte modo: observe que podemos mover os discos para outra haste se n = 1 ou 2. Com efeito, se temos somente um anel basta mover este para qualquer outra haste com um nico movimento.
2 Este

jogo foi inventado, em 1882, pelo matemtico Francs douard Lucas.

224

Induo Matemtica

Se temos 2 anis ento movemos o menor deles para a segunda haste, o maior para a terceira haste e, nalmente, o menor para a terceira haste, realizando um total de 3 movimentos. Para calcular o caso geral, vamos empregar um mtodo chamado de mtodo recursivo: o nmero ak+1 de movimentos necessrios para mover k + 1 anis ser expresso como uma funo de ak . De fato, se temos k + 1 anis na primeira haste e sabemos mover k anis de uma haste para outra utilizando ak movimentos, ento podemos mover todos os k + 1 anis para a segunda haste usando 2ak + 1 movimentos. De fato, movemos todos eles, exceto o maior, para a terceira haste usando ak movimentos. A seguir, colocamos o maior na segunda haste usando 1 movimento. Imediatamente, deslocamos todos os anis da terceira haste para a segunda haste usando mais ak movimentos. Logo, movemos todos os k + 1 anis utilizando 2ak + 1 movimentos. Em resumo:
ak+1 = 2ak + 1,

(6.5)

onde ak o nmero de movimentos necessrios para mover k discos de uma haste para outra. Vamos agora usar induo para provar que ak = 2k 1. Uma vez constatada a veracidade da armao para k = 1, 2, para calcular ak , por hiptese de induo, vamos assumir que ak = 2k 1. Temos pela equao (6.5):
ak+1 = 2ak + 1 = 2(2k 1) 1 = 2k+1 1.

como queramos demonstrar.


Vamos aproveitar o Exemplo 6.18 para discutir algumas equaes que aparecem em muitas situaes em Matemtica: as equaes de recorrncia. Em geral, uma equao de recorrncia uma equao envolvendo uma certa quantidade de termos de sequncia xn . Para ilustrar isso, observe a equao (6.5). Aqui, estaremos interessados em um tipo particular de equao de recorrncia, as equaes de recorrncia lineares.

6.5

Induo e Recorrncias

225

Denio 6.19. Uma equao de recorrncia linear de grau


presso da forma:
xn+1 =rk1 xn + rk2 xn1 + + r0 xnk+1 x1 = a1 , x2 = a2 , . . . , xk = ak ,

k uma ex-

(6.6)

onde r0 , r1 , . . . , rk1 so nmeros reais e r0 = 0.


Por exemplo, so equaes de recorrncia lineares as seguintes equaes
2xn 3xn+1 = 0

2 3xn + xn+1 = 5xn+2 3

e no so equaes de recorrncia lineares as equaes


2(xn )3 5xn+1 = 0

2 3xn + xn+1 = 5xn+2 + 3. 3

Exemplo 6.20 (Sequncia de Fibonacci). Um exemplo muito interessante

de equao de recorrncia a sequncia conhecida por sequncia de Fibonacci, devido ao matemtico italiano Leonardo di Pisa (1170-1250). Esta sequncia adquiriu muita fama devido a suas conexes com reas das mais variadas na cultura humana. Ela aparece em problemas de Biologia, Arquitetura, Engenharia, Fsica, Qumica e muitos outras reas da cincia e arte. Denimos a sequncia de Fibonacci como sendo a sequncia Fn que satisfaz a seguinte equao de recorrncia:
F1 = 1; F2 = 1; Fn = Fn1 + Fn2 , se n 3.

Agora vamos utilizar induo para mostrar algumas de suas propriedades.

226

Induo Matemtica

Exemplo 6.21. Considere Fn a sequncia de Fibonacci. Mostre que


Fn < 7 4
n

.
n

7 Soluo. Denamos a proposio p(n) := Fn < 4 . Para n = 1 temos 7 que F1 = 1 < 4 , de modo que p(1) verdadeira. Suponhamos que

p(1), p(2), . . . , p(n), n 2,

sejam todas verdadeiras. Mostraremos que Fn+1 <


Fn+1 = Fn + Fn1 < <
7 4 7 n 4

7 n+1 . 4

Com efeito,

7 n1 4 7 n1 4 7 n1 . 4

7 n1 4 7 4

< 1+

Como 1 +

7 4

<

7 2 4 ,

segue-se que Fn+1 <


Fn+1 <
7 n+1 . 4

7 2 4

7 n1 . 4

Portanto,

Exemplo 6.22. Dada a seguinte relao de recorrncia


a0 = 8; a1 = 10; an = 4an1 3an2 , n 2.

Mostre que an = 7 + 3n , para todo n Z+ . Soluo. Denamos a proposio P (n) : an = 7 + 3n . P (0) verdadeira, pois P (0) = 7 + 30 = 7 + 1 = 8. Suponhamos que P (k) verdadeiro para cada inteiro k tal que 1 k n. Vamos mostrar que P (k) verdade para

6.5

Induo e Recorrncias

227

k = n + 1. Com efeito, an+1 = 4an 3an1 = 4(7 + 3n ) 3(7 + 3n1 ) = 7 + 4 3n 3 3n1 = 7 + 3n1 4 3 3 = 7 + 3n+1 .

= 7 + 3n1 9 = 7 + 3n1 32

Vamos agora discutir o caso geral da equao de recorrncia linear (6.6). Para isso, vamos fazer algumas observaes preliminares que deixaremos a cargo do leitor:
se an e bn so solues da equao (6.6), ento an + bn tambm

soluo;

se an soluo da equao (6.6) e um nmero real, ento an

tambm soluo.

Com isto em mente, vamos descrever agora como obter todas as solues xn da equao (6.6) em funo de n. Observe que dados os termos iniciais a1 , a2 , . . . , ak a sequencia xn ca inteiramente determinada pela equao de recorrncia. O interessante aqui determinar o termo xn+1 sem que seja preciso o clculo dos termos xn , xn1 , . . . , xnk+1 . Vamos primeiro procurar o que se chama de soluo particular da equao (6.6). Particular porque ela assume uma forma caracterstica e porque no assumiremos que as condies x1 = a1 , . . . , xk = ak valham. Vamos procurar solues do tipo xn = n , onde um nmero real positivo. Neste caso, temos que:
n+1 = xn+1 =rk1 xn + rk2 xn1 + + r0 xnk+1 = rk1 n + rk2 n1 + + r0 nk+1 .

228

Induo Matemtica

Passando os termos do lado direito da igualdade e colocando em evidncia o termo nk+1 temos:

nk+1 k rk1 k1 rk2 k2 r0 = 0.

(6.7)

Assim, como k = 0, pois > 0, temos que


k rk1 k1 rk2 k2 r0 = 0.

(6.8)

O polinmio
p() = k rk1 k1 rk2 k2 r0

recebe o nome especial de polinmio caracterstico da equao de recorrncia (6.6). Acabamos de mostrar que qualquer raiz do polinmio caracterstico gera uma soluo particular da equao (6.6). Vamos assumir que a equao (6.8) possui k razes diferentes, digamos 1 > 2 > > k . Ento vale o seguinte teorema:

Teorema 6.23. Se escolhermos nmeros reais c1 , c2 , . . . , ck , ento


n n xn = c1 n 1 + c2 2 + + ck k

(6.9)

uma soluo da equao de recorrncia, onde os termos iniciais ai para i = 1, 2, . . . , k so:


i i ai = c1 i 1 + c2 2 + + ck k .

Demonstrao. Para mostrar o teorema, como x1 = a1 , . . . xk = ak pela denio dos ai 's, basta mostrar que xn uma soluo. Ora, o produto de uma soluo por um nmero real tambm uma soluo. Assim, como n i uma soluo para i = 1, 2, . . . , k e ci um

6.6

Exerccios

229

nmero real, temos que ci n i soluo para i = 1, 2, . . . , k . Como j vimos acima, a soma de solues tambm uma soluo. Logo,
n n xn = c1 n 1 + c2 2 + + ck k

uma soluo. Neste ponto, voltamos a equao (6.6). Desde o princpio, dados os nmeros ai buscvamos a soluo xn tal que x1 = a1 , . . . , xk = ak . A Equao (6.9) nos d uma variedade de solues, onde podemos escolher os nmeros ci como bem entendermos. Usando equaes lineares, podemos mostrar que sempre possvel escolher os nmeros ci de modo que x1 = a1 , . . . , xk = ak . Isso encerra nossa busca. Para complementar esta seo, recomendamos a leitura do Captulo 3 de [4].

6.6

Exerccios

1. Se qn denota a soma qn = 12 + 22 + + n2 , prove que para todo


nN qn = n(n + 1)(2n + 1) . 6

2. Use o princpio da induo para provar as seguintes armaes: (a) 3n+1 + 2n+2 divisvel por 7 para todo n N; (b) a soma dos cubos de trs nmeros naturais consecutivos divisvel por 9; (c) 7 + 77 + 777 + + 777 . . . 7 = 7(10n+1 9n 10)/81; (d) (n + 1)(n + 2) . . . (n + n) = 2n 1 3 5 (2n 1). 3. Use o princpio da induo para provar as seguintes desigualdades:
nvezes

230

Induo Matemtica

(a) 2n1 (an + bn ) > (a + b)n , a = b;

n N, com a, b R, a + b > 0 e

(b) + + + + >

1 1 1 1 n, para todo n N; n 1 2 3 1 1 1 1 13 (c) + + + + > , para todo n N. n+1 n+2 n+3 2n 24 (n + 1) cos nx n cos(n + 1)x 1 . 4 sin2 x 2

4. Mostre a seguinte identidade trigonomtrica


cos x + 2 cos 2x + + n cos nx =

5. Um torneio de xadrez tem n jogadores. Cada jogador joga uma nica partida com cada um dos outros jogadores. Calcule o nmero total de partidas realizadas no torneio. 6. Demonstre que para qualquer n N vlida a igualdade
13 + 2 3 + 3 3 + + n3 = n(n + 1) 2
2

7. Demonstre que para qualquer n N valida desigualdade


an = 1+ 1 n
n

< 3.

8. Prove que, para todo n N e a > 0,


a+ a+ a + + a< 1+ 4a + 1 . 2

nradicais

9. Mostre que para qualquer nmero natural n 0, 11n+2 + 122n+1 sempre divisvel por 133. 10. Mostre que para todo n Z+ temos que 32n+1 + 2n+2 um mltiplo de 7.

6.6

Exerccios

231

11. Mostre que para todo n Z+ temos que 32n+2 + 26n+1 um mltiplo de 11. 12. Considere Fn a sequncia de Fibonacci . Mostre que
1 Fn = 5 1+ 5 2
n

1 5

1 5 2

13. Mostre as seguintes propriedades a respeito da sequncia de Fibonacci Fn :


n n

(a)
i=1 n

Fi = Fn+2 1; F2i = F2n+1 1;

(b)
i=1

F2i1 = F2n ;

(c)
i=1

2 (d) Fn1 Fn+1 Fn = (1)n .

14. De quantas formas diferentes podemos cobrir um tabuleiro de 2 n com peas de domins que cobrem exatamente duas celas do tabuleiro? 15. Calcular o nmero de regies em que o plano dividido por n retas distintas em cada uma das seguintes situaes: (a) as n retas so concorrentes; (b) no existem duas retas paralelas nem trs retas concorrentes.3 16. Dizemos que uma gura enquadrvel com rgua e compasso, se a partir dela possvel, utilizando apenas rgua e compasso, construir um quadrado de mesma rea. Prove que:
onde sabemos, este problema conhecido como a pizza de Steiner, o qual foi resolvido, em 1826, pelo notvel gemetra Jacob Steiner (1796-1863).
3 At

232

Induo Matemtica

(a) um tringulo sempre enquadrvel; (b) um polgono qualquer enquadrvel.

Sugesto para o item (b): Utilize induo dividindo a gura em tringulos.


17. D uma resposta situao Observao 6.17.

Sugesto: Observe a validade do argumento quando o conjunto A tem 2 elementos. Veja que B e C no se intersectam. Ou seja, o passo indutivo falha de n = 1 para n = 2.

Referncias Bibliogrcas
[1] AIGNER, M. e ZIEGLER, G. (2002). As Provas esto no Livro. Edgard Blcher. [2] GARCIA, A. e LEQUAIN, I. (2003). Elementos de lgebra. Projeto Euclides, IMPA. [3] LIMA, E. L.; CARVALHO, P. C. P.; WAGNER, E. e MORGADO, A.C. (2004). A Matemtica do Ensino Mdio. Volume 1. Sociedade Brasileira de Matemtica. [4] LIMA, E.L.; CARVALHO, P. C. P.; WAGNER, E. e MORGADO, A.C. (2004). A Matemtica do Ensino Mdio. Volume 2. Sociedade Brasileira de Matemtica. [5] LIMA,E.L.; CARVALHO,P. C. P.; WAGNER,E. e MORGADO,A.C. (2004). A Matemtica do Ensino Mdio. Volume 3. Sociedade Brasileira de Matemtica. [6] LIMA, E.L.; CARVALHO, P. C. P.; WAGNER,E. e MORGADO, A.C. (2001). Temas e Problemas. Sociedade Brasileira de Matemtica. [7] LIMA, E.L. (2001). lgebra Linear. Sociedade Brasileira de Matemtica.

285

286

REFERNCIAS BIBLIOGRFICAS

[8] MORAIS FILHO, D. C. (2007). Um Convite Matemtica. EDUFCG. [9] MORGADO, A.; CARVALHO, J.; CARVALHO, P.; FERNANDEZ, P. (1991). Anlise Combinatria e Probabilidade . Sociedade Brasileira de Matemtica. [10] RIBENBOIM, P. (2001). Nmeros Primos: Mistrios e Recordes. Sociedade Brasileira de Matemtica. [11] SANTOS, J. P. O. (1993) Introduo Teoria dos Nmeros. IMPA. [12] SANTOS, J. P. O.; MELLO, M. P. e MURARI, I. T. C. (2006). Introduo Anlise Combinatria. Editora Unicamp. [13] SOARES, M. G. (2005). Clculo em uma Varivel Complexa. Sociedade Brasileira de Matemtica.

em Matemtica em Rede Nacional

Mestrado Profissional

Iniciao Matemtica

Autores: Krerley Oliveira Adn J. Corcho

Unidade IV: Captulos VII e VIII

7
Desigualdades

Neste captulo estudaremos algumas desigualdades clssicas que so usadas frequentemente na resoluo de problemas matemticos, sendo estas aplicadas em contextos que variam desde o nvel mais simples at o mais complexo. Uma vez que uma inequao em uma ou mais variveis resolvida, o resultado d lugar a uma desigualdade que vlida para um certo conjunto de valores. Alguns exemplos simples de desigualdades so os seguintes: (a) (b) (c) (d)

x |x|, x2 < x,

para qualquer se

1 < x < 1;

x < 1;
para quaisquer

(x y )2 0,
x y

reais;

y x

2,

para quaisquer

x, y > 0.
233

234

Desigualdades

7.1

Desigualdade Triangular

A desigualdade triangular arma o seguinte

comprimento de um dos lados sempre inferior soma dos comprimentos dos outros dois lados, ou seja,
AB < AC + CB, AC < AB + BC

Teorema 7.1 (Desigualdade Triangular) . Dado um tringulo ABC o

e BC < BA + AC.

A
Figura 7.1:

Desigualdade Triangular

Em outras palavras, a desigualdade triangular a formulao matemtica da ideia intuitiva de que o caminho reto mais curto entre os pontos A e B. Em analogia com a geometria plana temos uma verso da desigualdade triangular para nmeros reais, que provamos a seguir.

Proposio 7.2. Sejam a e b nmeros reais quaisquer, ento


|a + b| |a| + |b|.

Demonstrao.
contrrio, se

a + b 0, ento |a + b| = a + b |a| + |b|. a + b < 0, ento |a + b| = a b |a| + |b|.


Se

Caso

7.1

Desigualdade Triangular

235

Corolrio 7.3. As seguintes desigualdades valem


|a b| |a| + |b|
(7.1) (7.2) (7.3)

|a b| |a| |b|,

|a b| |a| |b|

Demonstrao.

Para a primeira, escrevemos

|a| + | b| = |a| + |b|. A segunda |b + (a b)| |b| + |a b|. A ltima desigualdade segunda, trocando os papis de a e b. D

|a b| = |a + (b)| desigualdade decorre de |a| =


consequncia da

C A O P B
Figura 7.2:

Problema da central de energia

Exemplo 7.4. Quatro cidades rurais, A, B , C e D, esto situadas


geogracamente formando um quadriltero convexo. Deseja-se construir uma central de distribuio de energia para as quatro cidades de modo que a soma total das distncias da central a cada uma das quatro cidades seja a mnima possvel. Onde dever ser construda a central? Soluo.
no ponto

Mostraremos que a central de energia dever ser colocada

de interseco das diagonais do polgono

ABCD.

Com

236

Desigualdades

efeito, considerando um ponto

P,

diferente de

O,

(veja Figura 7.2) a

desigualdade triangular nos garante que

OA + OC = AC < P A + P C
e

OB + OD = BP < P B + P D,
de onde se segue que

OA + OC + OB + OD < P A + P C + P B + P D,
como espervamos.

separadas a uma distncia d. As torres so amarradas por uma corda AP B que vai do topo A da primeira torre para um ponto P no cho, entre as torres, e ento at o topo B da segunda torre, como na Figura 7.3. Qual a posio do ponto P que nos d o comprimento mnimo da corda a ser utilizada?
A B

Exemplo 7.5. Duas torres de alturas h1 e h2 , respectivamente, esto

P
Figura 7.3:

Problema das Torres

7.1

Desigualdade Triangular

237

Soluo.

Imaginemos que a superfcie do cho um espelho e que re-

etimos o ponto atravs deste, obtendo assim o ponto a Figura 7.4.

como mostra

A B

C P

B
Figura 7.4:

Soluo geomtrica do problema das torres


AB
que intercepta o cho no ponto

Consideremos o segmento

e para nossa surpresa vericaremos que este o ponto que nos d o comprimento mnimo das cordas. Com efeito, suponhamos que existe outro

situado entre as torres que nos d um comprimento menor

para a corda, ento da Figura 7.4 fcil ver que os tringulos e

B PD

so congruentes, assim como os tringulos

BP D

BP D BP D

tambm so congruentes. Logo, as seguintes igualdades seguem diretamente das congruncias:

BP = B P

BP = B P . AB P
e as igual-

Agora, usando a desigualdade triangular no tringulo dades acima, temos que

AP + P B = AP + P B AB = AP + P B = AP + P B,

238

Desigualdades

chegando assim concluso de que mento mnimo desejado.

AP + P B P

nos oferece o compri-

Agora calcularemos a que distncia est que

da base

D.

Lembremos

AC = h1 , BD = h2
tang(

CD = d

e observamos que

BP D) =

Da tem-se

PD =

dh2 . h1 + h2

h1 h2 = . PD d PD

7.2

Desigualdade das Mdias

As quantidades

Denio 7.6. Sejam a1 , a2 , . . . , an1 e an nmeros reais positivos.


mh (a1 , a2 , . . . , an ) = n , 1/a1 + 1/a2 + + 1/an n a1 a2 an ,
(7.4)

mg (a1 , a2 , . . . , an ) =

(7.5)

ma (a1 , a2 , . . . , an ) =

a1 + a2 + + an , n
2 2 a2 1 + a2 + + an n

(7.6)

mq (a1 , a2 , . . . , an ) =

(7.7)

so chamadas, respectivamente, de mdia harmnica, mdia geomtrica, mdia aritmtica e mdia quadrtica dos nmeros ai , i = 1, 2, . . . , n.
A seguir provaremos alguns resultados que estabelecem relaes de desigualdades entre as mdias denidas acima.

7.2

Desigualdade das Mdias

239

Proposio 7.7 (Desigualdade das Mdias Aritmtica e Quadrtica) .


Dados a1 , a2 , . . . , an nmeros reais positivos tem-se
a1 + a2 + + an n
2 2 a2 1 + a2 + + an , n

ou seja, ma (a1 , a2 , . . . , an ) mq (a1 , a2 , . . . , an ). Alm disso, a igualdade vale se, e somente se, a1 = a2 = = an . Demonstrao.
Usando a igualdade

1i<j n
conclumos que,

(ai aj ) = (n 1)

i=1

a2 i 2

ai aj
1i<j n

(7.8)

2
1i<j n

ai aj (n 1)

a2 i,
i=1

(7.9)

dado que o termo da esquerda em (7.8) no negativo. Somando em n ambos os membros de (7.9) a quantidade a2 i obtemos i=1

ai
i=1
donde, dividindo por

a2 i,
i=1

n2

e tomando a raiz quadrada, segue-se a desi-

gualdade desejada. Por ltimo, observamos que a igualdade em (7.9) atingida se, e somente se, e somente se,

Proposio 7.8 (Desigualdade das mdias Geomtrica e Aritmtica) .


Dados a1 , a2 , . . . , an nmeros reais positivos tem-se
a1 + a2 + + an n a1 a2 an , n

a1 = a2 = = an .

1i<j n

(ai aj )2 = 0,

o que verdade se,

240

Desigualdades

ou seja, mg (a1 , a2 , . . . , an ) ma (a1 , a2 , . . . , an ). Alm disso, a igualdade vale se, e somente se, a1 = a2 = = an . Demonstrao.
A prova desta desigualdade mais tcnica e exige um pouco mais de esforo. Dividiremos a mesma em dois passos.

Passo 1.
fato,

A desigualdade vale para

Procederemos por induo.

n = 2m . Para n = 2

a desigualdade vale. De

( a1 a2 )2 = a1 + a2 2 a1 a2 0. a +a Assim, a1 + a2 2 a1 a2 e conseqentemente 1 2 a1 a2 . 2 Agora provamos que se a desigualdade vale para n = k , tambm vale para n = 2k . Com efeito,
+a2k a1 ++ak + ak+1 +k a1 + + a2k k = 2k 2 (1) k a1 ak + k ak+1 a2k 2 (2) k a1 ak k ak+1 a2k = 2k a1 a2k ,
onde em (1) e (2) usamos a validade da desigualdade em para e para para

ento

n=k

n = 2, respectivamente. Logo, como j provamos a validade n = 2, claro que vale tambm para n = 4, 8, . . . , 2m , . . . , como
Dado

espervamos.

Passo 2.
todo

inteiro positivo, ento a desigualdade vale para

n<2

Para vericar isto, denimos o nmero

L=

n a1 an ,

7.2

Desigualdade das Mdias

241

e como a desigualdade vale para

n = 2m , =

temos ento que

a1 + + an + L + + L 2m
Portanto,

2m n vezes

2m

2m

a1 an L2m n Ln L2m n = L.

a1 + + an + (2m n)L L, 2m a1 + + an 2m L (2m n)L = nL,

logo

obtendo assim que

a1 + + an nL = n n a1 an ,
o que nos d a desigualdade desejada. Como para qualquer inteiro positivo positivo

sempre existe um inteiro

tal que

n<2

, a desigualdade ca provada para todo

A prova de que a igualdade s ocorre quando

pode tambm ser feita por induo e deixamos a cargo do leitor.

n. a1 = a2 = = an

Proposio 7.9 (Desigualdade das Mdias Harmnica e Geomtrica) .


Dados a1 , a2 , . . . , an nmeros reais positivos tem-se
n n a1 a2 an , 1/a1 + 1/a2 + + 1/an

ou seja, mh (a1 , a2 , . . . , an ) mg (a1 , a2 , . . . , an ). Alm disso, a igualdade vale se, e somente se, a1 = a2 = = an . Demonstrao.
n
Usando a Proposio 7.8 com os nmeros vale que

ai

substitu-

dos por

1/ai (i = 1, 2 . . . , n)
1/n

i=1

1 ai

1 = mg (1/a1 , . . . , 1/an ) ma (1/a1 , . . . , 1/an ) = n

i=1

1 . ai

242

Desigualdades

Invertendo esta ltima desigualdade, obtemos ento

mh (a1 , a2 , . . . , an ) mg (a1 , a2 , . . . , an ),
concluindo-se assim a prova. Notemos que as igualdades s ocorrem

1/a1 = 1/a2 = = 1/an an .


se

equivalem as igualdades

a1 = a2 = =

O prximo resultado resume as relaes provadas, nas proposies 7.7, 7.8 e 7.9, para as mdias

mh , mg , ma

mq .

Teorema 7.10

(Desigualdade das Mdias)

meros reais positivos a1 , a2 , . . . , an1 e an se vericam as seguintes desigualdades: min(a1 , . . . , an ) mh (a1 , a2 , . . . , an )


mg (a1 , a2 , . . . , an ) ma (a1 , a2 , . . . , an )

. Para toda coleo de n-

(7.10)

mq (a1 , a2 , . . . , an ) max(a1 , . . . , an ).

Alm disso, em cada caso a igualdade ocorre se, e somente se, a1 =


a2 = = an .

nusa sempre menor ou igual que a metade da hipotenusa. Alm disso, a igualdade s ocorre quando o tringulo retngulo issceles (ou seja, seus catetos so iguais). Soluo.
c = x+y
Usando a Figura 7.5, temos que a hipotenusa

Exemplo 7.11. Num tringulo retngulo a altura relativa hipote-

dada por

e usando o teorema das alturas para um tringulo retngulo

7.2

Desigualdade das Mdias

243

temos que

h2 = xy ,

logo

h=

xy.

A desigualdade entre as mdias

geomtrica e aritmtica nos d que

h=

xy

x+y c = , 2 2

como queramos. Alm disso, a altura a metade da hipotenusa se, e

a x

h y c

Interpretao geomtrica da desigualdade das mdias geomtrica e aritmtica


Figura 7.5: somente se, a igualdade entre as mdias ocorre, ou seja, quando Ento, os catetos

x = y.

a e b do tringulo so iguais, sendo este issceles.

Exemplo 7.12 (Desigualdade Isoperimtrica para Tringulos) . O pe-

rmetro de um tringulo de lados a, b e c a soma p = a + b + c. Entre todos os tringulos com permetro xado p o de maior rea o tringulo equiltero. Soluo.
Usando a Frmula de Hern temos que a rea de um tringulo com permetro

dada pela expresso

A=
onde

p p ( 2 2

a)( p b)( p c), 2 2 mg ma


3
temos que,

a, b

so os lados do tringulo.

Usando agora a desigualdade

p 2

p p p a+ b+ c 2 2 2 3

p2 . 12 3

244

Desigualdades

Logo a maior rea possvel

2 p , a qual atingida quando 12 3

p 2

a=

p 2

b=

p 2

c a = b = c,
Notemos que neste caso,

ou seja, quando o tringulo equiltero. 2 p a2 3 = . 4 12 3

Entre todos os paraleleppedos com rea lateral xada A o de maior volume o cubo (ou seja, o paraleleppedo com todos seus lados iguais).
c

Exemplo 7.13

(Desigualdade Isoperimtrica para Paraleleppedos)

b a
Figura 7.6:

A rea lateral de um paraleleppedo de lados a, b e c dada por AL = 2(ab + bc + ac).


Denotando por

Soluo.

a, b

as medidas das arestas do paralele-

ppedo sabemos que a soma das reas de todas as faces do paraleleppedo, ou seja,

AL = 2(ab + ac + bc).
Sendo

o volume do paraleleppedo e usando a desigualdade entre as

mdias aritmtica e geomtrica temos que

V 2 = ab ac bc

ab + ac + bc 3 V = a = b = c.

AL 6

(7.11)

Assim, o maior volume possvel

AL 3 , obtido quando 6

ab =

ac = bc,

consequentemente

7.3

Desigualdade de Cauchy-Schwarz

245

7.3

Desigualdade de Cauchy-Schwarz
(Desigualdade de Cauchy-Schwarz)

Teorema 7.14

an e b1 , . . . , bn nmeros reais tem-se |a1 b1 + + an bn |


2 x2 1 + + an

. Dados a1 , . . . ,
(7.12)

2 b2 1 + + bn

Alm disso, a igualdade s ocorre se existir um nmero real , tal que a1 = b1 , . . . , an = bn ou b1 = a1 , . . . , bn = an . Demonstrao.
n
Usando a identidade de Lagrange:

a2 i
i=1
temos que

b2 i
i=1

=
i=1

ai b i

1i<j n n n

( ai b j aj b i ) 2

ai b i
i=1

a2 i
i=1 i=1

b2 i,

de onde se obtm diretamente a desigualdade de Cauchy-Schwarz. Alm disso, a igualdade ocorre se, e somente se,

1i<j n

(ai bj aj bi )2 = 0 ai bj aj bi = 0, 1 i < j n, tal que ai = bi


ou

o que verdade se, e somente se, existe com

bi = ai ,

i = 1, 2, . . . , n.

e hipotenusa c xada, o que tem maior soma dos catetos s = a + b o tringulo issceles. Soluo.
Usando a desigualdade de Cauchy-Schwarz temos que

Exemplo 7.15. Entre todos os tringulos retngulos de catetos a e b

a+b=a1+b1

a2 + b2 12 + 12 = c 2

246

Desigualdades

e este mximo atingido quando

1 = b.

Em qualquer caso

a = 1 e b = 1 devemos ter a = b.

ou

1 = a

i n, nmeros reais, tem-se


n n

Exemplo 7.16

(Desigualdade de Minkowski)

. Dados ai , bi com 1
n

i=1

(ai + bi )2

a2 i +
i=1 i=1

b2 i.

Soluo.

Partimos da seguinte igualdade:

(ai + bi )2 =
i=1 i=1

a2 i +
i=1

b2 i +2
i=1

ai b i .

(7.13)

Aplicando a desigualdade de Cauchy-Schwarz no lado direito de (7.13) temos que

i=1

(ai + bi )

a2 i
i=1

+
i=1

b2 i

+2
i=1 n

a2 i
i=1

b2 i
(7.14)

2 . b2 i

a2 i +
i=1 i=1

Tomando raiz quadrada em ambos os membros de (7.14) obtemos a desigualdade de Minkowski.

7.4

Desigualdade de Jensen

A Desigualdade de Jensen est estreitamente relacionada com o conceito de convexidade, o qual explicamos a seguir.

7.4

Desigualdade de Jensen

247

Denio 7.17. Uma funo f : [, ] R dita convexa se para


quaisquer a, b [, ] e para todo [0, 1] satisfaz
f a + (1 )b f (a) + (1 )f (b).

y y = f (x) (b, f (b)) (a, f (a))

a
Figura 7.7:

Grco de uma funo convexa

Geometricamente, a denio de convexidade signica que para cada par de pontos

escolhidos no intervalo

[, ]

o grco da

funo encontra-se abaixo do segmento de reta secante que junta os pontos

(a, f (a))

(b, f (b)),

como mostra a Figura 7.7.

[, ].

Exemplo 7.18. A funo f (x) = x2 convexa em qualquer intervalo

Soluo.

Sejam

a, b [, ] e suponhamos, sem perda de generalidade,

248

Desigualdades

que

a < b.

Ento, para todo

[0, 1]

valem as desigualdades:

(a + (1 )b)2 = 2 a2 + (1 )2 b2 + 2(1 )ab


(1)

= a2 [2 + (1 )] + b2 [(1 )2 + (1 )] = a2 + (1 )b2 , ab
a2 +b2 . 2

2 a2 + (1 )2 b2 + (1 )(a2 + b2 )

(7.15)

onde na passagem (1) usamos a desigualdade

[, ] com positivo.

Exemplo 7.19. A funo f (x) = 1/x convexa em qualquer intervalo


Sendo

Soluo.

a, b [, ]

com

a < b,

para todo

[0, 1]

tem-se

1 = ( + (1 ))2
(1)

a b + (1 ) + (1 )2 b a a b = 2 + (1 ) + (1 ) + (1 )2 b a (1 ) (1 ) = a + +(1 )b + a b a b (1 ) = a + (1 )b + a b 2 +
onde na passagem (1) usamos que meros positivos

= 2 + 2(1 ) + (1 )2

(7.16)

b.

De (7.16) segue-se que

a/b + b/a 2

para quaisquer n-

1 1 1 + (1 ) , a + (1 )b a b
mostrando isto a convexidade da funo

1 . x

7.4

Desigualdade de Jensen

249

Observemos que, usando a desigualdade entre as mdias aritmtica e quadrtica obtemos

a1 + a2 + + an n
em outras palavras

2 2 a2 1 + a2 + + an , n

2 2 (ma (a1 , a2 , . . . , an ))2 ma (a2 1 , a2 , . . . , an ).

(7.17)

Por outro lado, a desigualdade entre as mdias harmnica e aritmtica nos garantem que

1 ma (1/a1 , 1/a2 . . . , 1/an ). ma (a1 , a2 , . . . , an ) x2

(7.18)

O seguinte resultado garante que as propriedades (7.17) e (7.18), satisfeitas pelas funes convexas convexa. e

1 , so vlidas para qualquer funo x

Teorema 7.20

(Desigualdade de Jensen)

funo convexa e sejam i [0, 1] (i = 1, . . . , n) tais que Ento, para quaisquer ai [, ] (i = 1, . . . , n) vale
f (1 a1 + + n an ) 1 f (a1 ) + + n f (an ). 1/n, a desigualdade de Jensen nos diz que f

. Seja f : [, ] R uma
n i=0

i = 1.

(7.19)

Observao 7.21. Observemos que, quando 1 = 2 = = n =


a1 + a2 + + an f (a1 ) + f (a2 ) + + f (an ) , n n

ou seja, f (ma (a1 , . . . , an )) ma (f (a1 ), . . . , f (an )).

250

Desigualdades

Demonstrao.

Faremos a prova por induo. Para

n=2

a validade

decorre diretamente da denio.

Suponhamos que dado

natural

(7.19) vale, ento temos que provar a validade de

n+1

n+1

f
j =1
Notemos que

j aj

j f (aj ).
j =i

(7.20)

n+1

j aj =
j =1

j =1 n

j aj + 1

j an+1
j =1
(7.21)

=
j =1 n
onde

j aj + (1 )an+1 ,
n j

=
j =1

j . Assim, usando que


j =i n n+1

= 1 e a hiptese de induo,

obtemos

f
j =1

j aj f
n

j =1

j aj +(1 )f (an+1 )
(7.22)

j =1

j f (aj ) + (1 )f (an+1 )

n+1

j f (aj ),
j =1

como queramos provar.

7.5

Exerccios

1. Provar que em todo tringulo a soma dos comprimentos das medianas menor que o permetro do tringulo e maior que o semipermetro deste.

7.5

Exerccios

251

2. Os centros de trs crculos que no se intersectam esto sobre uma reta. Prove que se um quarto crculo toca de forma tangente os trs crculos, ento o raio deste maior que pelo menos um dos raios dos trs crculos dados.

n
3. Dado

inteiro positivo, provar que

j =1
4. A soma de trs nmeros positivos seus quadrados no menor que

2n 1 . j n+1
Provar que a soma de

6.

12.

5. Determinar as dimenses do paraleleppedo de menor diagonal possvel, sabendo que a soma dos comprimentos de todas suas arestas

12.

6. Encontrar todas as solues positivas do sistema de equaes no lineares

x 2 + + x 2 = 1 1 10 1 1 2 + + 2 = 100.
x1 x10

7. Demonstrar que, se

a1 , a2 , . . . , an

so nmeros positivos tais que

a1 a2 an = 1
ento

(1 + a1 )(1 + a2 ) (1 + an ) 2n .
8. Prove que a mdia geomtrica super-aditiva, isto , para nmeros no negativos

ai

bi , 1 i n,
n
n

tem-se

n
n

ai +
i=1

i=1

bi

(ai + bi ).
i=1

252

Desigualdades

Alm disso, estude em que condies ocorre a igualdade.


Sugesto: Use a desigualdade entre as mdias geomtrica e aritmtica.

9. Usar o mtodo de induo para provar a desigualdade de CauchySchwarz.

n
10. Para todo

Use este fato para dar outra i=1 prova da desigualdade de Cauchy-Schwarz.

real

(ai + bi )2 0.

11. Use a desigualdade de Cauchy-Schwarz para dar uma prova alternativa da desigualdade entre as mdias aritmtica e quadrtica (ma

mq ).
n

12. Prove que

i=1
13. Prove que

1 ai b i 2

a2 i
i=1

+
i=1

b2 i

a4 + b4 + c4 abc(a + b + c). a 0, b 0
e

14. Prove que se

c 0,

ento

(a + b)(a + c)(b + c) 8abc.


15. Prove a desigualdade de Bernoulli: qualquer

(1 + x)n > 1 + nx,

para

positivo e

n d

inteiro positivo.

16. Prove que se

a, b, c

so inteiros positivos, ento:

(a + b + c + d)
17. Prove que se

1 1 1 1 + + + a b c d

16.

c 0, ento (ab + bc + ca) a bc + b ac + c ab.


e

a 0, b 0

7.5

Exerccios

253

18. Prove que se

x 0, x 0,

ento

3x3 6x2 + 4 0. 2x + 3/8 4 x. r1


e

Sugesto: Use a desigualdade entre as mdias aritmtica e geomtrica.

19. Prove que se 20. Sejam

ento

C1

C2

dois crculos concntricos de raios

r2 ,

res-

pectivamente, com pontos ponto

r1 < r2 .

Sobre o crculo

C1

se marcam dois encontrar o

P1 e P2 diametralmente opostos. Deseja-se P sobre o crculo C2 que maximiza a soma d(P ) = P P1 + P P2 .

254

Desigualdades

8
Polinmios

8.1

Operaes com Polinmios

A necessidade de estudar equaes polinomiais aparece em problemas prticos da humanidade desde pocas muito remotas. Indcios arqueolgicos indicam que os babilnicos j tinha o domnio de tcnicas de resoluo de algumas equaes do primeiro grau e do segundo grau, apresentadas em forma de problemas cotidianos. Contudo, o grande avano terico no estudo das equaes polinomiais s se iniciou com o Renascimento na Europa. No incio do sculo XVI, Viti introduziu o uso de letras para representar quantidades desconhecidas. Na mesma poca, um outro grande desao estava perturbando as mentes matemticas de toda a Europa, em especial as da Itlia. A

soluo explcita utilizando as operaes elementares (soma, subtrao, multiplicao, diviso, radiciao e potenciao) da equao do

255

256

Polinmios

terceiro grau no era conhecida e muitos dos melhores matemticos da poca trabalharam neste problema, destacando-se entre eles Nicolo Fontana, o

Tartaglia

(gago, em italiano). A histria da soluo

desta equao est repleta de intrigas, disputas e acusaes, envolvendo Tartaglia e Cardano. Hoje os historiadores atribuem a Tartaglia a primazia na descoberta da soluo da equao do terceiro grau como conhecemos. desta poca tambm a soluo da equao do

quarto grau, atribuda a Ludovico Ferrari. Entretanto, apesar dos muitos esforos empreendidos na direo de encontrar a soluo geral da equao do quinto grau, mais de 200 anos se passaram sem nenhum sucesso. noruegus Niels Abel mostrou que At que em 1824, o matemtico

impossvel

resolver as equaes de

grau cinco em sua forma geral. Ou seja, nem todas as equaes de grau cinco podem ser resolvidas com as operaes elementares. Mais ainda, em 1830 o matemtico francs Evariste Galois descobriu um mtodo que determina quando uma equao de grau

qualquer

resolvel com

as operaes elementares, encerrando um belssimo captulo do estudo das equaes polinomiais e da Matemtica. Neste captulo iremos estudar um pouco mais formalmente os polinmios e suas propriedades.

Denio 8.1. Um polinmio na varivel x uma expresso do tipo


p(x) = an xn + an1 xn1 + + a1 x + a0

onde a0 , a1 , . . . , an so nmeros. Se an = 0, dizemos que n o grau do polinmio e a0 , a1 , . . . , an so seus coecientes. O coeciente an chamado de coeciente lder do polinmio. todos os coecientes iguais a zero.

Observao 8.2. No se dene o grau do polinmio nulo, que tem

8.1

Operaes com Polinmios

257

Por exemplo,

p(x) = 3x 1

um polinmio de grau 1; um polinmio de grau 3;

4 x um monmio de grau 4; 2 v (x) = x4 + 5x2 + 1 um polinmio 2 t(x) = u(x) = 7


um polinmio de grau

q (x) = 4x3 + 7x + 1

de grau 4;

0.

n, ou simplesmente uma equao de grau n, uma sentena p(x) = 0, onde p(x) um polinmio de grau n com coecientes reais. Por exemplo, 2x 1 = 0 uma equao 5 3 do primeiro grau, enquanto x + 4x + 5x 1 = 0 uma equao de grau 5. Note que nem todos os coecientes precisam ser diferentes de
Uma equao polinomial de grau zero. Para obtermos o

valor

do polinmio

+ a1 x + a0

no nmero real

p(x) = an xn + an1 xn1 + r, devemos substituir x por r para obter

o nmero real

p(r) = an rn + an1 rn1 + + a1 r + a0 .


Por exemplo, o valor do polinmio

4 23 7 2 + 1 = 19.

p(x) = 4x3 7x + 1

em

p(2) =

Dizemos que um nmero real

uma raiz para a equao

an xn + an1 xn1 + + a1 x + a0 = 0
se o valor de seja, se

verica

p(x) = an xn + an1 xn1 + + a1 x + a0 an rn + an1 rn1 + + a1 r + a0 = 0.

em

zero, ou

258

Polinmios

Por exemplo,

raiz da equao:

2x 10 = 0.
Uma das vantagens dos polinmios sobre outros objetos matemticos que podemos denir as operaes de soma de polinmios e multiplicao de polinmios. Com estas operaes, o conjunto dos polinmios possui muitas propriedades similares dos nmeros inteiros, tornando prtico o seu uso. Vamos denir agora o que signica a

soma

de dois polinmios.

Para isso, vamos comear somando dois monmios e depois estender nossa denio para polinmios em geral.

polinmio:

p(x) = ak xk e q (x) = bk xk somamos seus coecientes, obtendo o polinmio t(x) = p(x) + q (x) = (ak + bk )xk . Em geral, para somar o polinmio p(x) = a0 + a1 x + a2 x2 + + an xn com o polinmio q (x) = b0 + b1 x + + bm xm , onde n m devemos somar todos os monmios de mesmo grau, obtendo o
Para somar dois monmios de mesmo grau

t(x) = p(x) + q (x) = c0 + c1 x + + cm xm


onde,

c i = ai + b i

para

Por exemplo, sendo

0in

ci = b i

para

i > n.

p(x) = 3x 1, q (x) = 4x3 + 7x + 1, t(x) = x4 , 2 v (x) = x4 + 5 x2 + 1 2

8.1

Operaes com Polinmios

259

temos que

p(x) + q (x) = 4x3 + (3 + 7)x 1 + 1 = 4x3 + 10x, v (x) + t(x) = ( )x4 + 5x2 + 1 = 5x2 + 1. 2 2
A seguir, enumeramos algumas propriedades simples e importantes da soma de polinmios que decorrem da denio dada e das propriedades anlogas vlidas para os nmeros reais. 1.

Associatividade .

Dados polinmios

p(x), q (x)

t(x),

vale

(p(x) + q (x)) + t(x) = p(x) + (q (x) + t(x))


2.

Elemento neutro .

Se 0 denota o polinmio nulo e

p ( x)

um

polinmio qualquer, ento

0 + p(x) = p(x).
3.

Elemento simtrico .
polinmio, ento o satisfaz:

p(x) = a0 + a1 x + + an xn um n polinmio q (x) = a0 a1 x an x


Se

p ( x) + q ( x) = 0 .
4.

Comutatividade .

Se

p(x)

q (x)

so polinmios, ento

p(x) + q (x) = q (x) + p(x).


Note que os nmeros inteiros possuem propriedades similares para a operao de soma de nmeros inteiros. Vamos agora denir o

produto

de dois polinmios. Para isso, vamos primeiramente denir o produto de dois monmios, como j zemos no caso de soma de polinmios.

260

Polinmios

n, m so nmeros naturais, denimos p(x) = an xn e q (x) = bm xm como:


Se

o produto dos monmios

p(x)q (x) = an bm xn+m .


Tendo isto em mente, para efetuarmos o produto do polinmio de grau

n, p(x) = a0 + a1 x + a2 x2 + + an xn pelo polinmio q (x) = b0 + b1 x + + bm xm de grau m, com n m, devemos:


Completamos a escrita de colocando Denimos

ak = 0

para

p(x) e de q (x) at o termo n + m k > n e bk = 0 para k > m;

t(x) = p(x)q (x) = c0 + c1 x + + cn+m xn+m


onde,

ci = a0 bi + a1 bi1 + + ai1 b1 + ai b0

para

0 i n + m.

Apesar de parecer complicada, a denio no to difcil de ser aplicada. Para tentar visualizar o processo de multiplicao de dois polinmios vamos pensar que os monmios so seres aliengenas vindos do distante planeta de Algebrum e possuam mos. Quando dois monmios se encontram, invariavelmente eles apertam as mos e desse aperto aparece o produto desses monmios. Assim, para multiplicar os polinmios

p(x)

q (x),

que so forma-

dos por dois grupos de monmios, devemos escolher o primeiro monmio de

p ( x)

e faz-lo apertar a mo de cada um dos monmios de

q (x),

somando os monmios obtidos. Aps isso, tomamos o segundo

monmio de mios de

p ( x)

e fazemos ele apertar a mo de cada um dos mon-

q (x), somando os monmios obtidos aos monmios anteriores. Repetimos o processo at o ltimo monmio de p(x).

8.1

Operaes com Polinmios

261

Deste modo, se obter

p(x)q (x)

fazemos:

p ( x) = x2 + 2x 3

q (x) = x2 + 5x + 1,

para

p(x)q (x) = x4 + 5x3 + x2 2x3 + 10x2 + 2x + 3x2 15x 3 = x4 + 3x3 + 14x2 13x 3.
Observe que com a denio de multiplicao de polinmios dada acima, o coeciente do termo e o

c0

igual a

a0 b 0 .

Do mesmo modo, o coeciente

cn+m = an bm . Como p(x) tem grau n (isto , an = 0) q (x) tem grau m (bm = 0), o coeciente cn+m = an bm = 0. Logo, polinmio p(x)q (x) tem grau n + m. Com isso, demonstramos o

n+m

seguinte fato:

Proposio 8.3. Se o polinmio p(x) tem grau n e o polinmio q(x)


tem grau m, ento o polinmio p(x)q (x) tem grau n + m.
Um caso particular interessante quando multiplicamos um nme-

c, que podemos considerar como sendo um polinmio de grau zero q (x) = c, por um polinmio p(x) = a0 + a1 x + + an xn . Neste caso,
ro ns obtemos o polinmio

cp(x) = ca0 + ca1 x + + can xn .


Do mesmo modo em que podemos vericar as propriedades da soma de polinmios a partir das propriedades similares dos nmeros reais, podemos tambm vericar as propriedades abaixo sobre a multiplicao de polinmios. Deixamos essa vericao como exerccio.

1.

Associatividade .

Dados polinmios

p(x), q (x)

t(x),

vale

(p(x)q (x))t(x) = p(x)(q (x)t(x))

262

Polinmios

2.

Elemento neutro .

Se 1 denota o polinmio constante e

p ( x)

um polinmio qualquer, ento

1p(x) = p(x).
3.

Comutatividade .

Se

p ( x)

q (x)

so polinmios, ento

p(x)q (x) = q (x)p(x).


4.

Distributividade .

Se

p(x), q (x)

t(x)

so polinmios, ento

(p(x) + q (x))t(x) = q (x)t(x) + p(x)t(x).


Note que, assim como nos inteiros, a propriedade de existncia de elementos inversos para a multiplicao de polinmios no vale. De

p(x) um polinmio de grau n maior ou igual a um, ento no existe um polinmio q (x) tal que p(x)q (x) = 1. De fato, suponha por absurdo, que exista q (x) um polinmio com grau m 0 tal que
fato, podemos vericar que se

p(x)q (x) = 1.

Ento, utilizando a Proposio 8.3 temos que o grau de

p(x)q (x)

n+m

que maior ou igual que um.

Como o grau do polinmio

constante

zero, temos que a igualdade acima no pode valer, onde

chegamos a um absurdo. Em resumo, os nicos polinmios que podem ter inversos com respeito operao de multiplicao so os polinmios constantes no nulos. Esta mais uma das semelhanas entre os inteiros e os polinmios.

8.2

Algoritmo de Euclides

263

8.2

Algoritmo de Euclides
a(x) divide o polinmio b(x) se existir q (x) a(x) = x2 + x + 1
divide o polinmio

Diremos que um polinmio tal que

b(x) = q (x)a(x).
pois

Por exemplo, o polinmio

x3 1

(x 1)(x2 + x + 1) = x3 1. a(x)
divide o polinmio

Devido Proposio 8.3, se o polinmio no nulo

b(x),

ento o grau de

a( x )

menor ou igual ao grau de

b(x).

Agora, vamos enunciar um fato que vale para os inteiros e que vale tambm para os polinmios e que ser de grande utilidade. Pedimos que o leitor releia o

algoritmo de Euclides ,

estudado no Captulo 3.

No conjunto dos polinmios, ainda vale

mios com coecientes reais, b(x) = 0. Ento, existem polinmios com coecientes reais q (x) e r(x), com r(x) = 0 ou grau de r(x) menor que o grau de b(x) tais que:
a(x) = b(x)q (x) + r(x).

Teorema 8.4 (Algoritmo de Euclides) . Sejam a(x) e b(x) dois polin-

Alm disso, q (x) e r(x) esto determinados de modo nico. Demonstrao.


que Vamos mostrar primeiro a unicidade. De fato, assuma

a(x) = b(x)q1 (x) + r1 (x) = b(x)q2 (x) + r2 (x),


com

r1

r2

de graus menores que o grau de

b.

Assim,

b(q1 q2 ) = r2 r1 .

264

Polinmios

Consequentemente,

q1 = q2 , b.

j que caso contrrio, o polinmio

q2 )

teria grau pelo menos igual ao grau de

e o polinmio

grau menor que o grau de

b(q1 r2 r1 tem

Vamos agora mostrar a existncia. Os passos da prova so idnticos a prova do algoritmo de Euclides para nmeros inteiros, demonstrado no Captulo 3. De fato, a ideia reduzir o grau do dividendo at

que ele se torne menor que o do divisor e a diviso se torne imediata.

a tem grau menor que b, ento tomamos o resto com sendo r = a e o quociente como sendo q = 0. Suponhamos que a(x) = an xn + + a1 x + a0 tenha grau n e b(x) = bm xm + + b1 x + b0 tenha grau m e que n > m. Dena
Note que se

c1 (x) = a(x)
Observe que o grau de por

an nm x b(x). bm

b,

digamos com

c1 no mximo n 1. Se c1 puder se dividido c1 (x) = b(x)q (x) + r(x), com grau de r(x) menor

que o grau de

b(x),

ento

a(x) = b(x)

an nm an x + c1 (x) = b(x)( xnm + q (x)) + r(x). bm bm

a(x) por b(x) pelo problema de dividir o polinmio c1 (x) por b(x), com c1 (x) de grau menor que a(x). Repetimos o processo, utilizando c1 no lugar de a(x), obtendo o polinmio c2 (x) de grau menor que o de c1 (x). Como a cada
Logo, reduzimos o problema de dividir o polinmio passo reduzimos o grau do dividendo em pelo menos uma unidade, ao

n m passos, obteremos um polinmio com grau menor que o grau de b(x), que claramente divisvel por b(x). Procedendo como antes, achamos q (x) e r (x) tais que a(x) = b(x)q (x)+ r (x) e r (x) com grau menor que o grau de b(x).
m de no mximo

8.2

Algoritmo de Euclides

265

Por exemplo, se

q (x) = 10x

a(x) = 10x3 3x + 2 r(x) = 13x + 2 temos que

b(x) = x2 + 1,

tomando

10x3 3x + 2 = (x2 + 1)10x + (13x + 2).


Note que o grau de

x2 + 1 . x

r(x) = 13x + 2 p(x)

menor que o grau de

b(x) =

Se na expresso do polinmio por um nmero real

decidimos substituir a varivel o polinmio

s,

estaremos

avaliando

p(x) x

em

s 2,

e denotamos este nmero por Por exemplo, se temos que

p(s). p(x) = x + 3x + 1,
2

ento substituindo

por

p(2) = 22 + 3 2 + 1 = 11
e fazendo

x = 3 p(3) = (3)2 + 3 (3) + 1 = 1.

p(s) = 0 dizemos que s anula o polinmio no nulo p(x), ou ainda, que s uma raiz do polinmio p(x). 3 Por exemplo, para p(x) = x 8, temos que 2 uma raiz de p(x) 3 j que p(2) = 2 8 = 0.
Quando Um fato muito importante que consequncia do algoritmo de Euclides o seguinte teorema:

Teorema 8.5. Se s uma raiz do polinmio p(x), ento o polinmio

x s divide p(x). Reciprocamente, se x s divide p(x), ento s raiz de p(x).

Demonstrao.

Primeiramente, assuma que

caso, existe um polinmio

q (x)

tal que

x s divida p(x). Neste p(x) = q (x)(x s). Avaliando

266

Polinmios

o polinmio

p(x)

em

s,

temos que:

p(s) = q (s)(s s) = q (s) 0 = 0.


Logo

uma raiz de

p(x) ento x s divide p(x), vamos utilizar o algoritmo da diviso, com a(x) = p(x) e b(x) = x s. Neste caso, temos que existem q (x) e r (x) de modo que r (x) = 0 ou o grau de r (x) menor que o grau de x s e alm disso vale
Para provar que raiz de

p(x). se s uma

p(x) = q (x)(x s) + r(x). r(x), podemos escrever que r(x) = c R. Ento, p(x) = q (x)(x s)+ c e 0 = p(s) = q (s) 0+ c = c. Portanto, r (x) = 0 e p(x) = q (x)(x s), isto , x s divide p(x).
Observe que, com as condies do resto A proposio anterior nos permite determinar o nmero mximo de razes reais de um polinmio no nulo. De fato, vamos mostrar.

Proposio 8.6. O nmero mximo de razes reais do polinmio no


nulo p(x) = an xn + an1 xn1 + + a1 x + a0 n. Demonstrao.
Digamos que distintas do polinmio

p(x).

Observe que podemos utilizar a Propo-

s0 < s1 < s2 < < sk

sejam razes

sio 8.5 para garantir que existe um polinmio no nulo que

q 1 ( x)

tal

p(x) = q1 (x)(x s0 ).
Assim, pela Proposio 8.3, o grau de Note que

q 1 ( x)

deve ser igual a

p(si ) = q1 (si )(si s0 ). Como para todo i = 1, 2, . . . , k temos que si > s0 com p(si ) = 0, temos que, necessariamente, q1 (si ) = 0. Assim, em particular, temos que q1 (s1 ) = 0. Logo, podemos aplicar

n 1.

8.2

Algoritmo de Euclides

267

a proposio novamente para obter que existe um polinmio no-nulo

q 2 ( x)

tal que

q1 (x) = q2 (x)(x s1 ). q1 (x) n 1, pela Proposio 8.3, o grau de q2 (x) deve ser igual a n 2. Novamente, temos que q1 (si ) = q2 (si )(si s1 ), si > s1 e p(si ) = 0 para todo i = 2, . . . , k . Disto segue que, necessariamente, q2 (si ) = 0, se i = 2, 3, . . . , k. Assim, temos que q2 (s2 ) = 0.
Assim, como o grau de Logo, podemos repetir esse argumento para obter um polinmio

q3 (x) de grau n 3, de modo que s3 , s4 , . . . , sk

q3 (x). Repetindo o argumento, encontramos uma sequncia q1 (x), q2 (x), q3 (x), . . . com graus no mximo n 1, n 2, n 3, . . . o que nos leva a concluir que no podemos repetir esse argumento mais que n vezes, j que os graus dos polinmios q1 (x), q2 (x), q3 (x), . . . esto diminuindo. Ou seja, no podemos ter mais que n razes para o polinmio p(x), o
so razes de que conclui a prova.

Alertamos que, apesar da Proposio 8.6 nos garantir que existem no mximo

n razes reais de um polinmio de grau n no nulo, existem 2 polinmios que no possuem razes reais. Por exemplo, p(x) = x + 1 2 no possui razes rais, j que x 0 para todo nmero real x.
Uma consequncia da Proposio 8.6 a seguinte:

Proposio 8.7. Se dois polinmios p(x) e q(x) de grau n avaliados

em n + 1 nmeros r1 , r2 , . . . , rn+1 coincidem, isto , p(ri ) = q (ri ) para i = 1, 2, 3, . . . , n + 1, ento p(x) e q (x) so iguais.
Considere o polinmio

Demonstrao.
que se

t(x)

no-nulo, o grau de

t(x) = p(x) q (x). t(x) no mximo n, j

Observe que

p(x)

268

Polinmios

q (x) tm graus p(ri ) = q (ri ) e


e

iguais a

n.

Observe ainda que

t(ri ) = 0,

j que

t(ri ) = p(ri ) q (ri ) = 0.


Logo,

t(x)

tem grau no mximo

e mais de

razes, contradizendo a

Proposio 8.6. No Exerccio 23 faremos uma aplicao interessante dessa proposio, propondo que voc prove que dados nmeros reais

an+1 e r1 , r2 , . . . , rn+1 , que p(ri ) = ai .

ento existe um nico polinmio

a1 , a2 , . . . , de grau n tal

8.3 Sempre Existem Razes de um Polinmio?


Pode parecer frustrante o fato de que um polinmio com coecientes reais pode no possuir razes reais. Por exemplo, quando tentamos

aplicar a frmula de Bhaskara equao

x2 + 1 = 0,

encontramos

elas se escreveriam como

= 4

e, consequentemente, se fosse possvel escrever as solues,

x1 =
e

4 2

4 x2 = 2
claro que as expresses acima no tm sentido no conjunto dos

nmeros reais, pois no existe nmero cujo quadrado seja no possvel extrair a raiz quadrada de

vrias geraes de matemticos. Desde Hern de Alexandria h dois mil anos atrs, os matemticos encontram expresses como a do tipo acima, envolvendo razes de nmeros negativos.

4.

Isso tirou o sono de

4, ou seja,

8.3

Sempre Existem Razes de um Polinmio?

269

A primeira reao da comunidade matemtica foi rejeitar esses nmeros

complexos

e simplesmente desconsiderar razes de nmeros

negativos. Porm, j no sculo XVI, Cardano se deu conta de que os nmeros complexos surgem naturalmente quando desejamos resolver uma equao do terceiro ou quarto grau, mas relutava quanto ao seu uso, dizendo que esses nmeros eram to sutis, quanto inteis. No sculo seguinte, motivado pela sugesto de Albert Girard que uma equao de grau

n possui n razes, Rene Descartes observou que


para as razes que no so reais.

os nmeros reais eram insucientes para representar todas essas razes e utilizou o termo

imaginrias

A notao tradicional

mais tarde, com Leonard Euler, que tambm o pai do termo

i = 1 s veio a ser introduzida um sculo

nmero

complexo.

Euler e o matemtico francs Jean D'Alambert zeram apli-

caes dos nmeros complexos a problemas prticos, como projeo de mapas e hidrodinmica. Euler e Lagrange, grandes matemticos

da histria da humanidade, tentaram mostrar a armao de Girard, de que

uma equao de grau n possui n razes,

mas sem sucesso. A

primeira prova correta de tal teorema s apareceu no nal do sculo XVIII com os trabalhos de Gauss.

8.3.1 Nmeros Complexos e Razes de Polinmios


O conjunto dos nmeros complexos, denotado pela letra junto das expresses

C,

o con-

C = {x + iy ; x, y R},
onde que

satisfaz

meramente um smbolo que nos ajudar a denir as operaes

i2 = 1.

Costuma-se denotar

por

1.

Destacamos

de soma e de multiplicao de nmeros complexos. Essas operaes

270

Polinmios

tero as mesmas propriedades que as operaes de nmeros reais, como associatividade, comutatividade, elemento neutro, etc. Por exemplo, so nmeros complexos

Vamos denir a soma e multiplicao de nmeros complexos. Dados dois nmeros complexos

2 3i, 3 + i a + bi

3i.

c + di

denimos a soma como:

(a + bi) + (c + di) = (a + b) + (c + d)i


e denimos a multiplicao como

(a + bi)(c + di) = (ac bd) + (bc + ad)i


Por exemplo, se tomamos os nmeros

2 3i

3 + 4i

ento

(2 3i) + (3 + 4i) = 5 + i
e

(2 3i)(3 + 4i) = (2 3 (3 4)) + (3 3 + 2 4)i = 18 i.


Aqui ns estamos considerando nos permite

0 + 3i = 3i r

colocar

os nmeros reais dentro do conjunto dos nmeros como sendo um nmero

3 + 0 i = 3.

Isso

complexos, considerando cada nmero real complexo da forma

Fica para o leitor a vericao de que valem as propriedades de associatividade, comutatividade, etc. O elemento neutro da soma o elemento

r + 0 i.

modo, o elemento neutro da multiplicao por

0+0i

que simplesmente denotaremos por

0.

Do mesmo

1.

O leitor curioso pode achar mais informaes sobre nmeros

1+0 i, que ser denotado

complexos e solues de equaes algbricas em [5] ou [13].

8.3

Sempre Existem Razes de um Polinmio?

271

Assim, dado um nmero complexo (de coecientes complexos ou reais)

a1 x + a0

em

z,

obtendo o nmero complexo

z faz sentido avaliar o polinmio p(x) = an xn + an1 xn1 + +

p(z ) = an z n + an1 z n1 + + a1 z + a0 .
Por exemplo, se polinmio, j que:

p(x) = x2 + 4,

ento

2i

2i

so razes deste

p(2i) = (2i)2 + 4 = 4 + 4 = 0.
e

p(2i) = (2i)2 + 4 = 4i2 + 4 = 4 + 4 = 0.


Note que

p(x) no possui nenhuma raiz real, mas possui duas razes

complexas. Como j mencionamos, a grande vantagem em utilizar os nmeros complexos em vez dos nmeros reais que, dado um polinmio qualquer com coecientes complexos, ele sempre tem uma raiz complexa. Isso foi o assunto da tese de doutorado do

Prncipe da

Matemtica, Johann Carl Friedrich Gauss (1777-1855). no constante com coecientes complexos de grau n possui exatamente n razes complexas, contadas com multiplicidade.
Uma demonstrao do Teorema Fundamental da lgebra foge do objetivo deste livro. Podem ser dadas vrias demonstraes diferentes desse teorema, utilizando diversas teorias matemticas avanadas. Uma demonstrao desse teorema pode ser achada em [13].

Teorema 8.8

(Teorema Fundamental da lgebra)

. Todo polinmio

272

Polinmios

8.4

Exerccios
p(x)
por

1. Calcule o quociente e o resto da diviso de os polinmios

q (x)

para

p(x)

q ( x)

dados:

(a) (b) (c)

p(x) = 3x3 2x + 1 p(x) = x5 1


e

q (x) = 7x 1; q ( x) = x2 + x + 1
de forma que

q (x) = x 1;
e

p(x) = 3x5 2x3 + 1 A

2. Encontre os valores de

A B x+1 = + . 2 x x x x1
3. Se os polinmios

a + b.

x2 x +4 e (x a)2 +(x + b) so iguais, encontre a e b que tornam iguais P2 (x) = (x + a)2 b? x4 + 1 P (x).
tem quociente os polinmios

4. Quais os valores de

P1 ( x ) = x x 6
5. A diviso de

P (x)

por

x+2

e resto 1.

Encontre o polinmio

6. Qual o resto da diviso do polinmio

x100

por

x + 1?
polinmio

7. Determine o resto da diviso do polinmio

g (x) = x,

onde

p(x) pelo p(x) = (x 1)(x 2) . . . (x n) + b .


divisvel por

8. Mostre que

xn 1

x1

para todo

n 1.

9. Faa os seguintes itens: (a) encontre o quociente da diviso de

xn+1 1

por

x 1;

8.4

Exerccios

273

(b) utilize a diviso anterior para calcular a soma

1 + x + x2 +

x3 + + xn

dos

primeiros termos de uma progresso

geomtrica de razo

x.
divisvel

10. Determine o valor de por

x a,

onde

a para que o polinmio P (x) seja P (x) = x3 + (1 a)x2 + (1 + a)x 1. P (x) = x100 2x50 + 1
da diviso do polinmio

11. Mostre que o polinmio

divisvel por

x 1.
12. Mostre que o resto

r(x)

p(x)

por

r(x) = p(s).

xs

Dado o polinmio denimos a

p(x) = an xn + an1 xn1 + + a1 x + a0 derivada de p(x) como sendo o polinmio:

p (x) = nan xn1 + (n 1)an1 xn2 + + 2a2 x + a1 . 5x4 e a 3 2 2 derivada do polinmio x +5x +2x1 o polinmio 3x +10x+2.
Por exemplo, a derivada do polinmio o polinmio 13. Usando as informaes do Exerccio 12, calcule: (a) a derivada dos polinmios: (i) (ii) (ii)

x5

x + 1; x4 + 3 ; 1 + x + x2 + x3 + + xn . p(0) = 1,
calcule tambm o polinmio

(b) Sabendo que

p(x)

cuja derivada (i) (ii)

x4 . x2 + 1 .

274

Polinmios

(ii)

x3 + 2 x2 + 3 . p ( x)
e

(c) Prove que se (i) (ii)

q (x)

so polinmios, ento

(p + q ) (x) = p (x) + q (x) (pq ) (x) = p (x)q (x) + p(x)q (x)


Sugesto: Faa primeiro para monmios.

Denimos uma uma raiz

raiz mltipla
( x a)
2

de um polinmio divide

tal que

p(x) como sendo p(x). Caso a seja uma raiz

que no raiz mltipla, dizemos que ela 14. Mostre que se,

raiz simples.

raiz

a raiz mltipla de um polinmio p(x) se, e somente de p(x) e de p (x).

Sugesto: Use o exerccio anterior.

15. Para quais valores de (a) (b)

nN

tem-se que divisvel por divisvel por

1 + x2 + x4 + . . . + x2n2 1 + x3 + x6 + . . . + x3n3

1 + x + . . . + xn1 ? 1 + x + . . . + xn1 ?

(c) Generalize. 16. (a) Resolva a equao que

1 2

20x3 30x2 + 12x 1 = 0,

sabendo-se

uma de suas razes.

(b) Uma raiz da equao

a + 1,

ache as outras duas.

x3 (2a +1)x2 + a(a +2)x a(a +1) = 0 aZ


para que o polinmio

17. Ache os possveis valores de

a2 x4 + 4x3 + 4ax + 7
seja divisvel por

x + 1.

8.4

Exerccios

275

Um polinmio com coecientes reais no constante

p(x) dito ir-

redutvel

se

constantes.

p(x) = a(x)b(x), ento a(x) ou b(x) so polinmios Quando p(x) no for irredutvel, diremos simples-

mente que ele

redutvel.

Os polinmios irredutveis desempe-

nham papel anlogo no conjunto dos polinmios ao dos nmeros primos em

Z. 1
irredutvel.

18. Prove que todo polinmio de grau 19. Prove que se real, ento

f (x) um polinmio de grau 2 e possui uma raiz f (x) redutvel. f (x)


de grau mpar

20. Mostre que todo polinmio

redutvel.

Um polinmio com coecientes inteiros no constante

p(x) dito

irredutvel sobre Q se p(x) = a(x)b(x) com a(x) e b(x) polinmios


com coecientes racionais, ento constantes. Um teorema importante que descreve uma condio para um polinmio ser irredutvel sobre senstein, que diz:

a(x)

ou

b(x)

so polinmios

o conhecido critrio de Ei-

Teorema 8.9

(Critrio de Eisenstein)

+ an xn um polinmio com coecientes inteiros. Suponha que exista um primo p tal que:

. Seja f (x) = a0 + a1 x +

(a) p an ; (b) p | a0 , p | a1 , . . . , p | an1 ; (c) p2 a0 . Ento, f (x) irredutvel sobre Q.

276

Polinmios

Para uma prova desse resultado veja o livro [2]. Faa os seguintes problemas: 21. Mostre que os seguintes polinmios

f (x)

so irredutveis sobre

Q.
Sugesto: Use o critrio de Eisenstein .

(a) (b) (c)

f ( x) = x4 + 2 x3 + 2 x2 + 2 x + 2 ; f (x) = x6 + 15; f (x) = x4 + 10x3 + 20x2 + 30x + 22. Q.

22. Determine quais dos polinmios abaixo so irredutveis sobre


Sugesto: Use o critrio de Eisenstein .

(a) (b) (c)

x3 x + 1 x3 + 2x + 10 x4 x + 1

O problema a seguir trata do polinmio de interpolao de Lagrange. 23. Demonstre a proposio a seguir:

Polinmio de Interpolao de Lagrange.


R, i = 1, 2, . . . , n,
com os

Sejam

ai s

dois a dois distintos e

ai , bi em os bi s nem

todos nulos. Considere os polinmios

p i ( x) = b i

(x ai ) (x ai1 )(x ai+1 ) (x an ) (ai a1 ) (ai ai1 )(ai ai+1 ) (ai an )

8.4

Exerccios

277

para

i = 1, 2, . . . , n.

Ento, o polinmio

p(x) =
i=1

p i ( x) n,
tal que

o nico polinmio de grau menor que para todos

p(ai ) = bi ,

i = 1, 2, . . . , n. p(x)
de grau 7 tal que e

24. Determine o polinmio

p(1) = p(2) = = p(7) = 8

p(0) = 1.

278

Polinmios

A
Apndice: Funes

Estamos acostumados a expresses cotidianas que retratam uma

o quanto Joo ganha funo do que ele trabalha , ou ainda a distncia que percorremos uma funo da velocidade e do tempo que viajamos . Essas e outras
relao entre grandezas, como por exemplo, expresses ilustram a noo de funo como uma relao entre grandezas de dois conjuntos dados. Matematicamente, a noo de funo foi melhor entendida muito recentemente, com os avanos tericos ocorridos no nal do sculo XIX e incio do sculo XX. Entretanto, o seu uso como instrumento e os estudos para tornar sua denio um objeto claro so bem antigos e datam pelo menos desde o incio do clculo diferencial, onde a noo de funo era por vezes entendida como sua

expresso analtica.

O entendimento dessa noo foi crucial para o

avano da Matemtica e importante que o estudante de Matemtica tenha claro seu signicado. Para iniciar a discusso um pouco mais formalmente da noo de funo, vamos denir intuitivamente uma funo como um objeto matemtico composto de trs ingredientes: um conjunto no vazio

A,

279

280

Apndice: Funes

chamado de de

domnio da funo, um conjunto no vazio B , chamado contradomnio da funo e uma correspondncia, que associa a

cada elemento do primeiro conjunto um nico elemento do segundo conjunto. O trio domnio, contradomnio e correspondncia damos o nome de funo. Para simplicar o seu uso, foi criada uma notao que empacota todos os trs ingredientes. Denotamos uma funo por

f : AB x f (x)
para indicar que um elemento de

A o domnio, B o contradomnio e que se x A ento a ele associaremos o elemento f (x) de B .


Para caracterizar uma funo, precisamos dar

importante no confundir uma funo com sua expresso analtica, quando esta dada.

seus trs ingredientes: domnio, contradomnio e correspondncia, e no somente a correspondncia

y = f (x).

Exemplo A.1. Seja a funo f denida de modo que o seu domnio


o conjunto dos nmeros naturais e o contradomnio o conjunto dos nmeros naturais, e a correspondncia tal que a cada nmero natural n associamos o seu quadrado n2 . Observe que podemos denotar isso compactamente por:
f : NN n n2

Veja tambm que se dermos simplesmente a expresso analtica x x2 ou y = x2 para nossa funo, ela no estaria caracterizada, pois no saberamos qual o domnio e o seu contradomnio.
Em alguns casos onde o domnio e o contradomnio esto xados e claros para o interlocutor, podemos nos referir a uma funo simplesmente invocando sua correspondncia

y = f (x).

281

mado pelas pessoas do Brasil e o segundo conjunto como sendo o conjunto L das letras do alfabeto. A correspondncia ser a seguinte: a cada pessoa do Brasil, associaremos a primeira letra do seu nome. Assim, uma pessoa chamada Mrio, ser associada letra M. Em notao de funo:
f : P L x f ( x)

Exemplo A.2. Considere o domnio como sendo o conjunto P for-

onde f (x) a primeira letra do nome de x. pontos de uma sala de aula e o contradomnio como sendo os nmeros reais. A cada ponto x da sala de aula associamos sua temperatura t(x) em um dado momento, medida por um termmetro instalado na sala. Observe que t : S R assim denida uma funo, pois cada ponto possui uma nica temperatura bem denida no instante xado, que um nmero real. Por outro lado, se trocarmos os papis do domnio e contradomnio e a cada nmero real associamos o ponto da sala que tem aquela temperatura, no teremos uma funo, pois pode haver mais de um ponto com a dada temperatura ou ainda uma temperatura que no atingida por nenhum ponto da sala. uma funo, isto , cuja a nossa aparente correspondncia no de fato uma correspondncia, pois no associa a cada elemento x do domnio um nico elemento f (x) do contradomnio. Para tanto, xe o domnio como sendo o conjunto dos nmeros reais no intervalo [0, 1] e como contradomnio o conjunto denido pelas sequncias de elementos no conjunto {0, 1, 2, . . . , 9}. Ou seja,

Exemplo A.3. Considere o domnio S como sendo o conjunto dos

Exemplo A.4. Vamos agora dar outro exemplo em que no temos

282

Apndice: Funes

Cada elemento x [0, 1] possui uma expanso decimal x = 0, x1 x2 x3 . . . . Dena f : [0, 1] colocando f (x) = (x1 , x2 , x3 , . . . ). A princpio, parece que f denida desse modo uma funo. Porm, olhando de perto vemos que o nmero 0, 1 possui mais de uma representao na base decimal, pois 0, 1 = 0, 09999 . . . . Portanto, f no est bem denida, isto , f no associa a cada elemento de [0, 1] um nico elemento de .

= (a1 , a2 , a3 , . . . ); ai {0, 1, 2, 3, . . . , 9} .

Denio A.5. Dada uma funo f denida por


f : AB x f ( x)

o conjunto imagem de f o subconjunto f (A) do contradomnio B formado pelos pontos y do contradomnio tais que existe algum ponto x no domnio A tal que y = f (x). Ou seja imagem de um ponto x A o ponto f (x). Denimos tambm a restrio de f a um subconjunto A de seu domnio a nova funo
A denida considerando-se o domnio como sendo o conjunto

f (A) = {y B ; existe x A tal que y = f (x)}.

e os

demais elementos os mesmos. Denotamos essa nova funo por ou ainda

f |A

No Exemplo A.2 poderamos trocar o domnio por um de seus subconjuntos no vazios. Por exemplo, poderamos considerar o subconjunto Alagoas.

f |A : A B x f ( x)

de

formado pelas pessoas do Brasil que nasceram em

283

Denio via Relaes


Um modo mais formal de denir funo usar a noo de relao entre dois conjuntos um subconjunto relao

B.

Uma relao entre

entre

R do produto cartesiano A e B que satisfaz duas condies:


e

A e B simplesmente A B . Uma funo uma


tais que

R unvoca: dados x1 , x2 A (x2 , y ) R ento x1 = x2 ; R total:


que dado dado

yB

( x1 , y ) R

x A existe y B tal que (x, y ) R. de modo x A, existe um nico y B tal que (x, y ) R.

Funes Injetoras, Sobrejetoras e Bijetoras


Denio A.6. Uma funo f : A B dita injetora (ou
se a seguinte propriedade vale: Dados x, y A tais que f (x) = f (y ), ento x = y. Outro modo equivalente de formular tal propriedade usando sua forma contrarrecproca: Se x, y A so tais que x = y, ento f (x) = f (y ). no injetora, pois f (1) = (1)2 = 12 = f (1). Por outro lado, se g : [0, +) R dada por g (x) = x2 , ento g injetora, pois dados dois nmeros no negativos a e b tais que g (a) = g (b), isto , a2 = b2 , ento a2 b2 = 0, de onde (a b)(a + b) = 0, restando as possibilidades a = b ou a = b. Como a e b so positivos, temos que a = b.
injetiva

Exemplo A.7. Por exemplo, a funo f : R R dada por f (x) = x2

284

Apndice: Funes

Denio A.8. Uma funo f : A B dita


brejetiva

sobrejetora

(ou

so-

) se a seguinte propriedade vale:

Dado y B existe x A, tal que f (x) = y.

Exemplo A.9. Por exemplo, a funo f : R R dada por f (x) =


x2 do exemplo anterior no sobrejetora, pois no existe nenhum nmero real x tal que f (x) = 1, por exemplo. Por outro lado, se considerarmos g (x) : R [0, +) dada por g (x) = x2 , ento g sobrejetora, pois dado qualquer nmero no negativo b, podemos tomar a como sendo a = b de modo que g (a) = a2 = b.

Denio A.10. Uma funo dita


ela injetora e sobrejetora.
Por exemplo, a funo

bijetora

(ou ainda

bijetiva

) se

f : R R dada por f (x) = x3 uma funo bijetora, pois injetora e sobrejetora, j que dado y R, existe um 3 nico x R tal que y = x . Quando f : A B bijetora, ento dado qualquer elemento y B , existe um elemento x A tal que f (x) = y (pois f sobrejetora) e esse elemento nico (pois f injetora). Em outros termos, podemos denir uma nova funo: g : B A associando a cada elemento y B o nico elemento x em A tal que f (x) = y . Em outras palavras, g (y ) = x, g
chamada de se e somente se,

f (x) = y.

funo inversa de f .
f
entre dois conjuntos

Quando existe uma bijeo que

B,

dizemos

tm a mesma quantidade de elementos ou

cardinalidade.

Para mais informaes sobre funes, recomendamos a leitura de [3].

Referncias Bibliogrcas
[1] AIGNER, M. e ZIEGLER, G. (2002).

As Provas esto Elementos de l-

no Livro. gebra.

Edgard Blcher.

[2] GARCIA, A. e LEQUAIN, I. (2003). Projeto Euclides, IMPA.

[3] LIMA, E. L.; CARVALHO, P. C. P.; WAGNER, E. e MORGADO, A.C. (2004).

A Matemtica do Ensino M-

dio. Volume 1.

Sociedade Brasileira de Matemtica.

[4] LIMA, E.L.; CARVALHO, P. C. P.; WAGNER, E. e MORGADO, A.C. (2004).

A Matemtica do Ensino M-

dio. Volume 2.
[5] LIMA,E.L.;

Sociedade Brasileira de Matemtica. C. P.; WAGNER,E. e

CARVALHO,P.

MORGADO,A.C. (2004).

A Matemtica do Ensino M-

dio. Volume 3.

Sociedade Brasileira de Matemtica.

[6] LIMA, E.L.; CARVALHO, P. C. P.; WAGNER,E. e MORGADO, A.C. (2001).

Temas e Problemas.

Socie-

dade Brasileira de Matemtica. [7] LIMA, E.L. (2001). de Matemtica.

lgebra Linear. Sociedade Brasileira

285

286

REFERNCIAS BIBLIOGRFICAS

[8] MORAIS FILHO, D. C. (2007).

Um Convite MatemCARVALHO, P.;

tica.

EDUFCG. CARVALHO, J.;

[9] MORGADO, A.;

FERNANDEZ, P. (1991).

Anlise Combinatria e ProNmeros Primos: Mistrios e Introduo Teoria dos N-

babilidade . Recordes. meros.

Sociedade Brasileira de Matemtica.

[10] RIBENBOIM, P. (2001).

Sociedade Brasileira de Matemtica.

[11] SANTOS, J. P. O. (1993) IMPA.

[12] SANTOS, J. P. O.; MELLO, M. P. e MURARI, I. T. C. (2006). Unicamp. [13] SOARES, M. G. (2005).

Introduo Anlise Combinatria.

Editora

Clculo em uma Varivel Com-

plexa.

Sociedade Brasileira de Matemtica.

Resolu c ao de Problemas Lista 01


Relembramos algumas dicas discutidas no livro-texto para ajudar na resolu c ao de um problema em Matem atica. (D1) Ler bem o enunciado do problema e utilizar todas as informa c oes dispon veis. (D2) Resolver casos particulares ou casos mais simples de problemas similares, para adquirir familiaridade com o problema. (D3) Mudar a representa c ao do problema, transformando-o em um problema equivalente. (D4) Usar a imagina c ao, pesquisando caminhos alternativos.

Al em dessas dicas, vamos acrescentar mais algumas outras:

(D5) Re una todos os fatos que voc e conhece e que estejam relacionados ao problema; Em seguida, selecione alguns deles que voc e ache que possam servir para resolver o problema. (D6) Se o problema permitir, ache um invariante. Isto e, uma quantidade ou propriedade que n ao muda e que possa ser usada para resolver seu problema. (D7) Se for poss vel e conveniente, comece de tr as-para-frente, iniciando na situa c ao nal onde voc e deseja chegar e, a partir de jogadas v alidas, tente chegar na situa c ao inicial. (D8) Antes de avan car demais, cheque o trabalho j a desenvolvido, vericando se voc e cometeu algum erro. Isso lhe poupar a tempo. Claramente, nenhuma dessas dicas faria sentido se lida isoladamente e desconectada de um problema espec co. Al em disso, nenhuma delas e universal, permitindo-lhe tornar-se um grande resolvedor de problemas. Na verdade, resolver problemas e algo que s o se aprende fazendo. Por em, adotar estrat egias como as que listamos acima pode ajud a-lo a ganhar tempo. A seguir, vamos listar alguns problemas para que voc e tente empregar as estrat egias descritas acima e, eventualmente, outras que voc e venha a criar. Boa sorte!

Buscando um Invariante
Em algumas situa c oes, a busca de uma quantidade ou de uma propriedade que n ao muda quando um processo ocorre, pode levar ` a solu c ao do problema. Essa quantidade e chamada de invariante. Isso e particularmente verdadeiro quando consideramos problemas quem involvem impossibilidades, como os que vamos descrever abaixo. Aplique a dica (D6) aos problemas 5, 6, 7 e 8. 1. Seis pessoas formando um c rculo seguram pequenos quadros em suas m aos, nos quais est ao escritos n umeros. A cada rodada, escolhe-se uma das pessoas e adiciona-se uma unidade ao n umero escrito no quadro dessa pessoa, bem como uma unidade aos n umeros nos quadros de seus vizinhos. (a) Se os n umeros iniciais forem 1,0,1,0,0,0 e poss vel, ap os repetir esse procedimento um certo n umero de vezes, fazer com que todos os quadros tenham os mesmos n umeros? (b) Se os n umeros iniciais forem 5,2,0,3,5,6 mostre que e poss vel, ap os repetir esse procedimento um certo n umero de vezes, fazer com que todos os quadros tenham os mesmos n umeros. (c) Se os n umeros iniciais forem 5,2,0,3,5,6, qual o menor n umero de jogadas de modo que todos os quadros tenham os mesmos n umeros? 2. Invente e resolva um problema, usando como inspira c ao o problema ante apropriado para que rior. Decida o grau de diculdade da resolu c ao. E ano? Discuta com seus colegas do PROFMAT. 3. Num tabuleiro de xadrez 8 8 e permitido escolher um quadrado 2 2 qualquer e trocar as cores de uma das linhas ou colunas deste quadrado. poss E vel que se chegue a uma situa c ao na qual todos os quadrados do tabuleiro 8 8 sejam brancos, exceto um? 4. Invente e resolva um problema, usando como inspira c ao o problema ante apropriado para que rior. Decida o grau de diculdade da resolu c ao. E ano? Discuta com seus colegas do PROFMAT. 5. Os n umeros 1, 2, 3, . . . , 99 s ao escritos no quadro-negro e e permitido realizar a seguinte opera c ao: apagar dois deles e substitu -los pela diferen ca do maior com o menor. Fazemos esta opera c ao sucessivamente at e restar apenas um u ltimo n umero no quadro. Pode o u ltimo n umero que restou ser o zero?

Resolvendo de Tr as-para-frente
Andar para tr as pode ser uma ferramenta importante na solu c ao de um problema. A t ecnica consiste em supor que chegamos ao objetivo que pretendemos (posi c ao vitoriosa, por exemplo, ou uma congura c ao especial) e, fazendo movimentos para tr as, tentamos chegar na situa c ao inicial. Resolva o seguinte problema usando a dica (D7): permitido a cada 6. Jo ao e Maria brincam com um monte de 30 palitos. E um deles retirar no seu turno 1, 2 ou 3 palitos. Ganha quem retirar o u ltimo palito. Sabendo que Jo ao come ca e que os dois aprenderam a jogar com o mestre sabetudo, quem ganha o jogo? E retirando-se 1,2,3 ou 4 palitos, quem ganharia? O jogo anterior se inclui numa classe chamada de jogos progressivamente nitos. Esta classe e constituida de jogos que necessariamente acabam ap os um n umero de jogadas. Outro exemplo deste tipo de jogo e o seguinte: 7. Suponha agora que na pilha existam 107 palitos e que a cada rodada, Jo ao e Maria se alternam, escolhendo um primo p, n 0 e inteiro n ao-negativo e retirando pn palitos. Ganha quem retirar o u ltimo palito. Pergunta-se: se Jo ao come ca o jogo, quem ganha? 8. Agora, Jo ao e Maria disp oem de dois montes com 30 palitos cada. Em cada turno, o jogador escolhe somente um dos montes e retira quantos palitos quiser, inclusive o monte inteiro. Ganha quem retirar o u ltimo palito. Sabendo que Jo ao come ca, quem ganha o jogo? 9. Jo ao e Maria se alternam desenhando diagonais de um 2012- agono. Perde quem desenhar uma diagonal cruzando alguma outra j a desenhada. Qual e a estrat egia vitoriosa para esse jogo?

Usando equa c oes


10. Demonstre que: (a) n4 + 4 n ao e primo se n > 1; (b) Generalize, mostrando que n4 + 4n n ao e primo, para todo n > 1. 11. Nove c opias de certas notas custam menos de R$ 10,00 e dez c opias das mesmas notas (com o mesmo pre co) custam mais de R$ 11,00. Quanto custa uma c opia das notas? 12. As p aginas de um livro s ao numeradas de 1 at e n. Ao somarmos estes n umeros, por engano um deles e somado duas vezes, obtendo-se o resultado incorreto: 2.012. Qual e o n umero da p agina que foi somado duas vezes?

13. Analise varia c oes deste problema. Por exemplo, permita que se some dois n umeros consecutivos duas vezes, ou ainda, o mesmo n umero tr es vezes. Mude o valor da soma. Para cada mudan ca que voc e zer, discuta com seus colegas do PROFMAT a s erie onde um desao desse poderia ser proposto. 14. Mostre que entre os ret angulos com um mesmo per metro, o de maior area e um quadrado. 15. Mostre que para quaisquer a, b, c reais vale a2 + b2 + c2 ab + ac + bc. 16. Usando cada d gito 1,2,3,4,5,6,7,8, 9 somente uma vez, decida se e poss vel escrever n umeros de modo que sua soma seja 100. 17. Invente e resolva um problema, usando como inspira c ao o problema ante apropriado para que rior. Decida o grau de diculdade da resolu c ao. E ano? Discuta com seus colegas do PROFMAT. 18. Jo ao est a na beira de um rio com dois baldes de 9 e 4 litros, sem marca c oes. Ele deseja medir exatamente 6 litros de agua para poder levar para Maria fazer uma deliciosa sopa para sua numerosa fam lia. Mostre como Jo ao dever a proceder para obter os 6 litros de agua. 19. Analise varia c oes deste problema. Por exemplo, permita que a quantidade a ser separada seja diferente de 6, ou ainda, que os baldes tenham outra capacidade. Por exemplo, discuta o que ocorre quando os baldes tem MDC diferente de 1. Decida o grau de diculdade da resolu c ao de cada apropriado para que ano? Discuta com seus problema que voc e criar. E colegas do PROFMAT. 20. Fa ca mentalmente as seguintes multiplica c oes: (a) 27 37 (b) 21 23 21. Invente e resolva um problema, usando como inspira c ao o problema ante apropriado para que rior. Decida o grau de diculdade da resolu c ao. E ano? Discuta com seus colegas do PROFMAT.

Sugest oes de Leitura


Revista Eureka! Na revista Eureka! e poss vel encontrar v arios problemas interessantes e desaadores, al em de artigos que tratam de assuntos que n ao s ao rotineiros na sala de aula. A revista est a dispon vel no site www.obm.org.br, onde podem ser encontradas as provas (muitas resolvidas) da Olimp ada Brasileira de Matem atica. Cole c ao de Inicia c ao Cient ca J unior da OBMEP A cole c ao e destinada aos alunos premiados na Olimp ada Brasileira de Matem atica das Escolas P ublicas. Pode ser baixada em http://www.obmep.org.br, onde e poss vel tamb em obter um banco de problemas e solu c oes. Tio Petros http://tiopetrus.blogia.com/ Um blog interessante, com v arios recursos que podem ser aproveitados. Cabe ao leitor uma leitura cr tica. Al em da cole c ao Olimp adas de Matem atica e da cole c ao Professor de Matem atica, que podem ser adquiridos no site www.sbm.org.br, recomendo a leitura dos seguintes livros: How to solve it - G. Polya, Princeton Science Library. Princeton University Press, 1945. Excelente livro sobre did atica da Matem atica e a arte de resolver problemas. O autor apresenta seu ponto de vista sobre as t ecnicas de resolu c ao de problemas, com alguns exemplos discutidos. A discuss ao envolve problemas de todos os n veis (Fundamental e M edio). Sem tradu c ao (que eu conhe ca) para o portugu es. Mathematical Circles: Russian Experience (Mathematical World, Vol. 7) Dimitri Fomin, Sergey Genkin, Ilia V. Itenberg. Excelente livro, com v arios problemas interessantes para ampliar a vis ao do estudante. Est a dividido em duas partes, sendo a primeira dedicada aos primeiros anos do Fundamental II. Os problemas s ao daqueles que consideramos de racioc nio, privilegiando a criatividade e ideias novas dos estudantes. Pode ser usado como livro para um clube ou grupo especial de treinamento de Matem atica na sua escola. Problem Solving Strategies - A. Engel. Problem Books in Mathematics, Springer. Uma colet anea de exerc cios, em geral do n vel de olimp adas de matem atica. Est a separado por temas (geometria, indu c ao etc) e com dicas e resolu c oes para os exerc cios propostos. Pode ser usado numa turma avan cada de prepara c ao para olimp adas ou exames dif ceis.

Resolu c ao de Problemas Lista 01 com dicas e discuss ao


Fa ca mentalmente as seguintes multiplica c oes: 1. 27 37 2. 21 23 Invente e resolva um problema, usando como inspira c ao o problema anterior. apropriado para que ano? Discuta Decida o grau de diculdade da resolu c ao. E com seus colegas do PROFMAT. Dica 1. Para o item a), note que 3 37 = 111. Para o item b), lembre-se que 212 = 441. Discuss ao 1. A ideia desse problema e simplesmente ilustrar que conhecimento pr evio relacionado ao que procuramos pode ajudar na resolu c ao de um problema. Creio que e o tipo problema que pode ser usado em qualquer s erie, especialmente as menores do Fundamental II. Discuta com seus colegas a respeito. Come car do zero um problema pode tornar mais dif cil encontrar a solu c ao do problema. Uma avalia c ao criteriosa de fatos conhecidos ou de problemas semelhantes pode nos ajudar a encontrar a solu c ao mais rapidamente.

Buscando um Invariante
Em algumas situa c oes, a busca de uma quantidade ou de uma propriedade que n ao muda quando um processo ocorre, pode levar ` a solu c ao do problema. Essa quantidade e chamada de invariante. Isso e particularmente verdadeiro quando consideramos problemas quem involvem impossibilidades, como os que vamos descrever abaixo. Aplique a dica (D6) aos problemas 5, 6, 7 e 8. 1. Seis pessoas formando um c rculo seguram pequenos quadros em suas m aos, nos quais est ao escritos n umeros. A cada rodada, escolhe-se uma das pessoas e adiciona-se uma unidade ao n umero escrito no quadro dessa pessoa, bem como uma unidade aos n umeros nos quadros de seus vizinhos. (a) Se os n umeros iniciais forem 1,0,1,0,0,0 e poss vel, ap os repetir esse procedimento um certo n umero de vezes, fazer com que todos os quadros tenham os mesmos n umeros? (b) Se os n umeros iniciais forem 6,3,0,0,3,6 mostre que e poss vel, ap os repetir esse procedimento um certo n umero de vezes, fazer com que todos os quadros tenham os mesmos n umeros. (c) Se os n umeros iniciais forem 6,3,0,0,3,6, qual o menor n umero de jogadas de modo que todos os quadros tenham os mesmos n umeros?

Dica 2. Observe o resto na divis ao por 3 da soma dos n umeros nos quadros e um invariante. imDiscuss ao 2. A no c ao-chave aqui e o que vem a ser um invariante. E portante frisar que algo n ao muda quando realizamos uma jogada. Neste caso, para o primeiro item, o invariante em quest ao e o resto da divis ao da soma total por 3. Como a soma da congura c ao inicial deixa resto 2 quando dividida por 3, temos que e imposs vel chegar na situa c ao onde todos os n umeros dos quadros s ao iguais come cando com 1,0,1,0,0,0, j a que se todos os n umeros s ao iguais, o resto da soma e zero quando dividida por 3. Para o segundo item, basta fazer uma constru c ao. Em geral, o aluno come ca com o processo tentativa-e-erro, at e pegar o jeito e inferir que jo importante guiar o aluno gadas s ao interessantes para atingir o objetivo. E neste processo, estimulando a infer encia de propriedades gerais. Isso permite, por exemplo, ter ideias sobre o terceiro item. Por exemplo, ele pode perceber que os n umeros s o aumentam a cada jogada. Isso permite inferir que o n umero m nimo de jogadas deve ser maior igual a (36-18)/3=6. Isso se deve ao fato que a primeira congura c ao que talvez seja poss vel que igual todos os n umeros e 6,6,6,6,6,6 (soma 36) e que a congura c ao inicial 6,3,0,0,3,6 tem soma 18. Assim, como a cada jogada acrescentamos 3, o m nimo e maior ou igual a 6. De fato, 6 e o m nimo pois uma sequ encia que resolve o problema e 6, 3, 0, 0, 3, 6 6, 4, 1, 1, 3, 6 6, 4, 2, 2, 4, 6 6, 5, 3, 3, 4, 6 6, 5, 4, 4, 5, 6 6, 6, 5, 5, 5, 6 6, 6, 6, 6, 6, 6. 2. Invente e resolva um problema, usando como inspira c ao o problema ante apropriado para que rior. Decida o grau de diculdade da resolu c ao. E ano? Discuta com seus colegas do PROFMAT. 3. Num tabuleiro de xadrez 8 8 e permitido escolher um quadrado 2 2 qualquer e trocar as cores de uma das linhas ou colunas deste quadrado. poss E vel que se chegue a uma situa c ao na qual todos os quadrados do tabuleiro 8 8 sejam brancos, exceto um? Dica 3. Olhe a paridade da diferen ca entre o n umero de quadrados brancos e quadrados pretos. Discuss ao 3. A paridade da diferen ca entre o n umero de quadrados brancos e quadrados pretos n ao muda a cada opera c ao permitida. Como no in cio temos 32 quadrados brancos e 32 quadrados pretos, a cada passo teremos sempre um n umero PAR como resultado da diferen ca entre os quadrados brancos e os quadrados pretos, mostrando que e imposs vel atingir a congura c ao pedida.

4. Invente e resolva um problema, usando como inspira c ao o problema ante apropriado para que rior. Decida o grau de diculdade da resolu c ao. E ano? Discuta com seus colegas do PROFMAT. 5. Os n umeros 1, 2, 3, . . . , 99 s ao escritos no quadro-negro e e permitido realizar a seguinte opera c ao: apagar dois deles e substitu -los pela diferen ca do maior com o menor. Fazemos esta opera c ao sucessivamente at e restar apenas um u ltimo n umero no quadro. Pode o u ltimo n umero que restou ser o zero?

Resolvendo de Tr as-para-frente
Andar para tr as pode ser uma ferramenta importante na solu c ao de um problema. A t ecnica consiste em supor que chegamos ao objetivo que pretendemos (posi c ao vitoriosa, por exemplo, ou uma congura c ao especial) e, fazendo movimentos para tr as, tentamos chegar na situa c ao inicial. Resolva o seguinte problema usando a dica (D7): permitido a cada 6. Jo ao e Maria brincam com um monte de 30 palitos. E um deles retirar no seu turno 1, 2 ou 3 palitos. Ganha quem retirar o u ltimo palito. Sabendo que Jo ao come ca e que os dois aprenderam a jogar com o mestre sabetudo, quem ganha o jogo? E retirando-se 1,2,3 ou 4 palitos, quem ganharia? Dica 4. Para o jogo retirando-se 1,2 ou 3 palitos, tente resolver o problema fazendo uso do m nimo de itens da lista abaixo. (a) Reduza o n umero de palitos no monte. (b) Tente com 7 e depois passe para 10 palitos. (c) Analise o que acontece quando seu advers ario est a na posi c ao 4. (d) Analise o que acontece quando seu advers ario est a na posi c ao 8. (e) Analise o que acontece quando seu advers ario est a em uma posi c ao m ultiplo de 4. Discuss ao 4. A estrat egia vencedora e deixar seu advers ario numa posi c ao m ultiplo de 4. Verique isso tra cando uma arvore de poss veis caminhos a partir de uma dada posi c ao, onde a posi c ao 0 signica vit oria. Isso pode ser feito construindo-se um grafo orientado, isto e, um conjunto de pontos (posi c oes) e setas indicando as jogadas permitidas. O jogo anterior se inclui numa classe chamada de jogos progressivamente nitos. Esta classe e constituida de jogos que necessariamente acabam ap os um n umero de jogadas. Outro exemplo deste tipo de jogo e o seguinte:

7. Suponha agora que na pilha existam 107 palitos e que a cada rodada, Jo ao e Maria se alternam, escolhendo um primo p, n 0 e inteiro n ao-negativo e retirando pn palitos. Ganha quem retirar o u ltimo palito. Pergunta-se: se Jo ao come ca o jogo, quem ganha? Dica 5. Observe que o primeiro n umero que n ao e pot encia de primo eo 6. Ou seja, se Jo ao deixa Maria com 6 palitos na jogada dela, Jo ao ganha. Discuss ao 5. O conjunto de posi c oes vitoriosas consiste nos m ultiplos de 6. De fato, se Jo ao deixa Maria com um m ultiplo de 6 palitos na jogada dela, depois de Maria retirar uma pot encia de primo, Jo ao pode retornar a uma posi c ao que e m ultiplo de 6 retirando 1,2,3,4 ou 5 palitos. Assim, no nal Maria estar a na posi c ao 6 e Jo ao ganhar a o jogo. 8. Agora, Jo ao e Maria disp oem de dois montes com 30 palitos cada. Em cada turno, o jogador escolhe somente um dos montes e retira quantos palitos quiser, inclusive o monte inteiro. Ganha quem retirar o u ltimo palito. Sabendo que Jo ao come ca, quem ganha o jogo? Dica 6. Olhe na express ao dos montes quando escritos na base 2. Discuss ao 6. A estrat egia vencedora desse jogo e deixar seu oponente numa situa c ao de modo que quando somamos os d gitos na base dois dos montes relativos a mesma pot encia, o resultado e par. Ou seja, os d gitos s ao iguais. Por exemplo, se os montes t em 15 e 21 palitos, primeiro escrevemos eles na base dois: 15 = (1111)2 Em seguida, escrevemos a soma 10101 +1111 11212 Chamaremos de posi c ao vencedora, aquela que obtiver como resultado das somas das colunas apenas os d gitos 2 e 0. As demais s ao as perdedoras. Por exemplo, a posi c ao acima (um monte com 15 e outro com 21) e posi c ao perdedora. Para vencer o jogo, basta o jogador transformar este resultado (11212) numa posi c ao vencedora, retirando palitos. Por exemplo, podemos retirar 6 palitos do monte com 21 para deix a-lo com 15 tamb em. Assim, a soma coluna a coluna dos d gitos ser a (2222). Para mostrar que a estrat egia e a vitoriosa, voc e deve vericar que A partir de uma posi c ao vencedora, sempre podemos ir para uma posi c ao perdedora. e 21 = (10101)2 .

A partir de uma posi c ao perdedora, nunca podemos ir para uma posi c ao vencedora Como o zero e uma posi c ao vencedora, essa e a estrat egia vitoriosa do jogo. 9. Jo ao e Maria se alternam desenhando diagonais de um 2012- agono. Perde quem desenhar uma diagonal cruzando alguma outra j a desenhada. Qual e a estrat egia vitoriosa para esse jogo? Dica 7. Observe que o que acontece quando um jogador tra ca uma diagonal principal, isto e, aquela que divide o pol gono em dois pol gonos com a mesma quantidade de v ertices. Discuss ao 7. Como temos uma quantidade par de v ertices, e poss vel tra car alguma diagonal principal. Digamos que Jo ao tenha tra cado uma diagonal principal, dividindo o 2012- agono em dois 1006- agonos, P1 e P2 . A seguir, observe que as diagonais que n ao cruzam a diagonal principal corresponde exatamente ` as diagonais de um dos dois pol gonos obtidos. Assim, para cada diagonal de Maria que n ao cruza a diagonal principal, Jo ao poder a tra car uma diagonal correspondente no outro pol gono. Ao nal, Maria ter a que cruzar a diagonal principal tra cada.

Usando equa c oes do primeiro e segundo graus


10. Nove c opias de certas notas custam menos de R$ 10,00 e dez c opias das mesmas notas (com o mesmo pre co) custam mais de R$ 11,00. Quanto custa uma c opia das notas? 11. As p aginas de um livro s ao numeradas de 1 at e n. Ao somarmos estes n umeros, por engano um deles e somado duas vezes, obtendo-se o resultado incorreto: 2.012. Qual e o n umero da p agina que foi somado duas vezes? Discuss ao 8. Primeiramente, relembre que a soma dos n primeiros n umeros e S = n(n + 1)/2. Observe tamb em a soma efetuada incorretamente est a entre S + 1 e S + n. Assim, sabemos que S + 1 2012 S + n, ou seja, n2 + n + 2 4024 n2 + 3n Testando os valores de n2 + n + 2 temos que n = 62, j a que 61 e pouco e 63 e demais e a fun c ao e crescente. Portanto, o valor desejado e obtido fazendo 2012 S = 58. 12. Analise varia c oes deste problema. Por exemplo, permita que se some dois n umeros consecutivos duas vezes, ou ainda, o mesmo n umero tr es vezes. Mude o valor da soma. Para cada mudan ca que voc e zer, discuta com seus colegas do PROFMAT a s erie onde um desao desse poderia ser proposto.

13. Determine os valores de a para os quais a fun c ao quadr atica ax2 ax + 12 e sempre positiva. 14. Mostre que entre os ret angulos com um mesmo per metro, o de maior area e um quadrado. 15. Mostre que para quaisquer a, b, c reais vale a2 + b2 + c2 ab + ac + bc. 16. Usando cada d gito 1,2,3,4,5,6,7,8, 9 somente uma vez, decida se e poss vel escrever n umeros de modo que sua soma seja 100. Discuss ao 9. A resposta deste problema e negativa, ou seja, e imposs vel realizar o que se pede. De fato, primeiramente observe que todos os n umeros constru dos t em que ser menores que 100. Portanto, cada n umero tem no m aximo dois d gitos. Digamos que os d gitos que forem usados na casa das unidades dos n umeros constru dos formem um conjunto com soma t. Naturalmente, o conjunto dos n umeros que foram usados nos algarismos das dezenas dos n umeros constru dos formar a um conjunto com soma 45 t, j a que 1 + 2 + 3 + . . . + 9 = 45. Portanto, teremos que a soma ser a (45 t)10 + t = 100, e essa equa c ao do primeiro grau n ao tem solu c oes inteiras. 17. Invente e resolva um problema, usando como inspira c ao o problema ante apropriado para que rior. Decida o grau de diculdade da resolu c ao. E ano? Discuta com seus colegas do PROFMAT. 18. Jo ao est a na beira de um rio com dois baldes de 9 e 4 litros, sem marca c oes. Ele deseja medir exatamente 6 litros de agua para poder levar para Maria fazer uma deliciosa sopa para sua numerosa fam lia. Mostre como Jo ao dever a proceder para obter os 6 litros de agua. 19. Analise varia c oes deste problema. Por exemplo, permita que a quantidade a ser separada seja diferente de 6, ou ainda, que os baldes tenham outra capacidade. Por exemplo, discuta o que ocorre quando os baldes tem MDC diferente de 1. Decida o grau de diculdade da resolu c ao de cada apropriado para que ano? Discuta com seus problema que voc e criar. E colegas do PROFMAT.

Resolu c ao de Problemas Lista 02

Divisibilidade
1. Prove que se n e mpar (a) n2 1 e divis vel por 8; (b) n3 n e divis vel por 24; (c) n2 + (n + 2)2 + (n + 4)2 + 1 e divis vel por 12. 2. Tr es n umeros primos p, q e r, maiores que 3, formam uma progress ao aritm etica, ou seja, q = p + d e r = p + 2d. Prove que d e divis vel por 6.1 3. Mostre que 311 3 e divis vel por 112 . 4. Um rob o possui dois bot oes, permitindo que a cada momento ele suba a degraus ou des ca b degraus de uma escada com innitos degraus. Sabendo que o rob o est a no in cio da escada, pergunta-se: (a) Se a = 12 e b = 3, e poss vel que o rob o visite todos os degraus ap os uma sucess ao desses movimentos? (b) Mostre que se a e b s ao primos entre si, o rob o consegue visitar todos os degraus. 5. Encontre o resto que deixa (a) 2001 2002 2003 2004 + 20052 quando e dividido por 7; (b) 2100 quando e dividido por 3; (c) 1237156 + 34
28

quando e dividido por 111.

6. Encontrar o u ltimo d gito dos n umeros (a) 19892005 ; (b) 777777 + 250 ; (c) 1 + 22 + 32 + + 20052 . 7. Verique que 111 . . . 1 = 222 . . . 2 +(333 . . . 3)2 .
2012 uns
1 Foi

1006 dois

1006 tr es

demonstrado em 2004 pelos Matem aticos B. Green e T. Tao que existem progress oes aritm eticas de tamanho arbitrariamente grande formadas somente por primos. A prova pode ser encontrada em www.arxiv.org.

8. Demonstre que o n umero 1 000 . . . 00 1 e composto.


2012 zeros

9. Considere o polin omio p(n) = am nm + am1 nm1 + + a0 de grau m 1 com coecientes inteiros e n N. Prove que p(n) e um n umero composto para innitos valores de n.2 10. Prove que se (x0 , y0 ) e uma solu c ao da equa c ao diofantina linear ax by = 1, ent ao a area do tri angulo cujos v ertices s ao (0, 0), (b, a) e (x0 , y0 ) e 1/2. 11. Ao entrar numa sala, Jo ao se depara com 100 interruptores e 100 l ampadas, numerados de 1 a 100. O interruptor n acende somente a l ampada n, para cada valor de n = 1, 2, . . . , 100. De in cio, todas as 100 l ampadas est ao apagadas. Jo ao aperta todos os interruptores m ultiplos de 2. A seguir, aperta todos os m ultiplos de 3, e assim sucessivamente, at eou nico interruptor m ultiplo de 100. Ao nal deste procedimento, pergunta-se: (a) Quais l ampadas est ao apagadas? (b) Quantas l ampadas acenderam exatamente 4 vezes? (c) Qual e o n umero da l ampada que mais acendeu?

2 Sugest ao: Use o fato de que existe a N tal que = |p(a)| > 1 e mostre que divide a p(k + a), para todo k Z.

Princ pio das Casas dos Pombos


12. Seja C um conjunto formado por cinco pontos de coordenadas inteiras no plano. Prove que o ponto m edio de algum dos segmentos com extremos em C tem tamb em coordenadas inteiras. 13. O conjunto dos d gitos 1, 2, ..., 9 e dividido em tr es grupos. Prove que o produto dos n umeros de algum dos grupos deve ser maior que 71. 14. Prove que se N e mpar ent ao para qualquer bije c ao p : IN IN do conjunto IN = {1, 2, . . . , N } o produto P (p) = (1p(1))(2p(2)) (N p(N )) e necessariamente par.3 15. Dado um conjunto de 25 pontos no plano tais que entre quaisquer 3 deles existe um par com dist ancia menor que 1. Prove que existe um c rculo de raio 1 que cont em pelo menos 13 dos 25 pontos dados. 16. Marquemos todos os centros dos 64 quadradinhos de um tabuleiro de poss xadrez de 8 8. E vel cortar o tabuleiro com 13 linhas retas que n ao passem pelos pontos marcados e de forma tal que cada peda co de recorte do tabuleiro tenha no m aximo um ponto marcado? 17. Prove que existem duas pot encias de 3 cuja diferen ca e divis vel por 2012. 18. Mostre que entre nove n umeros que n ao possuem divisores primos maiores que cinco, existem dois cujo produto e um quadrado. 19. Um disco fechado de raio um cont em sete pontos, cujas dist ancias entre quaisquer dois deles e maior ou igual a um. Prove que o centro do disco e um destes pontos. 20. Seja a um n umero irracional. Prove que existem innitos n umeros racionais r = p/q tais que |a r| < 1/q 2 .
21. Suponha que cada ponto do reticulado4 plano e pintado de vermelho ou azul. Mostre que existe algum ret angulo com v ertices no reticulado e todos da mesma cor. 22. Um certo livreiro vende pelo menos um livro por dia. Sabendo que o livreiro vendeu 463 livros durante 305 dias consecutivos, mostre que em algum per odo de dias consecutivos o livreiro vendeu exatamente 144 livros.

3 Dica:

4 Reticulado

O produto de v arios fatores e par se, e somente se, um dos fatores e par. refere-se ao conjunto dos pontos do plano com coordenadas inteiras

Resolu c ao de Problemas Lista 02

Solu c oes dos Exerc cios de Divisibilidade


1. Prove que se n e mpar (a) n2 1 e divis vel por 8; Demonstra c ao. Como n e mpar, temos que n = 2q + 1 para algum q Z. Assim, n2 1 = (2q + 1)2 1 = 4q 2 + 4q + 1 1 = 4q 2 + 4q = 4q (q + 1). Como q e q + 1 s ao consecutivos, um deles tem que ser par. Logo, n2 1 e divis vel por 8.

(b) n3 n e divis vel por 24; Demonstra c ao. Usaremos o fato que se a, b s ao primos entre si, ent ao c e divis vel por ab se, e somente se, c e divis vel por a e por b. Assim, para mostrar que n3 n e m ultiplo de 24, basta mostrar que n3 n e m ultiplo de 3 e de 8, uma vez que 3 e 8 s ao primos entre si. Observe que n3 n = n(n2 1). Como n e mpar, segue do exerc cio anterior que n3 n e divis vel por 8. A divisibilidade por 3 decorre do lema dos restos. De fato, se n 0 mod 3, n3 n ser a m ultiplo de 3, claramente. Se n 1 mod 3, temos que n3 n ser a m ultiplo de 3, j a que 13 1 0 mod 3. Se n 2 mod 3, temos que n3 n ser a m ultiplo de 3, j a que 23 2 0 completando assim a prova. mod 3,

(c) n2 + (n + 2)2 + (n + 4)2 + 1 e divis vel por 12.

Demonstra c ao. Novamente, usaremos o fato que se a, b s ao primos entre si, ent ao c e divis vel por ab se, e somente se, c e divis vel por a e por b. Assim, para mostrar que a express ao e m ultiplo de 12, basta mostrar que 3 e de 4. Vamos primeiro ` a divisibilidade por 4. Observe que como n e mpar, basta vericar os casos quando n 1 mod 4 e n 3 mod 4. Quando n 1 mod 4 temos que n2 + (n + 2)2 + (n + 4)2 + 1 12 + (1 + 2)2 + (1 + 4)2 + 1 36 0 Do mesmo modo, quando n 3 mod 4 temos que n2 + (n + 2)2 + (n + 4)2 + 1 32 + (3 + 2)2 + (3 + 4)2 + 1 84 0 mod 4. mod 4.

Para vericar a divisibilidade por 3, repetimos o argumento usando os restos da divis ao por 3. Observe que quando n 0 mod 3 temos que n2 + (n + 2)2 + (n + 4)2 + 1 02 + (0 + 2)2 + (0 + 4)2 + 1 21 0 mod 3.

Procedendo de modo inteiramente an alogo para os casos n 1 mod 3 e n 2 mod 3, completamos a prova.

2. Tr es n umeros primos p, q e r, maiores que 3, formam uma progress ao aritm etica, ou seja, q = p + d e r = p + 2d. Prove que d e divis vel por 6.1 Demonstra c ao. Temos que d e par, j a que d = q p, com p e q primos maiores que dois, logo mpares. Basta mostrar que d e divis vel por 3. Ora, supondo o contr ario, temos que d 1 mod 3 ou n 2 mod 3. Analisando o resto de q e r m odulo 3 nas tabelas a seguir (em fun c ao dos restos de d nas colunas e p nas linhas). Para q temos mod 3 p1 p2 Para r temos: mod 3 p1 p2 d1 0 1 d2 1 0. d1 2 0 d2 0 1.

1 Foi demonstrado em 2004 pelos Matem aticos B. Green e T. Tao que existem progress oes aritm eticas de tamanho arbitrariamente grande formadas somente por primos. A prova pode ser encontrada em www.arxiv.org.

Logo, se d n ao e divis vel por 3, ou q ou r ser ao divis veis por 3, o que e imposs vel, j a que s ao ambos n umeros primos.

3. Mostre que 311 3 e divis vel por 112 . 4. Um rob o possui dois bot oes, permitindo que a cada momento ele suba a degraus ou des ca b degraus de uma escada com innitos degraus. Sabendo que o rob o est a no in cio da escada, pergunta-se: (a) Se a = 12 e b = 3, e poss vel que o rob o visite todos os degraus ap os uma sucess ao desses movimentos? Demonstra c ao. Se apertamos o bot ao de subir x vezes e o de descer y vezes, o rob o ir a subir ou descer 12x 3b degraus. Note que esse n umero e sempre m ultiplo de 3, n ao sendo poss vel para o rob o com uma sequ encia desses movimentos atingir um degrau com n umero que n ao e divis vel por 3. (b) Mostre que se a e b s ao primos entre si, o rob o consegue visitar todos os degraus. Demonstra c ao. Como a e b s ao primos entre si, pelo Teorema de Bezout-Bachet, e poss vel encontrar x e y de modo que ax by = 1. Assim, o rob o pode sair do n vel inicial e ir para o degrau 1 apertandose x vezes o bot ao de subir e em seguida apertando-se y vezes o bot ao de descer. Repetindo esse procedimento, o rob o pode atingir qualquer degrau. 5. Encontre o resto que deixa (a) 2001 2002 2003 2004 + 20052 quando e dividido por 7; Demonstra c ao. Vamos usar o lema dos restos. 2001 quando dividido por 7 deixa resto 6; Assim, 2002 e divis vel por 7 e 2005 deixa resto 3 quando dividido por 7. Logo, 20052 deixa resto 2, j a que 32 deixa resto 2 quando dividido por 7. (b) 2100 quando e dividido por 3; Demonstra c ao. 22 resto 1 quando dividido por 3. Logo, 2100 = (22 )5 0 deixa resto 1 quando dividido por 3. (c) 1237156 + 34
28

quando e dividido por 111.

6. Encontrar o u ltimo d gito dos n umeros (a) 19892005 ;

(b) 777777 + 250 ; (c) 1 + 22 + 32 + + 20052 . 7. Verique que 111 . . . 1 = 222 . . . 2 +(333 . . . 3)2 .
2012 uns 1006 dois 1006 tr es

Demonstra c ao. Observe que 111 . . . 1 =


2012 uns

102012 1 9

e que 222 . . . 2 = 2
1006 dois

101006 1 9

(333 . . . 3)2 = 9
1006 tr es

101006 1 9

Portanto, basta vericar que 102012 1 101006 1 101006 1 =2 +9 9 9 9 De fato, cancelando 9 em ambos os lados 102012 1 = 2 (101006 1) + (101006 1)2 , como quer amos demonstrar.
2

8. Demonstre que o n umero 1 000 . . . 00 1 e composto.


2012 zeros

Demonstra c ao. Para termos uma id eia da prova deste fato, vamos provar que 1001 e composto. Ora, temos que 1001 = 11 91. Do mesmo modo, 100001 = 11 9091 e que 10000001 = 11 909091. Assim, depois dessa an alise inicial, ca bem claro que nosso candidato para divisor do n umero pedido e o 11; Podemos vericar sem maiores diculdades que 1 000 . . . 00 1 = 11 9090 . . . 90 91.
2012 zeros 1005 vezes

9. Considere o polin omio p(n) = am nm + am1 nm1 + + a0 de grau m 1 com coecientes inteiros e n N. Prove que p(n) e um n umero composto para innitos valores de n.2 10. Prove que se (x0 , y0 ) e uma solu c ao da equa c ao diofantina linear ax by = 1, ent ao a area do tri angulo cujos v ertices s ao (0, 0), (b, a) e (x0 , y0 ) e 1/2. 11. Ao entrar numa sala, Jo ao se depara com 100 interruptores e 100 l ampadas, numerados de 1 a 100. O interruptor n acende somente a l ampada n, para cada valor de n = 1, 2, . . . , 100. De in cio, todas as 100 l ampadas est ao apagadas. Jo ao aperta todos os interruptores m ultiplos de 2. A seguir, aperta todos os m ultiplos de 3, e assim sucessivamente, at eou nico interruptor m ultiplo de 100. Ao nal deste procedimento, pergunta-se: (a) Quais l ampadas est ao apagadas?
k 1 Demonstra c ao. Dado um n umero natural n = p 1 pk , com pi s primos e i s 0, sabemos que o n umero de divisores de n e

d(n) = k i=1 (i + 1). Para responder essa pergunta, basta analisar quantos divisores cada n umero de 1 a 100 possui. Na verdade, basta vericar a paridade da quantidade de divisores de cada n umero, j a que se d(n) for par, a l ampada n car a acesa. Da express ao acima, e claro que d(n) e mpar se, e somente se, i e par, para cada valor de i. Assim, d(n) e mpar se, e somente se, n e um quadrado. Logo, as l ampadas que car ao apagadas s ao aquelas que t em n umero que e um quadrado perfeito. (b) Quantas l ampadas acenderam exatamente 4 vezes? Demonstra c ao. Basta contar quantas l ampadas tem 8 ou 9 divisores. Fazendo a contagem usando a express ao em produto de primos, obtemos a lista: 24, 30, 36, 40, 42, 54, 56, 66, 70, 78, 88 e 100. (c) Qual e o n umero da l ampada que mais acendeu? Demonstra c ao. Essa l ampada corresponde aos n umeros que tem mais divisores. O problema est a mal-formulado, j a que temos cinco n umeros com exatamente 12 divisores: 60, 72, 84, 90 e 96.

2 Sugest ao: Use o fato de que existe a N tal que = |p(a)| > 1 e mostre que divide a p(k + a), para todo k Z.

Resolu c ao de Problemas Lista 03

1. De quantas maneiras podemos escolher tr es n umeros distintos do conjunto I50 = {1, 2, 3, . . . , 49, 50} de modo que sua soma seja a) um m ultiplo de 3? b) um n umero par? 2. Considere o conjunto In = {1, 2, 3, . . . , n 1, n}. Diga de quantos modos e poss vel formar subconjuntos de k elementos nos quais n ao haja n umeros consecutivos? 3. Considere as letras da palavra PERMUTA. Quantos anagramas de 4 letras podem ser formados, onde: a) n ao h a restri c oes quanto ao n umero de consoantes ou vogais? b) o anagrama come ca e termina por vogal? c) a letra R aparece? d) a letra T aparece e o anagrama termina por vogal? 4. Calcular a soma de todos os n umeros de 5 algarismos distintos formados com os algarismos 1, 3, 5, 7 e 9. 5. Quantos n umeros podem ser formados pela multiplica c ao de alguns ou de todos os n umeros 2, 2, 3, 3, 3, 5, 5, 6, 8, 9, 9? 6. Quantos s ao os n umeros naturais de sete d gitos nos quais o d gito 4 gura exatamente 3 vezes e o d gito 8 gura exatamente 2 vezes? 7. De quantas maneiras uma comiss ao de 4 pessoas pode ser formada, de um grupo de 6 homens e 6 mulheres, se a mesma e composta de um n umero maior de homens do que de mulheres? 8. O comprimento de uma palavra e a quantidade de caracteres que ela possui. Encontre a quantidade de palavras de comprimento 5 que podemos formar fazendo uso de 10 caracteres distintos, de forma que n ao existam tr es caracteres consecutivos id enticos em cada palavra. 9. De quantos modos 6 casais podem sentar-se ao redor de uma mesa circular de tal forma que marido e mulher n ao quem juntos? 10. Quantas s ao as permuta c oes das letras da palavra PROFMAT em que o P ocupa o primeiro lugar, ou o T ocupa o segundo lugar, ou o A o sexto lugar?

11. De quantas formas podemos representar o n umero 15 como soma de v arios n umeros naturais? 12. Quantos quadrados perfeitos existem entre 40.000 e 640.000 que s ao m ultiplos simultaneamente de 3, 4 e 5? 13. Oito amigos v ao ao cinema assistir a um lme que custa um real. Quatro deles possuem uma nota de um real e quatro possuem uma nota de dois reais. Sabendo-se que o caixa do cinema n ao possui nenhum dinheiro, de quantas formas eles podem organizar uma la para pagar o lme permitindo o troco pelo caixa? 14. Encontre o n umero de zeros que termina o n umero 2010!. 15. A fun c ao de Euler associa a cada n umero natural n o valor (n) igual ao n umero de inteiros positivos menores ou iguais a n relativamente primos com n. Ou seja, (n) = {1 m n; (m, n) = 1} . Usando os princ pios estudados, mostre que se n se decomp oe em fatores k 1 2 primos como n = p p . . . p , ent a o 1 2 k (n) = n 1 1 p1 1 1 p2 ... 1 1 pk .

You might also like